Как Ρ‚ΠΎΠΊ ΠΈΠ΄Π΅Ρ‚ Π² Ρ†Π΅ΠΏΠΈ – Π­Π»Π΅ΠΊΡ‚Ρ€ΠΈΠΊΠΈ! ΠžΠ±ΡŠΡΡΠ½ΠΈΡ‚Π΅ ΠΌΠ½Π΅ нСсколько элСмСнтарных ΠΏΠΎ Ρ„ΠΈΠ·ΠΈΠΊΠ΅ Π²Π΅Ρ‰Π΅ΠΉ.. . Описал Π²Π½ΡƒΡ‚Ρ€ΠΈ…

Π‘ΠΎΠ΄Π΅Ρ€ΠΆΠ°Π½ΠΈΠ΅

Как Ρ‡ΠΈΡ‚Π°Ρ‚ΡŒ схСмы. НапряТСниС ΠΈ сила Ρ‚ΠΎΠΊΠ°

Как Ρ‡ΠΈΡ‚Π°Ρ‚ΡŒ схСмы? Π’ ΠΏΡ€ΠΎΡˆΠ»ΠΎΠΉ ΡΡ‚Π°Ρ‚ΡŒΠ΅ ΠΌΡ‹ с Π²Π°ΠΌΠΈ рассмотрСли, ΠΊΠ°ΠΊ выглядят обозначСния основных радиоэлСмСнтов Π½Π° схСмС. Π’ этой ΡΡ‚Π°Ρ‚ΡŒΠ΅ ΠΌΡ‹ ΠΏΠΎΠ³ΠΎΠ²ΠΎΡ€ΠΈΠΌ ΠΎ Ρ‚Π°ΠΊΠΈΡ… понятиях, ΠΊΠ°ΠΊ элСктричСский Ρ‚ΠΎΠΊ, напряТСниС ΠΈ сила Ρ‚ΠΎΠΊΠ°. Π₯отя я ΡƒΠΆΠ΅ писал ΠΎ Π½ΠΈΡ… Π² самых ΠΏΠ΅Ρ€Π²Ρ‹Ρ… ΡΡ‚Π°Ρ‚ΡŒΡΡ…, Π½ΠΎ Π² этой ΡΡ‚Π°Ρ‚ΡŒΠ΅ ΠΏΠΎΠΏΡ€ΠΎΠ±ΡƒΠ΅ΠΌ всС это ΡΠ»ΠΎΠΆΠΈΡ‚ΡŒ Π² ΠΎΠ΄Π½Ρƒ ΠΊΡƒΡ‡Ρƒ, Ρ‡Ρ‚ΠΎΠ±Ρ‹ Π²Π°ΠΌ Π±Ρ‹Π»ΠΎ Π»Π΅Π³Ρ‡Π΅ ΡƒΠ»ΠΎΠ²ΠΈΡ‚ΡŒ ΡΡƒΡ‚ΡŒ Π΄Π΅Π»Π°.

ΠŸΡ€ΠΎΠ²ΠΎΠ΄Π½ΠΈΠΊΠΈ элСктричСского Ρ‚ΠΎΠΊΠ°

НачнСм с самого-самого Π½Π°Ρ‡Π°Π»Π°. Как Π²Ρ‹ Π·Π½Π°Π΅Ρ‚Π΅, всС схСмы состоят ΠΈΠ· ΠΏΡ€ΠΎΠ²ΠΎΠ΄ΠΊΠΎΠ² ΠΈΠ»ΠΈ ΠΏΠ΅Ρ‡Π°Ρ‚Π½Ρ‹Ρ… Π΄ΠΎΡ€ΠΎΠΆΠ΅ΠΊ, ΠΊΠΎΡ‚ΠΎΡ€Ρ‹Π΅ ΡΠΎΠ΅Π΄ΠΈΠ½ΡΡŽΡ‚ Ρ€Π°Π·Π»ΠΈΡ‡Π½Ρ‹Π΅ радиоэлСмСнты Π² Π΅Π΄ΠΈΠ½ΠΎΠ΅ Ρ†Π΅Π»ΠΎΠ΅. НапримСр, Π² ΡΡ‚Π°Ρ‚ΡŒΠ΅ β€œΡΠ°ΠΌΡ‹ΠΉ простой ΡƒΡΠΈΠ»ΠΈΡ‚Π΅Π»ΡŒ Π·Π²ΡƒΠΊΠ°β€œ, я с ΠΏΠΎΠΌΠΎΡ‰ΡŒΡŽ ΠΏΡ€ΠΎΠ²ΠΎΠ΄ΠΊΠΎΠ² соСдинял Ρ€Π°Π·Π»ΠΈΡ‡Π½Ρ‹Π΅ радиоэлСмСнты ΠΈ Ρƒ мСня ΠΏΠΎΠ»ΡƒΡ‡ΠΈΠ»Π°ΡΡŒ схСма, которая усиливаСт Π·Π²ΡƒΠΊΠΎΠ²Ρ‹Π΅ частоты

Для Ρ‚ΠΎΠ³ΠΎ, Ρ‡Ρ‚ΠΎΠ±Ρ‹ всС Π±Ρ‹Π»ΠΎ красиво, эстСтично ΠΈ Π·Π°Π½ΠΈΠΌΠ°Π»ΠΎ ΠΌΠ°Π»ΠΎ пространства, прямо Π½Π° ΠΏΠ»Π°Ρ‚Π°Ρ… ΡΠΎΠ·Π΄Π°ΡŽΡ‚ β€œΠΏΡ€ΠΎΠ²ΠΎΠ΄ΠΊΠΈβ€, ΠΊΠΎΡ‚ΠΎΡ€Ρ‹Π΅ ΡƒΠΆΠ΅ Π½Π°Π·Ρ‹Π²Π°ΡŽΡ‚ΡΡ ΠΏΠ΅Ρ‡Π°Ρ‚Π½Ρ‹ΠΌΠΈ Π΄ΠΎΡ€ΠΎΠΆΠΊΠ°ΠΌΠΈ.

Как Ρ‡ΠΈΡ‚Π°Ρ‚ΡŒ схСмы. НапряТСниС ΠΈ сила Ρ‚ΠΎΠΊΠ°

Π’ Π΄ΠΎΠΌΠ°ΡˆΠ½ΠΈΡ… условиях всС это дСлаСтся с ΠΏΠΎΠΌΠΎΡ‰ΡŒΡŽ Ρ‚Π΅Ρ…Π½ΠΎΠ»ΠΎΠ³ΠΈΠΈ Π›Π£Π’ (Π›Π°Π·Π΅Ρ€Π½ΠΎ-Π£Ρ‚ΡŽΠΆΠ½Π°Ρ-ВСхнология).Β 

На Π΄Ρ€ΡƒΠ³ΠΎΠΉ сторонС ΠΏΠ΅Ρ‡Π°Ρ‚Π½ΠΎΠΉ ΠΏΠ»Π°Ρ‚Ρ‹ ΡƒΠΆΠ΅ Ρ€Π°ΡΠΏΠΎΠ»Π°Π³Π°ΡŽΡ‚ΡΡ радиоэлСмСнты

Как Ρ‡ΠΈΡ‚Π°Ρ‚ΡŒ схСмы. НапряТСниС ΠΈ сила Ρ‚ΠΎΠΊΠ°

Π’Π°ΠΊ ΠΊΠ°ΠΊ Ρ€Π°Π΄ΠΈΠΎΠ»ΡŽΠ±ΠΈΡ‚Π΅Π»ΠΈ ΡΡ‚Π°Ρ€Π°ΡŽΡ‚ΡΡ Π΄Π΅Π»Π°Ρ‚ΡŒ свои устройства ΠΊΠ°ΠΊ ΠΌΠΎΠΆΠ½ΠΎ мСньшС ΠΏΠΎ Π³Π°Π±Π°Ρ€ΠΈΡ‚Π°ΠΌ, Ρ‚ΠΎ ΠΈ ΠΏΠ»ΠΎΡ‚Π½ΠΎΡΡ‚ΡŒ ΠΌΠΎΠ½Ρ‚Π°ΠΆΠ° возрастаСт. ΠŸΠΎΡΡ‚ΠΎΠΌΡƒ Π² Π½Π΅ΠΊΠΎΡ‚ΠΎΡ€Ρ‹Ρ… случаях радиоэлСмСнты ΠΈ ΠΏΠ΅Ρ‡Π°Ρ‚Π½Ρ‹Π΅ Π΄ΠΎΡ€ΠΎΠΆΠΊΠΈ Ρ€Π°ΡΠΏΠΎΠ»Π°Π³Π°ΡŽΡ‚ ΠΏΠΎ ΠΎΠ±Π΅ стороны ΠΏΠ»Π°Ρ‚Ρ‹.

ΠŸΡ€ΠΎΠΌΡ‹ΡˆΠ»Π΅Π½Π½Ρ‹Π΅ ΠΏΠ΅Ρ‡Π°Ρ‚Π½Ρ‹Π΅ ΠΏΠ»Π°Ρ‚Ρ‹ ΡƒΠΆΠ΅ Π΄Π΅Π»Π°ΡŽΡ‚ многослойными. Они состоят ΠΈΠ· слоСв,Β  ΠΊΠ°ΠΊ Ρ‚ΠΎΡ€Ρ‚ ΠΈΠ· ΠΊΠΎΡ€ΠΆΠ΅ΠΉ:

Как Ρ‡ΠΈΡ‚Π°Ρ‚ΡŒ схСмы. НапряТСниС ΠΈ сила Ρ‚ΠΎΠΊΠ°

ΠŸΡ€ΡΠΌΠΎ Π²Π½ΡƒΡ‚Ρ€ΠΈ Π½ΠΈΡ…Β  Π΅ΡΡ‚ΡŒ Π΄ΠΎΡ€ΠΎΠΆΠΊΠΈ, ΠΊΠΎΡ‚ΠΎΡ€Ρ‹Π΅ ΡΠΎΠ΅Π΄ΠΈΠ½ΡΡŽΡ‚ΡΡ мСТслойно. ΠžΡ‡Π΅Π½ΡŒ сильно экономится ΠΏΠ»ΠΎΡ‰Π°Π΄ΡŒ Π½Π° повСрхностях ΠΏΠ΅Ρ‡Π°Ρ‚Π½ΠΎΠΉ ΠΏΠ»Π°Ρ‚Ρ‹. Π‘ΡƒΠΌ Β SMDΒ  Ρ‚Π΅Ρ…Π½ΠΎΠ»ΠΎΠ³ΠΈΠΉ Π²Ρ‹Π·Π²Π°Π» Π² свою ΠΎΡ‡Π΅Ρ€Π΅Π΄ΡŒ Π½ΡƒΠΆΠ΄Ρƒ Π² многослойных ΠΏΠ΅Ρ‡Π°Ρ‚Π½Ρ‹Ρ… ΠΏΠ»Π°Ρ‚Π°Ρ….

ЭлСктричСский Ρ‚ΠΎΠΊ

Π”ΡƒΠΌΠ°ΡŽ, Π²Ρ‹Β  Π½Π΅ Ρ€Π°Π· ΡΠ»Ρ‹ΡˆΠ°Π»ΠΈ Ρ‚Π°ΠΊΠΎΠ΅ Π²Ρ‹Ρ€Π°ΠΆΠ΅Π½ΠΈΠ΅: β€œΠΏΠΎ этому ΠΏΡ€ΠΎΠ²ΠΎΠ΄Ρƒ Ρ‚Π΅Ρ‡Π΅Ρ‚ ток”. Π­Π»Π΅ΠΊΡ‚Ρ€ΠΎΠ½ΠΈΠΊΡƒ ΠΏΡ€ΠΎΡ‰Π΅ ΠΎΠ±ΡŠΡΡΠ½ΡΡ‚ΡŒ ΠΊΠ°ΠΊ Ρ€Π°Π· с Ρ‚ΠΎΡ‡ΠΊΠΈ зрСния Π³ΠΈΠ΄Ρ€Π°Π²Π»ΠΈΠΊΠΈ. Π Π°Π· Ρ‚ΠΎΠΊ Ρ‚Π΅Ρ‡Π΅Ρ‚, Π·Π½Π°Ρ‡ΠΈΡ‚, Π² нашСм случаС, ΠΏΡ€ΠΎΠ²ΠΎΠ΄ΠΎΠΊ – это шланг ΠΈΠ»ΠΈ Ρ‚Ρ€ΡƒΠ±Π° для элСктричСского Ρ‚ΠΎΠΊΠ°. ΠŸΠΎΠ»ΡƒΡ‡Π°Π΅Ρ‚ΡΡ, Ρ‡Ρ‚ΠΎ Ρ‚Π°ΠΊ. А Ρ‡Ρ‚ΠΎ Ρ‚Π°ΠΊΠΎΠ΅ элСктричСский Ρ‚ΠΎΠΊ?

ЭлСктричСский Ρ‚ΠΎΠΊ – это упорядочСнноС Π΄Π²ΠΈΠΆΠ΅Π½ΠΈΠ΅ заряТСнных частиц, Ρ‡Π°Ρ‰Π΅ всСго элСктронов, Π² ΠΎΠ΄Π½ΠΎΠΌ Π½Π°ΠΏΡ€Π°Π²Π»Π΅Π½ΠΈΠΈ. По Π°Π½Π°Π»ΠΎΠ³ΠΈΠΈ с Π³ΠΈΠ΄Ρ€Π°Π²Π»ΠΈΠΊΠΎΠΉ, элСктроны – это ΠΌΠΎΠ»Π΅ΠΊΡƒΠ»Ρ‹ Π²ΠΎΠ΄Ρ‹. ЭлСктричСский Ρ‚ΠΎΠΊ – ΠΏΠΎΡ‚ΠΎΠΊ Π²ΠΎΠ΄Ρ‹. Π”ΡƒΠΌΠ°ΡŽ, этого ΠΏΠΎΠΊΠ° Π±ΡƒΠ΄Π΅Ρ‚ достаточно. Одними словами сыт Π½Π΅ Π±ΡƒΠ΄Π΅ΡˆΡŒ, поэтому Π΄Π°Π²Π°ΠΉΡ‚Π΅ нарисуСм рисунок, Ρ‡Ρ‚ΠΎΠ±Ρ‹ ΠΏΠΎΡ€Π°Π΄ΠΎΠ²Π°Ρ‚ΡŒ Π³Π»Π°Π·Π°:

Π’ Π΄Π°Π½Π½Ρ‹ΠΉ ΠΌΠΎΠΌΠ΅Π½Ρ‚ шланг валяСтся Π³Π΄Π΅-Π½ΠΈΠ±ΡƒΠ΄ΡŒ Π² ΠΎΠ³ΠΎΡ€ΠΎΠ΄Π΅ ΠΈ Π² Π½Π΅ΠΌ ΠΎΡΡ‚Π°Π»Π°ΡΡŒ Π²ΠΎΠ΄Π°. Π¨Π»Π°Π½Π³ Π½ΠΈΠΊΡƒΠ΄Π° Π½Π΅ ΠΏΠΎΠ΄ΠΊΠ»ΡŽΡ‡Π΅Π½, Ρ‚ΠΎ Π΅ΡΡ‚ΡŒ ΠΌΠΎΠ»Π΅ΠΊΡƒΠ»Ρ‹ Π²ΠΎΠ΄Ρ‹ Π² шлангС находятся Π² Π½Π΅ΠΏΠΎΠ΄Π²ΠΈΠΆΠ½ΠΎΠΌ состоянии.

По Π°Π½Π°Π»ΠΎΠ³ΠΈΠΈ с элСктроникой, ΠΌΠ΅Π΄Π½Ρ‹ΠΉ ΠΏΡ€ΠΎΠ²ΠΎΠ΄ΠΎΠΊ Π»Π΅ΠΆΠΈΡ‚ Π½Π° столС ΠΈ Π½ΠΈΠΊΡƒΠ΄Π° Π½Π΅ ΠΏΠΎΠ΄ΠΊΠ»ΡŽΡ‡Π΅Π½.

Но Π²ΠΎΡ‚ настал Π²Π΅Ρ‡Π΅Ρ€. Надо ΠΏΠΎΠ»ΠΈΡ‚ΡŒ ΠΏΠΎΠΌΠΈΠ΄ΠΎΡ€Ρ‹ ΠΈ ΠΎΠ³ΡƒΡ€Ρ†Ρ‹, ΠΈΠ½Π°Ρ‡Π΅ ΠΊ Π·ΠΈΠΌΠ΅ ΠΎΡΡ‚Π°Π½Π΅Ρ‚Π΅ΡΡŒ Π±Π΅Π· закуски. Как Ρ‚ΠΎΠ»ΡŒΠΊΠΎ ΠΌΡ‹ ΠΎΡ‚ΠΊΡ€Ρ‹Π²Π°Π΅ΠΌ ΠΊΡ€Π°Π½, Π²ΠΎΠ΄Π° Π² шлангС Π½Π°Ρ‡ΠΈΠ½Π°Π΅Ρ‚ Π΄Π²ΠΈΠΆΡƒΡ…Ρƒ:

Π’Π΅ΠΏΠ΅Ρ€ΡŒ вопрос Π½Π° засыпку: ΠΏΠΎΡ‡Π΅ΠΌΡƒ ΠΊΠΎΠ³Π΄Π° ΠΌΡ‹ ΠΎΡ‚ΠΊΡ€Ρ‹Π»ΠΈ ΠΊΡ€Π°Π½ΠΈΠΊ, Π²ΠΎΠ΄Π° ΠΏΠΎΠ±Π΅ΠΆΠ°Π»Π° ΠΏΠΎ ΡˆΠ»Π°Π½Π³Ρƒ?Β  Боздалось давлСниС… ΠΌΠΎΠ»Π΅ΠΊΡƒΠ»Ρ‹ Ρ‡Ρ‚ΠΎ Π»Π΅Π²Π΅Π΅ стали Π΄Π°Π²ΠΈΡ‚ΡŒ Π½Π° ΠΌΠΎΠ»Π΅ΠΊΡƒΠ»Ρ‹ Ρ‡Ρ‚ΠΎ ΠΏΡ€Π°Π²Π΅Π΅ ΠΈ Π΄Π²ΠΈΠΆΡƒΡ…Π° Π½Π°Ρ‡Π°Π»Π°ΡΡŒ. Но ΠΊΡ‚ΠΎ Ρ‚ΠΎΠ»ΠΊΠ°Π» Ρ‚Π΅ ΠΌΠΎΠ»Π΅ΠΊΡƒΠ»Ρ‹, ΠΊΠΎΡ‚ΠΎΡ€Ρ‹Π΅ Ρ‚ΠΎΠ»ΠΊΠ°Π»ΠΈ ΠΌΠΎΠ»Π΅ΠΊΡƒΠ»Ρ‹? Π­Ρ‚ΠΎ Π»ΠΈΠ±ΠΎ насос, Π»ΠΈΠ±ΠΎ Π²ΠΎΠ΄Π° Π² водобашнС ΠΏΠΎΠ΄ воздСйствиСм Π³Ρ€Π°Π²ΠΈΡ‚Π°Ρ†ΠΈΠΎΠ½Π½ΠΎΠΉ силы Π—Π΅ΠΌΠ»ΠΈ.

Π’ элСктроникС элСктроны Ρ‚ΠΎΠ»ΠΊΠ°Π΅Ρ‚ Ρ‚Π°ΠΊ называСмая Π­Π”Π‘. Π’ любой элСктричСской схСмС Π΅ΡΡ‚ΡŒ Ρ‚ΠΎΡ‚ самый β€œΠ½Π°ΡΠΎΡβ€, ΠΊΠΎΡ‚ΠΎΡ€Ρ‹ΠΉ Ρ‚ΠΎΠ»ΠΊΠ°Π΅Ρ‚ элСктроны ΠΏΠΎ ΠΏΡ€ΠΎΠ²ΠΎΠ΄ΠΊΠ°ΠΌ ΠΈ радиоэлСмСнтам. Он ΠΌΠΎΠΆΠ΅Ρ‚ находится Π² самой схСмС, Π»ΠΈΠ±ΠΎ ΠΏΠΎΠ΄ΠΊΠ»ΡŽΡ‡Π°Ρ‚ΡŒΡΡ Π² схСму ΠΈΠ·Π²Π½Π΅. Как Ρ‚ΠΎΠ»ΡŒΠΊΠΎ элСктроны Π½Π°Ρ‡ΠΈΠ½Π°ΡŽΡ‚ Π΄Π²ΠΈΠΆΡƒΡ…Ρƒ Π² ΠΏΡ€ΠΎΠ²ΠΎΠ΄ΠΊΠ΅ Π² ΠΎΠ΄Π½ΠΎΠΌ Π½Π°ΠΏΡ€Π°Π²Π»Π΅Π½ΠΈΠΈ, Ρ‚ΠΎ ΠΌΠΎΠΆΠ½ΠΎ ΡƒΠΆΠ΅ ΡΠΊΠ°Π·Π°Ρ‚ΡŒ, Ρ‡Ρ‚ΠΎ Π² ΠΏΡ€ΠΎΠ²ΠΎΠ΄ΠΊΠ΅ стал Ρ‚Π΅Ρ‡ΡŒ

элСктричСский Ρ‚ΠΎΠΊ.

НапряТСниС

А Ρ‚Π΅ΠΏΠ΅Ρ€ΡŒ ΠΏΡ€Π΅Π΄ΡΡ‚Π°Π²ΡŒΡ‚Π΅ Ρ‚Π°ΠΊΡƒΡŽ ΡΠΈΡ‚ΡƒΠ°Ρ†ΠΈΡŽ. Π£ нас Π΅ΡΡ‚ΡŒ водонасос, Π½ΠΎ шланг ΠΌΡ‹ Π·Π°ΠΊΡƒΠΏΠΎΡ€ΠΈΠ»ΠΈ ΠΏΡ€ΠΎΠ±ΠΊΠΎΠΉ.

Π’ΠΎΠ΄Π° Π²Ρ€ΠΎΠ΄Π΅ Π±Ρ‹ Π³ΠΎΡ‚ΠΎΠ²Π° Π±Π΅ΠΆΠ°Ρ‚ΡŒ, Π½ΠΎ Π±Π΅ΠΆΠ°Ρ‚ΡŒ Ρ‚ΠΎ Π½Π΅ΠΊΡƒΠ΄Π°! Π’Π°ΠΌ ΠΏΡ€ΠΎΠ±ΠΊΠ° Π·Π°ΠΊΡƒΠΏΠΎΡ€ΠΈΠ²Π°Π΅Ρ‚ шланг. Но Π½Π° саму ΠΏΡ€ΠΎΠ±ΠΊΡƒ сСйчас оказываСтся Π΄Π°Π²Π»Π΅Π½ΠΈΠ΅, ΠΊΠΎΡ‚ΠΎΡ€ΠΎΠ΅ создаСт насосная станция. ΠžΡ‚ Ρ‡Π΅Π³ΠΎ зависит Π΄Π°Π²Π»Π΅Π½ΠΈΠ΅ Π½Π° ΠΏΡ€ΠΎΠ±ΠΊΡƒ? Π”ΡƒΠΌΠ°ΡŽ понятно, Ρ‡Ρ‚ΠΎ ΠΎΡ‚ мощности насоса. Если ΠΌΠΎΡ‰Π½ΠΎΡΡ‚ΡŒ насоса Π±ΡƒΠ΄Π΅Ρ‚ приличная, Ρ‚ΠΎ ΠΏΡ€ΠΎΠ±ΠΊΠ° Π²Ρ‹Π»Π΅Ρ‚ΠΈΡ‚ со ΡΠΊΠΎΡ€ΠΎΡΡ‚ΡŒΡŽ ΠΏΡƒΠ»ΠΈ, Π»ΠΈΠ±ΠΎ Π΄Π°Π²Π»Π΅Π½ΠΈΠ΅ ΠΏΠΎΡ€Π²Π΅Ρ‚ шланг, Ссли ΠΏΡ€ΠΎΠ±ΠΊΠ° Ρ‚ΡƒΠ³ΠΎ сидит Π² шлангС.

ВсС Ρ‚ΠΎ ΠΆΠ΅ самоС ΠΌΠΎΠΆΠ½ΠΎ ΡΠΊΠ°Π·Π°Ρ‚ΡŒ ΠΈ ΠΏΡ€ΠΎ водобашню. Π”Π°Π²Π»Π΅Π½ΠΈΠ΅ Π½Π° Π΄Π½Π΅ башни зависит ΠΎΡ‚ Ρ‚ΠΎΠ³ΠΎ, сколько Π²ΠΎΠ΄Ρ‹ Π½Π°Π»ΠΈΡ‚ΠΎ Π² башню. Если башня ΠΏΠΎΠ΄ завязку, Ρ‚ΠΎ ΠΈ Π΄Π°Π²Π»Π΅Π½ΠΈΠ΅ Π½Π° Π΄Π½Π΅ башни Π±ΡƒΠ΄Π΅Ρ‚ большоС, ΠΈ Π½Π°ΠΎΠ±ΠΎΡ€ΠΎΡ‚.

Как Ρ‡ΠΈΡ‚Π°Ρ‚ΡŒ схСмы. НапряТСниС ΠΈ сила Ρ‚ΠΎΠΊΠ°

А Ρ‚Π΅ΠΏΠ΅Ρ€ΡŒ ΠΏΡ€ΠΈΠΊΠΈΠ½ΡŒΡ‚Π΅ ΠΊΠ°ΠΊΠΎΠ΅ Π΄Π°Π²Π»Π΅Π½ΠΈΠ΅ Π½Π° Π΄Π½Π΅ ΠΎΠΊΠ΅Π°Π½Π°, особСнно Π² ΠœΠ°Ρ€ΠΈΠ°Π½ΡΠΊΠΎΠΉ Π²ΠΏΠ°Π΄ΠΈΠ½Π΅ πŸ˜‰

Π§Ρ‚ΠΎ ΠΌΠΎΠΆΠ½ΠΎ ΡΠΊΠ°Π·Π°Ρ‚ΡŒ ΠΏΡ€ΠΎ Π΄Π°Π²Π»Π΅Π½ΠΈΠ΅ Π² этих Π΄Π²ΡƒΡ… случаях? Оно Π²Ρ€ΠΎΠ΄Π΅ ΠΊΠ°ΠΊ Π΅ΡΡ‚ΡŒ, Π½ΠΎ ΠΌΠΎΠ»Π΅ΠΊΡƒΠ»Ρ‹ Π²ΠΎΠ΄Ρ‹ стоят Π½Π° мСстС.

Π’Π°ΠΊ Π²ΠΎΡ‚, ΠΏΠΎ Π°Π½Π°Π»ΠΎΠ³ΠΈΠΈ с элСктроникой, это Π΄Π°Π²Π»Π΅Π½ΠΈΠ΅ называСтся напряТСниСм. НапримСр, Π²Ρ‹, Π½Π°Π²Π΅Ρ€Π½ΠΎΠ΅, Π½Π΅ Ρ€Π°Π· ΡΠ»Ρ‹ΡˆΠ°Π»ΠΈ Ρ‚Π°ΠΊΠΎΠ΅ Π²Ρ‹Ρ€Π°ΠΆΠ΅Π½ΠΈΠ΅, Ρ‚ΠΈΠΏΠ° β€œΠ±Π»ΠΎΠΊ питания ΠΌΠΎΠΆΠ΅Ρ‚ Π²Ρ‹Π΄Π°Ρ‚ΡŒ напряТСниС ΠΎΡ‚ 0 ΠΈ Π΄ΠΎ 30 Π’ΠΎΠ»ΡŒΡ‚β€. Или говоря дСтским языком, ΡΠΎΠ·Π΄Π°Ρ‚ΡŒ β€œΡΠ»Π΅ΠΊΡ‚Ρ€ΠΈΡ‡Π΅ΡΠΊΠΎΠ΅ давлСниС” Π½Π° своих ΠΊΠ»Π΅ΠΌΠΌΠ°Ρ… (ΠΎΡ‚ΠΌΠ΅Ρ‚ΠΈΠ» Π½Π° Ρ„ΠΎΡ‚ΠΎ) ΠΎΡ‚ 0 ΠΈ Π΄ΠΎ 30 Π’ΠΎΠ»ΡŒΡ‚. НулСвой ΡƒΡ€ΠΎΠ²Π΅Π½ΡŒ, ΠΎΡ‚ΠΊΡƒΠ΄Π° ΠΈΠ΄Π΅Ρ‚ отсчСт элСктричСского давлСния, обозначаСтся минусом.

Как Ρ‡ΠΈΡ‚Π°Ρ‚ΡŒ схСмы. НапряТСниС ΠΈ сила Ρ‚ΠΎΠΊΠ°

ЭлСктричСскоС Π΄Π°Π²Π»Π΅Π½ΠΈΠ΅Β  – это Π΅Ρ‰Π΅ Π½Π΅ Π·Π½Π°Ρ‡ΠΈΡ‚, Ρ‡Ρ‚ΠΎ Π΅ΡΡ‚ΡŒ элСктричСский Ρ‚ΠΎΠΊ. Для Ρ‚ΠΎΠ³ΠΎ, Ρ‡Ρ‚ΠΎΠ±Ρ‹ появился элСктричСский Ρ‚ΠΎΠΊ Π΄ΠΎΠ»ΠΆΠ½Π° Π±Ρ‹Ρ‚ΡŒ Π΄Π²ΠΈΠΆΡƒΡ…Π° элСктронов Π² ΠΎΠ΄Π½ΠΎΠΌ Π½Π°ΠΏΡ€Π°Π²Π»Π΅Π½ΠΈΠΈ, Π° ΠΎΠ½ΠΈ Π² Π΄Π°Π½Π½Ρ‹ΠΉ ΠΌΠΎΠΌΠ΅Π½Ρ‚ Ρ‚ΡƒΠΏΠΎ стоят Π½Π° мСстС.Β  А Ρ€Π°Π· Π΄Π²ΠΈΠΆΡƒΡ…ΠΈ Π½Π΅Ρ‚Ρƒ, Ρ‚ΠΎ ΠΈ Π½Π΅Ρ‚ элСктричСского Ρ‚ΠΎΠΊΠ°. Но Ρ‚ΠΎ, Ρ‡Ρ‚ΠΎ ΡƒΠΆΠ΅ Π΅ΡΡ‚ΡŒ Π΄Π°Π²Π»Π΅Π½ΠΈΠ΅ – это прСдпосылка ΠΊ Π·Π°Ρ€ΠΎΠΆΠ΄Π΅Π½ΠΈΡŽ элСктричСского Ρ‚ΠΎΠΊΠ°.

Π’Ρ‹ прямо сСйчас ΠΌΠΎΠΆΠ΅Ρ‚Π΅ ΡΠΎΠ·Π΄Π°Ρ‚ΡŒ Π΄Π°Π²Π»Π΅Π½ΠΈΠ΅ Π²ΠΎΠ·Π΄ΡƒΡ…Π° Π² своСм ΠΎΡ€Π³Π°Π½ΠΈΠ·ΠΌΠ΅. Для этого достаточно Π½Π°Π±Ρ€Π°Ρ‚ΡŒ Π²ΠΎΠ·Π΄ΡƒΡ…Π° Π² Π»Π΅Π³ΠΊΠΈΠ΅ ΠΈ Π·Π°ΠΊΡ€Ρ‹Ρ‚ΡŒ Ρ€ΠΎΡ‚. ΠŸΠΎΡ‚ΠΎΠΌ Π²Ρ‹ΠΏΡƒΡΡ‚ΠΈΡ‚ΡŒ Π²ΠΎΠ·Π΄ΡƒΡ… ΠΈ Π½Π°Π΄ΡƒΡ‚ΡŒ Ρ‰Π΅ΠΊΠΈ, Π½Π΅ открывая Ρ€ΠΎΡ‚. Π’ это врСмя Ρƒ вас Π½Π° Ρ‰Π΅ΠΊΠΈ ΠΌΠΎΠ»Π΅ΠΊΡƒΠ»Ρ‹ Π²ΠΎΠ·Π΄ΡƒΡ…Π° Π±ΡƒΠ΄ΡƒΡ‚ ΠΎΠΊΠ°Π·Ρ‹Π²Π°Ρ‚ΡŒ Π΄Π°Π²Π»Π΅Π½ΠΈΠ΅. Π§Π΅ΠΌ большС Π²Ρ‹ Π²Ρ‹Π΄Ρ‹Ρ…Π°Π΅Ρ‚Π΅ Π²ΠΎΠ·Π΄ΡƒΡ…Π°, Ρ‚Π΅ΠΌ напряТСннСС ΡΡ‚Π°ΡŽΡ‚ ваши Ρ‰Π΅ΠΊΠΈ ΠΎΡ‚ давлСния. Π”Π²ΠΈΠΆΡƒΡ…Π° ΠΈΠ΄Π΅Ρ‚ ΠΈΠ· области высокого давлСния Π² ΠΎΠ±Π»Π°ΡΡ‚ΡŒ Π½ΠΈΠ·ΠΊΠΎΠ³ΠΎ давлСния. Π’ Π²Π°ΡˆΠΈΡ… Π»Π΅Π³ΠΊΠΈΡ… Π²Ρ‹ создали большоС Π΄Π°Π²Π»Π΅Π½ΠΈΠ΅, Π° Π΄Π°Π²Π»Π΅Π½ΠΈΠ΅ снаруТи оказалось мСньшС. ΠŸΠΎΡΡ‚ΠΎΠΌΡƒ-Ρ‚ΠΎ Ρ‰Ρ‘Ρ‡ΠΊΠΈ ΠΈ Π½Π°Π΄ΡƒΠ»ΠΈΡΡŒ.

Π‘ Ρ‚ΠΎΡ‡ΠΊΠΈ зрСния элСктроники, Π½Π° ΠΎΠ΄Π½ΠΎΠΌ Ρ‰ΡƒΠΏΠ΅ Π±Π»ΠΎΠΊΠ° питания высокоС Π΄Π°Π²Π»Π΅Π½ΠΈΠ΅, Π° Π½Π° Π΄Ρ€ΡƒΠ³ΠΎΠΌ Π½ΠΈΠ·ΠΊΠΎΠ΅. ΠŸΠΎΡΡ‚ΠΎΠΌΡƒ, ΠΏΠΎΠ»ΠΎΠΆΠΈΡ‚Π΅Π»ΡŒΠ½Ρ‹ΠΉΒ  Ρ‰ΡƒΠΏ Π±Π»ΠΎΠΊΠ° питания Π΄Π° ΠΈ Π²ΠΎΠΎΠ±Ρ‰Π΅ всСх ΠΏΡ€ΠΈΠ±ΠΎΡ€ΠΎΠ² ΡΡ‚Π°Ρ€Π°ΡŽΡ‚ΡΡ ΡΠ΄Π΅Π»Π°Ρ‚ΡŒ красным, ΠΌΠΎΠ» Ρ‚ΠΈΠΏΠ° Π±Π΅Ρ€Π΅Π³ΠΈΡ‚Π΅ΡΡŒ, здСсь высокоС Π΄Π°Π²Π»Π΅Π½ΠΈΠ΅! А ΠΎΡ‚Ρ€ΠΈΡ†Π°Ρ‚Π΅Π»ΡŒΠ½Ρ‹ΠΉ Ρ‰ΡƒΠΏΒ  – Ρ‡Π΅Ρ€Π½Ρ‹ΠΌ ΠΈΠ»ΠΈ синим. Π’ΡƒΡ‚ Ρ‚ΠΈΠΏΠ° Π΄Π°Π²Π»Π΅Π½ΠΈΠ΅ минимальноС (Π½ΡƒΠ»Π΅Π²ΠΎΠ΅).

Π’ элСктроникС, Ρ‡Ρ‚ΠΎΠ±Ρ‹ ΡƒΠΊΠ°Π·Π°Ρ‚ΡŒ, Π½Π° ΠΊΠ°ΠΊΠΎΠΌ Π²Ρ‹Π²ΠΎΠ΄Π΅ большС ” элСктричСскоС давлСниС”, Π° Π½Π° ΠΊΠ°ΠΊΠΎΠΌ мСньшС ΠΏΡ€ΠΎΡΡ‚Π°Π²Π»ΡΡŽΡ‚ Π΄Π²Π° Π·Π½Π°ΠΊΠ°: плюс ΠΈ минус, соотвСтствСнно ΠΏΠΎΠ»ΠΎΠΆΠΈΡ‚Π΅Π»ΡŒΠ½Ρ‹ΠΉ ΠΈ ΠΎΡ‚Ρ€ΠΈΡ†Π°Ρ‚Π΅Π»ΡŒΠ½Ρ‹ΠΉ. На плюсС ΠΈΠ·Π±Ρ‹Ρ‚ΠΎΡ‡Π½ΠΎΠ΅ β€œΠ΄Π°Π²Π»Π΅Π½ΠΈΠ΅β€, Π° Π½Π° минусС – нСдостаточноС.

Как Ρ‡ΠΈΡ‚Π°Ρ‚ΡŒ схСмы. НапряТСниС ΠΈ сила Ρ‚ΠΎΠΊΠ°ΠšΠ°ΠΊ Ρ‡ΠΈΡ‚Π°Ρ‚ΡŒ схСмы. НапряТСниС ΠΈ сила Ρ‚ΠΎΠΊΠ°

ΠŸΠΎΡΡ‚ΠΎΠΌΡƒ, Ссли Π·Π°ΠΌΠΊΠ½ΡƒΡ‚ΡŒ эти Π΄Π²Π° Π²Ρ‹Π²ΠΎΠ΄Π° ΠΌΠ΅ΠΆΠ΄Ρƒ собой, элСктричСский Ρ‚ΠΎΠΊ устрСмится ΠΎΡ‚ плюса ΠΊ минусу, Π½ΠΎΒ  Π½Π°ΠΏΡ€ΡΠΌΡƒΡŽ этого Π΄Π΅Π»Π°Ρ‚ΡŒ ΠΊΡ€Π°ΠΉΠ½Π΅ Π½Π΅ рСкомСндуСтся, Ρ‚Π°ΠΊ ΠΊΠ°ΠΊ это ΡƒΠΆΠ΅ Π±ΡƒΠ΄Π΅Ρ‚ Π½Π°Π·Ρ‹Π²Π°Ρ‚ΡŒΡΡ ΠΊΠΎΡ€ΠΎΡ‚ΠΊΠΈΠΌ Π·Π°ΠΌΡ‹ΠΊΠ°Π½ΠΈΠ΅ΠΌ.

Π˜Ρ‚Π°ΠΊ, ΠΎΠ΄Π½Π° ΡΠΎΡΡ‚Π°Π²Π»ΡΡŽΡ‰Π°Ρ для зароТдСния элСктричСского Ρ‚ΠΎΠΊΠ° Ρƒ нас ΡƒΠΆΠ΅ Π΅ΡΡ‚ΡŒ – это напряТСниС.

ВСрнСмся снова ΠΊ Π³ΠΈΠ΄Ρ€Π°Π²Π»ΠΈΠΊΠ΅.

Π”Π°Π²Π»Π΅Π½ΠΈΠ΅ ΠΌΡ‹ создали, Π½ΠΎ элСктричСского Ρ‚ΠΎΠΊΠ° Π΄ΠΎ сих ΠΏΠΎΡ€ Π½Π΅Ρ‚Ρƒ. Π§Ρ‚ΠΎ Π½Π°Π΄ΠΎ ΡΠ΄Π΅Π»Π°Ρ‚ΡŒ? ΠŸΡ€Π°Π²ΠΈΠ»ΡŒΠ½ΠΎ, ΡƒΠ±Ρ€Π°Ρ‚ΡŒ ΠΏΡ€ΠΎΠ±ΠΊΡƒ ΠΈΠ· шланга ΠΈ Π΄Π°Ρ‚ΡŒ Π²ΠΎΠ΄ΠΈΡ‡ΠΊΠ΅ спокойно Π²Ρ‹Ρ‚Π΅ΠΊΠ°Ρ‚ΡŒ. Пошла Π΄Π²ΠΈΠΆΡƒΡ…Π°, Π·Π½Π°Ρ‡ΠΈΡ‚, пошСл элСктричСский Ρ‚ΠΎΠΊ!

ΠžΡ‚ ΠΊΠ°ΠΊΠΎΠ³ΠΎ слова образуСтся слово β€œΡ‚ΠΎΠΊβ€. Π― Π΄ΡƒΠΌΠ°ΡŽ, ΠΎΡ‚ слова поВОК. ΠŸΠΎΡ‚ΠΎΠΊ Π²ΠΎΠ΄Ρ‹, ΠΏΠΎΡ‚ΠΎΠΊ энСргии, ΠΏΠΎΡ‚ΠΎΠΊ свСта ΠΈ Ρ‚Π΄, Π° ΠΏΠΎΡ‚ΠΎΠΊ элСктронов Π² ΠΏΡ€ΠΎΠ²ΠΎΠ΄ΠΊΠ΅ называСтся просто β€œΡΠ»Π΅ΠΊΡ‚Ρ€ΠΈΡ‡Π΅ΡΠΊΠΈΠΌ током”. Π—Π½Π°Ρ‡ΠΈΡ‚, заставляя Ρ‚Π΅Ρ‡ΡŒ элСктроны, ΠΌΡ‹ Ρ‚Π΅ΠΌ самым создаСм элСктричСский Ρ‚ΠΎΠΊ πŸ˜‰

Π’Π΅ΠΏΠ΅Ρ€ΡŒ снова Π½Π°Π΄ΡƒΠΉΡ‚Π΅ свои ΠΏΡƒΡ…Π»Π΅Π½ΡŒΠΊΠΈΠ΅ Ρ‰Π΅Ρ‡ΠΊΠΈ ΠΈ ΠΏΡ‹Ρ‚Π°ΠΉΡ‚Π΅ΡΡŒ ΡΠΎΠ·Π΄Π°Ρ‚ΡŒ Π²Π½ΡƒΡ‚Ρ€ΠΈ полости Ρ€Ρ‚Π° ΠΎΡ‡Π΅Π½ΡŒ высокоС Π΄Π°Π²Π»Π΅Π½ΠΈΠ΅. Π§Ρ‚ΠΎ Ρƒ нас ΠΏΡ€ΠΎΠΈΠ·ΠΎΠΉΠ΄Π΅Ρ‚? Π’Π°ΡˆΠΈ Π³ΡƒΠ±ΠΊΠΈ Π½Π΅ Π²Ρ‹Π΄Π΅Ρ€ΠΆΠ°Ρ‚ ΠΈ ΠΏΠΎΡ‚ΠΎΠΊ Π²ΠΎΠ·Π΄ΡƒΡ…Π° устрСмится ΠΈΠ·ΠΎ Ρ€Ρ‚Π° Π² ΠΎΠΊΡ€ΡƒΠΆΠ°ΡŽΡ‰Π΅Π΅ пространство. Π’ΠΎ Π΅ΡΡ‚ΡŒ Π²Ρ‹ создали Π² полости Ρ€Ρ‚Π° высокоС Π΄Π°Π²Π»Π΅Π½ΠΈΠ΅, ΠΊΠΎΡ‚ΠΎΡ€ΠΎΠ΅ ΡƒΡΡ‚Ρ€Π΅ΠΌΠΈΠ»ΠΎΡΡŒ Π² ΠΎΠ±Π»Π°ΡΡ‚ΡŒ Π½ΠΈΠ·ΠΊΠΎΠ³ΠΎ давлСния, Ρ‚ΠΎ Π΅ΡΡ‚ΡŒ Π½Π°Ρ€ΡƒΠΆΡƒ. ΠŸΠΎΡ‡Ρ‚ΠΈ схоТим ΠΎΠ±Ρ€Π°Π·ΠΎΠΌ Π²Ρ‹ создаСтС β€œΠ²Π΅Ρ‚Π΅Ρ€β€ ΠΈΠ· ΠΏΡƒΠΊΠ°Π½Π°, напрягая свой ΠΆΠΈΠ²ΠΎΡ‚ΠΈΠΊ :-).

Π›Π°Π΄Π½ΠΎ, Π΄Π°Π²Π°ΠΉΡ‚Π΅ ΠΎΠ±ΠΎΠ±Ρ‰ΠΈΠΌ, всС Ρ‡Ρ‚ΠΎ ΠΌΡ‹ Ρ‚ΡƒΡ‚ пописали. Π­Π”Π‘ создаСт Π΄Π²ΠΈΠΆΡƒΡ…Ρƒ элСктронов ΠΏΠΎ ΠΏΡ€ΠΎΠ²ΠΎΠ΄ΠΊΡƒ. Для Ρ‚ΠΎΠ³ΠΎ, Ρ‡Ρ‚ΠΎΠ±Ρ‹ Π΄Π²ΠΈΠΆΡƒΡ…Π° Π±Ρ‹Π»Π°, элСктроны Π΄ΠΎΠ»ΠΆΠ½Ρ‹ ΠΊΡƒΠ΄Π°-Ρ‚ΠΎ Π½Π°ΠΏΡ€Π°Π²Π»ΡΡ‚ΡŒΡΡ, ΠΆΠ΅Π»Π°Ρ‚Π΅Π»ΡŒΠ½ΠΎ ΠΎΠ±Ρ€Π°Ρ‚Π½ΠΎ ΠΊ Π­Π”Π‘ источнику. Π’ ΠΈΠ΄Π΅Π°Π»Π΅, Π΄ΠΎΠ»ΠΆΠ½ΠΎ Π±Ρ‹Ρ‚ΡŒ ΠΊΠ°ΠΊ-Ρ‚ΠΎ Ρ‚Π°ΠΊ:

Как Π²Ρ‹ Π²ΠΈΠ΄ΠΈΡ‚Π΅, Ρ‚Ρ€ΡƒΠ±Π° Ρƒ нас Π²Ρ‹Ρ…ΠΎΠ΄ΠΈΡ‚ ΠΈΠ· насосной станции ΠΈ Π²Ρ…ΠΎΠ΄ΠΈΡ‚ Π² Π½Π°ΡΠΎΡΠ½ΡƒΡŽ ΡΡ‚Π°Π½Ρ†ΠΈΡŽ. Π’ΠΎ Π΅ΡΡ‚ΡŒ ΠΊΠΎΠ½Ρ‚ΡƒΡ€ Ρ‚Ρ€ΡƒΠ±Ρ‹ получаСтся Π·Π°ΠΌΠΊΠ½ΡƒΡ‚Ρ‹ΠΌ. Пока Ρ€Π°Π±ΠΎΡ‚Π°Π΅Ρ‚ насосная станция, Ρƒ нас Π΅ΡΡ‚ΡŒ Π΄Π²ΠΈΠΆΡƒΡ…Π° Π²ΠΎΠ΄Ρ‹. Как Ρ‚ΠΎΠ»ΡŒΠΊΠΎ насосная станция сдохнСт, Π΄Π²ΠΈΠΆΡƒΡ…Π° Π²ΠΎΠ΄Ρ‹ прСкратится. Π’Π°ΠΊΠΆΠ΅ Π½Π΅ΠΌΠ°Π»ΠΎΠ²Π°ΠΆΠ½ΠΎ Ρ‡Ρ‚ΠΎΠ±Ρ‹ Ρ‚Ρ€ΡƒΠ±Π° Π½Π΅ Π±Ρ‹Π»Π° тонкая Π² Π΄ΠΈΠ°ΠΌΠ΅Ρ‚Ρ€Π΅, ΠΈΠ½Π°Ρ‡Π΅ Π΅Π΅ ΠΏΠΎΡ€Π²Π΅Ρ‚, Ссли насосная станция Π±ΡƒΠ΄Π΅Ρ‚ большой мощности.

По Π°Π½Π°Π»ΠΎΠ³ΠΈΠΈ с элСктроникой ΠΏΠΎΠ»ΡƒΡ‡Π°Π΅ΠΌ всС Ρ‚ΠΎ ΠΆΠ΅ самоС. Π’ΠΎ-ΠΏΠ΅Ρ€Π²Ρ‹Ρ…, Π½ΡƒΠΆΠ½ΠΎ Ρ‡Ρ‚ΠΎΠ±Ρ‹ ΠΊΠΎΠ½Ρ‚ΡƒΡ€ Π±Ρ‹Π» Π·Π°ΠΌΠΊΠ½ΡƒΡ‚Ρ‹ΠΌ, Π²ΠΎ Π²Ρ‚ΠΎΡ€Ρ‹Ρ… – Ρ‡Ρ‚ΠΎΠ±Ρ‹ Π±Ρ‹Π» источник Π­Π”Π‘, ΠΈ Π²-Ρ‚Ρ€Π΅Ρ‚ΡŒΠΈΡ…, Ρ‡Ρ‚ΠΎΠ±Ρ‹ ΠΏΡ€ΠΎΠ²ΠΎΠ΄ Π²Ρ‹Π΄Π΅Ρ€ΠΆΠΈΠ²Π°Π» ΠΏΠΎΡ‚ΠΎΠΊ элСктронов.

Π‘ΠΈΠ»Π° Ρ‚ΠΎΠΊΠ°

Π’Π°ΠΊΠΆΠ΅ нас интСрСсуСт Π΅Ρ‰Π΅ ΠΎΠ΄ΠΈΠ½ Π½Π΅ΠΌΠ°Π»ΠΎΠ²Π°ΠΆΠ½Ρ‹ΠΉ Ρ„Π°ΠΊΡ‚ΠΎΡ€ – это ΠΊΠ°ΠΊΠΎΠΉ объСм Π²ΠΎΠ΄Ρ‹ Ρƒ нас Π²Ρ‹Π»ΡŒΠ΅Ρ‚ΡΡ ΠΈΠ· шланга Π·Π° ΠΊΠ°ΠΊΠΎΠ΅-Ρ‚ΠΎ врСмя.

Как Π΄ΡƒΠΌΠ°Π΅Ρ‚Π΅, с ΠΊΠ°ΠΊΠΈΠΌ Π½Π°ΠΏΠΎΡ€ΠΎΠΌ Π²ΠΎΠ΄Ρ‹ ΠΌΡ‹ быстрСС Π½Π°ΠΏΠΎΠ»Π½ΠΈΠΌ Π²Π΅Π΄Π΅Ρ€ΠΊΠΎ?

Π‘ Ρ‚Π°ΠΊΠΈΠΌ

Как Ρ‡ΠΈΡ‚Π°Ρ‚ΡŒ схСмы. НапряТСниС ΠΈ сила Ρ‚ΠΎΠΊΠ°

ΠΈΠ»ΠΈ с Ρ‚Π°ΠΊΠΈΠΌ?

Как Ρ‡ΠΈΡ‚Π°Ρ‚ΡŒ схСмы. НапряТСниС ΠΈ сила Ρ‚ΠΎΠΊΠ°

ΠΈΠ»ΠΈ Π²ΠΎΡ‚ с Ρ‚Π°ΠΊΠΈΠΌ?

Как Ρ‡ΠΈΡ‚Π°Ρ‚ΡŒ схСмы. НапряТСниС ΠΈ сила Ρ‚ΠΎΠΊΠ°

ΠŸΠΎΠ½ΡΡ‚Π½ΠΎΠ΅ Π΄Π΅Π»ΠΎ, Ρ‡Ρ‚ΠΎ с послСдним. ΠŸΠΎΡ‡Π΅ΠΌΡƒ Ρ‚Π°ΠΊ? Π”Π° ΠΏΠΎΡ‚ΠΎΠΌΡƒ Ρ‡Ρ‚ΠΎ, Π½Ρƒ ΠΏΡƒΡΡ‚ΡŒ скаТСм Π·Π° сСкунду, Ρƒ нас Π²Ρ‹Π»ΠΈΡ‚ΠΎΠΉ ΠΈΠ· Ρ‚Ρ€ΡƒΠ±Ρ‹ Π²ΠΎΠ΄Ρ‹ Π±ΡƒΠ΄Π΅Ρ‚ большС, Ρ‡Π΅ΠΌ ΠΈΠ· шланга. А объСм Π²Ρ‹Π»ΠΈΡ‚ΠΎΠΉ Π²ΠΎΠ΄Ρ‹ ΠΈΠ· Π·Π΅Π»Π΅Π½ΠΎΠ³ΠΎ шланга Π·Π° сСкунду Π±ΡƒΠ΄Π΅Ρ‚ большС, Ρ‡Π΅ΠΌ ΠΈΠ· ΠΆΠ΅Π»Ρ‚ΠΎΠ³ΠΎ, Ρ‚Π°ΠΊ ΠΊΠ°ΠΊ Π½Π°ΠΏΠΎΡ€ Π²ΠΎΠ΄Ρ‹ Π² ΠΆΠ΅Π»Ρ‚ΠΎΠΌ шлангС ΠΎΡ‡Π΅Π½ΡŒ слабый. И Ρ‚Π΅ΠΏΠ΅Ρ€ΡŒ Π΅Ρ‰Π΅ ΠΎΠ΄ΠΈΠ½ вопросик Π½Π° посошок. Какой ΠΏΠΎΡ‚ΠΎΠΊ струи Π±ΡƒΠ΄Π΅Ρ‚ ΠΎΠ±Π»Π°Π΄Π°Ρ‚ΡŒ бОльшСй силой? Ясно Π΄Π΅Π»ΠΎ, Ρ‡Ρ‚ΠΎ струя, которая Π²Ρ‹Ρ…ΠΎΠ΄ΠΈΡ‚ ΠΈΠ· Ρ‚Ρ€ΡƒΠ±Ρ‹. Π’Π°ΠΊΠΎΠΉ струСй ΠΌΠΎΠΆΠ½ΠΎ ΠΈ Π³ΠΈΠ΄Ρ€ΠΎΠ³Π΅Π½Π΅Ρ€Π°Ρ‚ΠΎΡ€Ρ‹ ΠΊΡ€ΡƒΡ‚ΠΈΡ‚ΡŒ.

Π”Π°Π²Π°ΠΉΡ‚Π΅ допустим, Ρ‡Ρ‚ΠΎ Ρƒ нас  Π΅ΡΡ‚ΡŒ большая Ρ‚Ρ€ΡƒΠ±Π°, ΠΈ ΠΊ Π½Π΅ΠΉ Π·Π°Π²Π°Ρ€Π΅Π½Ρ‹ Π΄Π²Π΅ Π΄Ρ€ΡƒΠ³ΠΈΠ΅, Π½ΠΎ ΠΎΠ΄Π½Π° Π² Π΄Π²Π° Ρ€Π°Π·Π° мСньшС Π΄ΠΈΠ°ΠΌΠ΅Ρ‚Ρ€ΠΎΠΌ, Ρ‡Π΅ΠΌ другая.

Из ΠΊΠ°ΠΊΠΎΠΉ Ρ‚Ρ€ΡƒΠ±Ρ‹ объСм Π²ΠΎΠ΄Ρ‹ Π±ΡƒΠ΄Π΅Ρ‚ Π²Ρ‹Ρ…ΠΎΠ΄ΠΈΡ‚ΡŒ большС Π·Π° сСкунду Π²Ρ€Π΅ΠΌΠ΅Π½ΠΈ? РазумССтся с Ρ‚ΠΎΠΉ, которая Ρ‚ΠΎΠ»Ρ‰Π΅ Π² Π΄ΠΈΠ°ΠΌΠ΅Ρ‚Ρ€Π΅, ΠΏΠΎΡ‚ΠΎΠΌΡƒ Ρ‡Ρ‚ΠΎ ΠΏΠ»ΠΎΡ‰Π°Π΄ΡŒ ΠΏΠΎΠΏΠ΅Ρ€Π΅Ρ‡Π½ΠΎΠ³ΠΎ сСчСния S2 большой Ρ‚Ρ€ΡƒΠ±Ρ‹ большС, Ρ‡Π΅ΠΌ ΠΏΠ»ΠΎΡ‰Π°Π΄ΡŒ ΠΏΠΎΠΏΠ΅Ρ€Π΅Ρ‡Π½ΠΎΠ³ΠΎ сСчСния S1 ΠΌΠ°Π»ΠΎΠΉ Ρ‚Ρ€ΡƒΠ±Ρ‹. Π‘Π»Π΅Π΄ΠΎΠ²Π°Ρ‚Π΅Π»ΡŒΠ½ΠΎ, сила ΠΏΠΎΡ‚ΠΎΠΊΠ° Ρ‡Π΅Ρ€Π΅Π· Π±ΠΎΠ»ΡŒΡˆΡƒΡŽ Ρ‚Ρ€ΡƒΠ±Ρƒ Π±ΡƒΠ΄Π΅Ρ‚ большС, Ρ‡Π΅ΠΌ Ρ‡Π΅Ρ€Π΅Π· ΠΌΠ°Π»ΡƒΡŽ, Ρ‚Π°ΠΊ ΠΊΠ°ΠΊ объСм Π²ΠΎΠ΄Ρ‹, ΠΊΠΎΡ‚ΠΎΡ€Ρ‹ΠΉ ΠΏΡ€ΠΎΡ‚Π΅ΠΊΠ°Π΅Ρ‚ Ρ‡Π΅Ρ€Π΅Π· ΠΏΠΎΠΏΠ΅Ρ€Π΅Ρ‡Π½ΠΎΠ΅ сСчСниС Ρ‚Ρ€ΡƒΠ±Ρ‹ S2, Π±ΡƒΠ΄Π΅Ρ‚Β  Π² Π΄Π²Π° Ρ€Π°Π·Π° большС, Ρ‡Π΅ΠΌ Ρ‡Π΅Ρ€Π΅Π· Ρ‚ΠΎΠ½ΠΊΡƒΡŽ Ρ‚Ρ€ΡƒΠ±Ρƒ.

Вак… Ρ‚Π΅ΠΏΠ΅Ρ€ΡŒ Π΄Π°Π²Π°ΠΉΡ‚Π΅ всС Ρ‡Ρ‚ΠΎ ΠΌΡ‹ Ρ‚ΡƒΡ‚ пописали ΠΏΡ€ΠΎ Π²ΠΎΠ΄ΠΈΡ‡ΠΊΡƒ, ΠΏΡ€ΠΈΠΌΠ΅Π½ΠΈΠΌ Π² элСктроникС. ΠŸΡ€ΠΎΠ²ΠΎΠ΄ΠΊΠΈ – это шланги ΠΈΠ»ΠΈ Ρ‚Ρ€ΡƒΠ±Ρ‹, Π² зависимости ΠΎΡ‚ Ρ€Π°Π·ΠΌΠ΅Ρ€Π°. Π’ΠΎΠ½ΠΊΠΈΠΉ ΠΏΡ€ΠΎΠ²ΠΎΠ΄ΠΎΠΊ – это Ρ‚ΠΎΠ½ΠΊΠΈΠΉ Π² Π΄ΠΈΠ°ΠΌΠ΅Ρ‚Ρ€Π΅ шланг, толстый ΠΏΡ€ΠΎΠ²ΠΎΠ΄ΠΎΠΊ – это толстый Π² Π΄ΠΈΠ°ΠΌΠ΅Ρ‚Ρ€Π΅ шланг, ΠΌΠΎΠΆΠ½ΠΎ ΡΠΊΠ°Π·Π°Ρ‚ΡŒ – Ρ‚Ρ€ΡƒΠ±Π°. ΠœΠΎΠ»Π΅ΠΊΡƒΠ»Ρ‹ Π²ΠΎΠ΄Ρ‹ – это элСктроны. Π‘Π»Π΅Π΄ΠΎΠ²Π°Ρ‚Π΅Π»ΡŒΠ½ΠΎ, толстый ΠΏΡ€ΠΎΠ²ΠΎΠ΄ΠΎΠΊ ΠΏΡ€ΠΈ ΠΎΠ΄ΠΈΠ½Π°ΠΊΠΎΠ²ΠΎΠΌ напряТСнии ΠΌΠΎΠΆΠ½ΠΎ ΠΏΡ€ΠΎΡ‚Π°Ρ‰ΠΈΡ‚ΡŒ большС элСктронов, Ρ‡Π΅ΠΌ Ρ‚ΠΎΠ½ΠΊΠΈΠΉ.

И Π΅Ρ‰Π΅, Π² ΠΊΠ°ΠΊΠΎΠΉ Ρ‚Ρ€ΡƒΠ±Π΅ сила ΠΏΠΎΡ‚ΠΎΠΊΠ° элСктронов Π±ΡƒΠ΄Π΅Ρ‚ большС? РазумССтся, Ρ‡Π΅Ρ€Π΅Π· толстый ΠΏΡ€ΠΎΠ²ΠΎΠ΄ΠΎΠΊ, Ρ‚Π°ΠΊ ΠΊΠ°ΠΊ количСство элСктронов Ρ‡Π΅Ρ€Π΅Π· ΠΏΠΎΠΏΠ΅Ρ€Π΅Ρ‡Π½ΠΎΠ΅ сСчСниС ΠΏΡ€ΠΎΠ²ΠΎΠ΄ΠΊΠ° Π·Π° Π΅Π΄ΠΈΠ½ΠΈΡ†Ρƒ Π²Ρ€Π΅ΠΌΠ΅Π½ΠΈ Π±ΡƒΠ΄Π΅Ρ‚ ΠΏΡ€ΠΎΡ…ΠΎΠ΄ΠΈΡ‚ΡŒ большС, Ρ‡Π΅ΠΌ Π² Ρ‚ΠΎΠ½ΠΊΠΎΠΌ ΠΏΡ€ΠΎΠ²ΠΎΠ΄ΠΊΠ΅ πŸ˜‰ А количСство элСктронов, ΠΊΠΎΡ‚ΠΎΡ€ΠΎΠ΅ ΠΏΡ€ΠΎΡ…ΠΎΠ΄ΠΈΡ‚ Ρ‡Π΅Ρ€Π΅Π· ΠΏΠΎΠΏΠ΅Ρ€Π΅Ρ‡Π½ΠΎΠ΅ сСчСниС ΠΏΡ€ΠΎΠ²ΠΎΠ΄Π½ΠΈΠΊΠ° Π·Π° ΠΊΠ°ΠΊΠΎΠΉ-Ρ‚ΠΎ ΠΏΡ€ΠΎΠΌΠ΅ΠΆΡƒΡ‚ΠΎΠΊ Π²Ρ€Π΅ΠΌΠ΅Π½ΠΈ, называСтся силой Ρ‚ΠΎΠΊΠ°. Π― вСдь Π³ΠΎΠ²ΠΎΡ€ΠΈΠ», Ρ‡Ρ‚ΠΎ Π³ΠΈΠ΄Ρ€Π°Π²Π»ΠΈΠΊΠ° ΠΈ элСктроника ΠΎΡ‡Π΅Π½ΡŒ взаимосвязаны ;-).

НС Π·Π°Π±Ρ‹Π²Π°Π΅ΠΌ, Ρ‡Ρ‚ΠΎ элСктроны ΠΎΠ±Π»Π°Π΄Π°ΡŽΡ‚ зарядом, поэтому ΠΎΡ„ΠΈΡ†ΠΈΠ°Π»ΡŒΠ½Π°Ρ тСрминология силы Ρ‚ΠΎΠΊΠ° Π·Π²ΡƒΡ‡ΠΈΡ‚ Ρ‚Π°ΠΊ: сила Ρ‚ΠΎΠΊΠ°Β  – это физичСская Π²Π΅Π»ΠΈΡ‡ΠΈΠ½Π°, равная ΠΎΡ‚Π½ΠΎΡˆΠ΅Π½ΠΈΡŽ количСства заряда ΠΏΡ€ΠΎΡˆΠ΅Π΄ΡˆΠ΅Π³ΠΎ Ρ‡Π΅Ρ€Π΅Π· ΠΏΠΎΠ²Π΅Ρ€Ρ…Π½ΠΎΡΡ‚ΡŒ (Ρ‡ΠΈΡ‚Π°Π΅ΠΌ ΠΊΠ°ΠΊ Ρ‡Π΅Ρ€Π΅Π· ΠΏΠ»ΠΎΡ‰Π°Π΄ΡŒ ΠΏΠΎΠΏΠ΅Ρ€Π΅Ρ‡Π½ΠΎΠ³ΠΎ сСчСния) Π·Π° ΠΊΠ°ΠΊΠΎΠ΅-Ρ‚ΠΎ врСмя. Π˜Π·ΠΌΠ΅Ρ€ΡΠ΅Ρ‚ΡΡ ΠΊΠ°ΠΊ ΠšΡƒΠ»ΠΎΠ½/сСкунда. Π§Ρ‚ΠΎΠ±Ρ‹ ΡΡΠΊΠΎΠ½ΠΎΠΌΠΈΡ‚ΡŒ врСмя ΠΈ ΠΏΠΎ Π΄Ρ€ΡƒΠ³ΠΈΠΌ ΠΌΠΎΡ€Π°Π»ΡŒΠ½ΠΎ-эстСтичСским Π½ΠΎΡ€ΠΌΠ°ΠΌ, ΠšΡƒΠ»ΠΎΠ½/сСкунду Π΄ΠΎΠ³ΠΎΠ²ΠΎΡ€ΠΈΠ»ΠΈΡΡŒ Π½Π°Π·Ρ‹Π²Π°Ρ‚ΡŒ АмпСром, Π² Ρ‡Π΅ΡΡ‚ΡŒ французского ΡƒΡ‡Π΅Π½ΠΎΠ³ΠΎ-Ρ„ΠΈΠ·ΠΈΠΊΠ°.

Π”Π°Π²Π°ΠΉΡ‚Π΅ Π΅Ρ‰Π΅ Ρ€Π°Π· глянСм Π½Π° шланг с Π²ΠΎΠ΄ΠΎΠΉ ΠΈ Π·Π°Π΄Π°Π΄ΠΈΠΌ сСбС вопросы. ΠžΡ‚ Ρ‡Π΅Π³ΠΎ зависит ΠΏΠΎΡ‚ΠΎΠΊ Π²ΠΎΠ΄Ρ‹? ΠŸΠ΅Ρ€Π²ΠΎΠ΅, Ρ‡Ρ‚ΠΎ ΠΏΡ€ΠΈΡ…ΠΎΠ΄ΠΈΡ‚ Π² Π³ΠΎΠ»ΠΎΠ²Ρƒ – это Π΄Π°Π²Π»Π΅Π½ΠΈΠ΅. ΠŸΠΎΡ‡Π΅ΠΌΡƒ ΠΌΠΎΠ»Π΅ΠΊΡƒΠ»Ρ‹ Π²ΠΎΠ΄Ρ‹ двиТутся Π² рисункС Π½ΠΈΠΆΠ΅ слСва-Π½Π°ΠΏΡ€Π°Π²ΠΎ? ΠŸΠΎΡ‚ΠΎΠΌΡƒ, Ρ‡Ρ‚ΠΎ Π΄Π°Π²Π»Π΅Π½ΠΈΠ΅ слСва, большС Ρ‡Π΅ΠΌ справа. Π§Π΅ΠΌ большС Π΄Π°Π²Π»Π΅Π½ΠΈΠ΅, Ρ‚Π΅ΠΌ быстрСС ΠΏΠΎΠ±Π΅ΠΆΠΈΡ‚ Π²ΠΎΠ΄ΠΈΡ‡ΠΊΠ° ΠΏΠΎ ΡˆΠ»Π°Π½Π³Ρƒ – это элСмСнтарно.

Π’Π΅ΠΏΠ΅Ρ€ΡŒ Ρ‚Π°ΠΊΠΎΠΉ вопрос: ΠΊΠ°ΠΊ ΠΌΠΎΠΆΠ½ΠΎ ΡƒΠ²Π΅Π»ΠΈΡ‡ΠΈΡ‚ΡŒ количСство элСктронов Ρ‡Π΅Ρ€Π΅Π· ΠΏΠ»ΠΎΡ‰Π°Π΄ΡŒ ΠΏΠΎΠΏΠ΅Ρ€Π΅Ρ‡Π½ΠΎΠ³ΠΎ сСчСния? ΠŸΠ΅Ρ€Π²ΠΎΠ΅, Ρ‡Ρ‚ΠΎ ΠΏΡ€ΠΈΡ…ΠΎΠ΄ΠΈΡ‚ Π½Π° ΡƒΠΌ – это ΡƒΠ²Π΅Π»ΠΈΡ‡ΠΈΡ‚ΡŒ Π΄Π°Π²Π»Π΅Π½ΠΈΠ΅. Π’ этом случаС ΡΠΊΠΎΡ€ΠΎΡΡ‚ΡŒ ΠΏΠΎΡ‚ΠΎΠΊΠ° Π²ΠΎΠ΄Ρ‹ увСличится,Β  Π½ΠΎ Π΅Π΅ ΠΌΠ½ΠΎΠ³ΠΎ Π½Π΅ ΡƒΠ²Π΅Π»ΠΈΡ‡ΠΈΡˆΡŒ, Ρ‚Π°ΠΊ ΠΊΠ°ΠΊ шланг порвСтся ΠΊΠ°ΠΊ Π³Ρ€Π΅Π»ΠΊΠ° Π² пасти Π’ΡƒΠ·ΠΈΠΊΠ°. Π’Ρ‚ΠΎΡ€ΠΎΠ΅ – это ΠΏΠΎΡΡ‚Π°Π²ΠΈΡ‚ΡŒ шланг большим Π΄ΠΈΠ°ΠΌΠ΅Ρ‚Ρ€ΠΎΠΌ. Π’ этом случаС Ρƒ нас количСство ΠΌΠΎΠ»Π΅ΠΊΡƒΠ» Π²ΠΎΠ΄Ρ‹ Ρ‡Π΅Ρ€Π΅Π· ΠΏΠΎΠΏΠ΅Ρ€Π΅Ρ‡Π½ΠΎΠ΅ сСчСниС Π±ΡƒΠ΄Π΅Ρ‚ ΠΏΡ€ΠΎΡ…ΠΎΠ΄ΠΈΡ‚ΡŒ большС, Ρ‡Π΅ΠΌ Π² Ρ‚ΠΎΠ½ΠΊΠΎΠΌ шлангС:

ВсС Ρ‚Π΅ ΠΆΠ΅ самыС ΡƒΠΌΠΎΠ·Π°ΠΊΠ»ΡŽΡ‡Π΅Π½ΠΈΡ ΠΌΠΎΠΆΠ½ΠΎ ΠΏΡ€ΠΈΠΌΠ΅Π½ΠΈΡ‚ΡŒ ΠΈ ΠΊ ΠΎΠ±Ρ‹ΠΊΠ½ΠΎΠ²Π΅Π½Π½ΠΎΠΌΡƒ ΠΏΡ€ΠΎΠ²ΠΎΠ΄ΠΊΡƒ. Π§Π΅ΠΌ ΠΎΠ½ большС Π² Π΄ΠΈΠ°ΠΌΠ΅Ρ‚Ρ€Π΅, Ρ‚Π΅ΠΌ большС ΠΎΠ½ смоТСт ΠΏΡ€ΠΎΡ‚Π°Ρ‰ΠΈΡ‚ΡŒ Ρ‡Π΅Ρ€Π΅Π· сСбя силу Ρ‚ΠΎΠΊΠ°. Π§Π΅ΠΌ мСньшС Π² Π΄ΠΈΠ°ΠΌΠ΅Ρ‚Ρ€Π΅, Ρ‚ΠΎ ΠΆΠ΅Π»Π°Ρ‚Π΅Π»ΡŒΠ½ΠΎ мСньшС Π΅Π³ΠΎ Π½Π°Π³Ρ€ΡƒΠΆΠ°Ρ‚ΡŒ, ΠΈΠ½Π°Ρ‡Π΅ Π΅Π³ΠΎ β€œΠΏΠΎΡ€Π²Π΅Ρ‚β€, Ρ‚ΠΎ Π΅ΡΡ‚ΡŒ ΠΎΠ½ Ρ‚ΡƒΠΏΠΎ сгорит. ИмСнно этот ΠΏΡ€ΠΈΠ½Ρ†ΠΈΠΏ Π·Π°Π»ΠΎΠΆΠ΅Π½ Π² ΠΏΠ»Π°Π²ΠΊΠΈΡ… прСдохранитСлях. Π’Π½ΡƒΡ‚Ρ€ΠΈ Ρ‚Π°ΠΊΠΎΠ³ΠΎ прСдохранитСля Ρ‚ΠΎΠ½ΠΊΠΈΠΉ ΠΏΡ€ΠΎΠ²ΠΎΠ΄ΠΎΠΊ. Π•Π³ΠΎ Ρ‚ΠΎΠ»Ρ‰ΠΈΠ½Π° зависит  ΠΎΡ‚ Ρ‚ΠΎΠ³ΠΎ, Π½Π° ΠΊΠ°ΠΊΡƒΡŽ силу Ρ‚ΠΎΠΊΠ° ΠΎΠ½ рассчитан

Как Ρ‡ΠΈΡ‚Π°Ρ‚ΡŒ схСмы. НапряТСниС ΠΈ сила Ρ‚ΠΎΠΊΠ°

Как Ρ‚ΠΎΠ»ΡŒΠΊΠΎ сила Ρ‚ΠΎΠΊΠ° Ρ‡Π΅Ρ€Π΅Π· ΠΏΡ€ΠΎΠ²ΠΎΠ΄ΠΎΠΊ прСвысит силу Ρ‚ΠΎΠΊΠ°, Π½Π° ΠΊΠΎΡ‚ΠΎΡ€ΡƒΡŽ рассчитан ΠΏΡ€Π΅Π΄ΠΎΡ…Ρ€Π°Π½ΠΈΡ‚Π΅Π»ΡŒ, Ρ‚ΠΎ ΠΏΠ»Π°Π²ΠΊΠΈΠΉ ΠΏΡ€ΠΎΠ²ΠΎΠ΄ΠΎΠΊ ΠΏΠ΅Ρ€Π΅Π³ΠΎΡ€Π°Π΅Ρ‚ ΠΈ Ρ€Π°Π·ΠΌΡ‹ΠΊΠ°Π΅Ρ‚ Ρ†Π΅ΠΏΡŒ. Π§Π΅Ρ€Π΅Π· ΠΏΠ΅Ρ€Π΅Π³ΠΎΡ€Π΅Π²ΡˆΠΈΠΉ ΠΏΡ€Π΅Π΄ΠΎΡ…Ρ€Π°Π½ΠΈΡ‚Π΅Π»ΡŒ Ρ‚ΠΎΠΊ ΡƒΠΆΠ΅ Ρ‚Π΅Ρ‡ΡŒ Π½Π΅ ΠΌΠΎΠΆΠ΅Ρ‚, Ρ‚Π°ΠΊ ΠΊΠ°ΠΊ ΠΏΡ€ΠΎΠ²ΠΎΠ΄ΠΎΠΊ Π² ΠΎΠ±Ρ€Ρ‹Π²Π΅

Как Ρ‡ΠΈΡ‚Π°Ρ‚ΡŒ схСмы. НапряТСниС ΠΈ сила Ρ‚ΠΎΠΊΠ°

Π—Π°ΠΊΠ»ΡŽΡ‡Π΅Π½ΠΈΠ΅

ЭлСктричСский Ρ‚ΠΎΠΊ Π² основном характСризуСтся Ρ‚Π°ΠΊΠΈΠΌΠΈ ΠΏΠ°Ρ€Π°ΠΌΠ΅Ρ‚Ρ€Π°ΠΌΠΈ, ΠΊΠ°ΠΊ напряТСниС ΠΈ сила Ρ‚ΠΎΠΊΠ°. ΠŸΡ€ΠΎΠ²ΠΎΠ΄Π° слуТат ΠΈΠΌΠ΅Π½Π½ΠΎ Ρ‚Π΅ΠΌΠΈ самыми β€œΡ‚Ρ€ΡƒΠ±Π°ΠΌΠΈ ΠΈ ΡˆΠ»Π°Π½Π³Π°ΠΌΠΈβ€ для Ρ‚ΠΎΠ³ΠΎ, Ρ‡Ρ‚ΠΎΠ±Ρ‹ ΠΏΠ΅Ρ€Π΅Π΄Π°Π²Π°Ρ‚ΡŒ элСктричСский Ρ‚ΠΎΠΊ Π½Π° расстояния. Они Π²Ρ‹Π±ΠΈΡ€Π°ΡŽΡ‚ΡΡ Π² зависимости ΠΎΡ‚ Ρ‚ΠΎΠ³ΠΎ, какая сила Ρ‚ΠΎΠΊΠ° Π±ΡƒΠ΄Π΅Ρ‚ Ρ‚Π΅Ρ‡ΡŒ Ρ‡Π΅Ρ€Π΅Π· Π½ΠΈΡ….

НапримСр, Π²ΠΎΡ‚ Ρ‚Π°ΠΊΠΈΠ΅ ΠΌΠ΅Π΄Π½Ρ‹Π΅ β€œΠΏΡ€ΠΎΠ²ΠΎΠ΄ΠΎΡ‡ΠΊΠΈβ€ ΠΈΡΠΏΠΎΠ»ΡŒΠ·ΡƒΡŽΡ‚ΡΡ для ΠΏΠ΅Ρ€Π΅Π΄Π°Ρ‡ΠΈ бСшСной силы Ρ‚ΠΎΠΊΠ° Π½Π° Π·Π°Π²ΠΎΠ΄Π°Ρ…, ΠΊΡ€ΡƒΠΏΠ½Ρ‹Ρ… Ρ„Π°Π±Ρ€ΠΈΠΊΠ°Ρ…, элСктросСтях ΠΈ Ρ‚Π΄. ΠΠ°Π·Ρ‹Π²Π°ΡŽΡ‚ ΠΈΡ… ΠΌΠ΅Π΄Π½Ρ‹ΠΌΠΈ шинами.

Как Ρ‡ΠΈΡ‚Π°Ρ‚ΡŒ схСмы. НапряТСниС ΠΈ сила Ρ‚ΠΎΠΊΠ°

Как Ρ‡ΠΈΡ‚Π°Ρ‚ΡŒ схСмы. НапряТСниС ΠΈ сила Ρ‚ΠΎΠΊΠ°

На послСднСй ΠΊΠ°Ρ€Ρ‚ΠΈΠ½ΠΊΠ΅ ΠΌΠΎΠΆΠ½ΠΎ ΡƒΠ²ΠΈΠ΄Π΅Ρ‚ΡŒ ΠΏΡ€Π΅Π΄ΠΎΡ…Ρ€Π°Π½ΠΈΡ‚Π΅Π»ΡŒ, ΠΊΠΎΡ‚ΠΎΡ€Ρ‹ΠΉ соСдиняСт ΡˆΠΈΠ½Ρ‹. Π•Π³ΠΎ Π½ΠΎΠΌΠΈΠ½Π°Π» 500 АмпСр. МоТно ΡΠΊΠ°Π·Π°Ρ‚ΡŒ, Ρ‡Ρ‚ΠΎ Ρ‡Π΅Ρ€Π΅Π· сСчСниС Ρ‚Π°ΠΊΠΎΠΉ ΠΌΠ΅Π΄Π½ΠΎΠΉ ΡˆΠΈΠ½Ρ‹ Π·Π° 1 сСкунду ΠΌΠΎΠΆΠ΅Ρ‚ ΠΏΡ€ΠΎΠ±Π΅ΠΆΠ°Ρ‚ΡŒ ΠΎΡ‡Π΅Π½ΡŒ большой заряд, Π° Ρ‚ΠΎΡ‡Π½Π΅Π΅ 500 ΠšΡƒΠ»ΠΎΠ½.

А Ρ‡Ρ‚ΠΎ Π±Ρ‹Π»ΠΎ Π±Ρ‹, Ссли ΠΌΡ‹ Ρ‚ΡƒΠ΄Π° поставили ΠΊΠ°ΠΊΠΎΠΉ-Π½ΠΈΠ±ΡƒΠ΄ΡŒ ΠΌΠ΅Π΄Π½Ρ‹ΠΉ Ρ‚ΠΎΠ½ΠΊΠΈΠΉ ΠΏΡ€ΠΎΠ²ΠΎΠ΄ΠΎΠΊ? Π― Π΄ΡƒΠΌΠ°ΡŽ, ΠΏΡ€ΠΎΠΈΠ·ΠΎΡˆΠ»ΠΎ Π±Ρ‹ Ρ‡Ρ‚ΠΎ-Ρ‚ΠΎ Ρ‚ΠΈΠΏΠ° этого

Как Ρ‡ΠΈΡ‚Π°Ρ‚ΡŒ схСмы. НапряТСниС ΠΈ сила Ρ‚ΠΎΠΊΠ°Β 

Β 

РСзюмС

ЭлСктричСский Ρ‚ΠΎΠΊ – это Π΄Π²ΠΈΠΆΠ΅Π½ΠΈΠ΅ Π² ΠΎΠ΄Π½ΠΎΠΌ Π½Π°ΠΏΡ€Π°Π²Π»Π΅Π½ΠΈΠΈ свободных элСктронов.

Π‘Π²ΠΎΠ±ΠΎΠ΄Π½Ρ‹Π΅ элСктроны Ρƒ нас ΠΈΠΌΠ΅ΡŽΡ‚ΡΡ Π² ΠΏΡ€ΠΎΠ²ΠΎΠ΄ΠΊΠ°Ρ…, ΠΊΠΎΡ‚ΠΎΡ€Ρ‹Π΅ Π² основном сдСланы ΠΈΠ· ΠΌΠ΅Π΄ΠΈ ΠΈ алюминия.

ЭлСктричСский Ρ‚ΠΎΠΊ характСризуСтся двумя ΠΏΠ°Ρ€Π°ΠΌΠ΅Ρ‚Ρ€Π°ΠΌΠΈ: напряТСниСм ΠΈ силой Ρ‚ΠΎΠΊΠ°.

Π§Ρ‚ΠΎΠ±Ρ‹ Π² ΠΏΡ€ΠΎΠ²ΠΎΠ΄ΠΊΠ΅ Π²ΠΎΠ·Π½ΠΈΠΊ элСктричСский Ρ‚ΠΎΠΊ, Π½Π°Π΄ΠΎ Ρ‡Ρ‚ΠΎΠ±Ρ‹ Π² ΠΎΠ΄Π½ΠΎΠΌ ΠΊΠΎΠ½Ρ†Π΅ ΠΏΡ€ΠΎΠ²ΠΎΠ΄ΠΊΠ° Π±Ρ‹Π»ΠΎ ΠΈΠ·Π±Ρ‹Ρ‚ΠΎΡ‡Π½ΠΎΠ΅ Π΄Π°Π²Π»Π΅Π½ΠΈΠ΅, Π° Π² Π΄Ρ€ΡƒΠ³ΠΎΠΌΒ  – нСдостаточноС.

Π’ΠΎΠΊ Ρ‚Π΅Ρ‡Π΅Ρ‚ ΠΎΡ‚ плюса ΠΊ минусу (хотя элСктроны Π±Π΅Π³ΡƒΡ‚ ΠΎΡ‚ минуса ΠΊ ΠΏΠ»ΡŽΡΡƒ)

Π‘ΠΈΠ»Π° Ρ‚ΠΎΠΊΠ° Ρ‡Π΅Ρ€Π΅Π· ΠΏΡ€ΠΎΠ²ΠΎΠ΄ΠΎΠΊ – это количСство заряда, ΠΊΠΎΡ‚ΠΎΡ€ΠΎΠ΅ ΠΏΡ€ΠΎΡ…ΠΎΠ΄ΠΈΡ‚ Ρ‡Π΅Ρ€Π΅Π· ΠΏΠ»ΠΎΡ‰Π°Π΄ΡŒ β€œΠΊΡ€ΡƒΠΆΠΎΡ‡ΠΊΠ°β€ (сСчСниС ΠΏΡ€ΠΎΠ²ΠΎΠ΄ΠΊΠ° ΠΏΠΎΠΏΠ΅Ρ€Π΅ΠΊ) Π·Π° ΠΎΠ΄Π½Ρƒ сСкунду. ВыраТаСтся Π² АмпСрах (ΠšΡƒΠ»ΠΎΠ½/ Π’ΠΎΠ»ΡŒΡ‚).

ΠŸΡ€ΠΎΠ²ΠΎΠ΄ΠΊΠΈ, Ρ‡Π΅Ρ€Π΅Π· ΠΊΠΎΡ‚ΠΎΡ€Ρ‹Π΅ Π±ΡƒΠ΄Π΅Ρ‚ ΠΏΡ€ΠΎΡ…ΠΎΠ΄ΠΈΡ‚ΡŒ большая сила Ρ‚ΠΎΠΊΠ°, Π΄Π΅Π»Π°ΡŽΡ‚ Ρ‚ΠΎΠ»Ρ‰Π΅, ΠΈΠ½Π°Ρ‡Π΅ Ρ‚ΠΎΠ½ΠΊΠΈΠ΅ ΠΏΡ€ΠΎΠ²ΠΎΠ΄Π° Π½Π°Π³Ρ€Π΅ΡŽΡ‚ΡΡ ΠΈ расплавятся, ΠΏΡ€ΠΈΡ‡ΠΈΠ½ΠΈΠ² Π²Ρ€Π΅Π΄ ΠΎΠΊΡ€ΡƒΠΆΠ°ΡŽΡ‰ΠΈΠΌ ΠΏΡ€Π΅Π΄ΠΌΠ΅Ρ‚Π°ΠΌ.

Каким ΠΎΠ±Ρ€Π°Π·ΠΎΠΌ Ρ‚Π΅Ρ‡Π΅Ρ‚ элСктричСство?

ЭлСктричСский Ρ‚ΠΎΠΊ ΠΌΠΎΠΆΠ΅Ρ‚ ΠΏΡ€ΠΈΠ²ΠΎΠ΄ΠΈΡ‚ Π² дСйствиС ΠΌΠ°ΡˆΠΈΠ½Ρ‹ Ρ‚ΠΎΠ»ΡŒΠΊΠΎ Ρ‚ΠΎΠ³Π΄Π°, ΠΊΠΎΠ³Π΄Π° ΠΎΠ½ Ρ†ΠΈΡ€ΠΊΡƒΠ»ΠΈΡ€ΡƒΠ΅Ρ‚ Π² Ρ†Π΅ΠΏΠΈ. ЭлСктричСская Ρ†Π΅ΠΏΡŒ β€” это ΠΊΠ°Π½Π°Π», ΠΏΠΎ ΠΊΠΎΡ‚ΠΎΡ€ΠΎΠΌΡƒ Ρ‚Π΅Ρ‡Π΅Ρ‚ элСктричСство. НачинаСтся Ρ†Π΅ΠΏΡŒ Π² источникС питания (Π½Π°ΠΏΡ€ΠΈΠΌΠ΅Ρ€, Π² Π±Π°Ρ‚Π°Ρ€Π΅ΠΉΠΊΠ΅), ΠΊ ΠΊΠΎΡ‚ΠΎΡ€ΠΎΠΌΡƒ ΡΠΎΠ΅Π΄ΠΈΠ½ΠΈΡ‚Π΅Π»ΡŒΠ½Ρ‹ΠΌ ΠΏΡ€ΠΎΠ²ΠΎΠ΄ΠΎΠΌ ΠΏΠΎΠ΄ΠΊΠ»ΡŽΡ‡Π΅Π½ ΠΏΠΎΡ‚Ρ€Π΅Π±ΠΈΡ‚Π΅Π»ΡŒ, Π½Π°ΠΏΡ€ΠΈΠΌΠ΅Ρ€, Π»Π°ΠΌΠΏΠ° накаливания.

ЦСпь Π½Π΅ оканчиваСтся Π½Π° ΠΏΠΎΡ‚Ρ€Π΅Π±ΠΈΡ‚Π΅Π»Π΅, Π° возвращаСтся ΠΏΠΎ ΠΊΠΎΠ»ΡŒΡ†Ρƒ снова ΠΊ источнику питания. Π‘ΠΈΠ»Π°, ΠΏΠΎΠ΄Π΄Π΅Ρ€ΠΆΠΈΠ²Π°ΡŽΡ‰Π°Ρ Ρ‚Π΅Ρ‡Π΅Π½ΠΈΠ΅ элСктричСского Ρ‚ΠΎΠΊΠ° Π² Ρ†Π΅ΠΏΠΈ, называСтся элСктродвиТущСй силой, ΠΈΠ»ΠΈ напряТСниСм. Π’Π°ΠΊ ΠΊΠ°ΠΊ ΠΏΠΎΡ‚Ρ€Π΅Π±ΠΈΡ‚Π΅Π»ΠΈ ΠΎΡΠ»Π°Π±Π»ΡΡŽΡ‚ Ρ‚ΠΎΠΊ Π² Ρ†Π΅ΠΏΠΈ, ΠΎΠ½ΠΈ Π½Π°Π·Ρ‹Π²Π°ΡŽΡ‚ΡΡ сопротивлСниями.

ПониманиС взаимосвязи ΠΌΠ΅ΠΆΠ΄Ρƒ элСктричСским Ρ‚ΠΎΠΊΠΎΠΌ, напряТСниСм ΠΈ сопротивлСниСм ΠΌΠΎΠΆΠ΅Ρ‚ Π±Ρ‹Ρ‚ΡŒ ΠΎΠ±Π»Π΅Π³Ρ‡Π΅Π½ΠΎ ΠΏΡƒΡ‚Π΅ΠΌ провСдСния Π°Π½Π°Π»ΠΎΠ³ΠΈΠΈ ΠΌΠ΅ΠΆΠ΄Ρƒ элСктричСским Ρ‚ΠΎΠΊΠΎΠΌ ΠΈ Π²ΠΎΠ΄ΠΎΠΉ, Ρ‚Π΅ΠΊΡƒΡ‰Π΅ΠΉ ΠΏΠΎ ΠΊΠ°Π½Π°Π»Ρƒ (рисунок Π²Π²Π΅Ρ€Ρ…Ρƒ). Π‘Π°Ρ‚Π°Ρ€Π΅ΠΉΠΊΠ° ΠΌΠΎΠΆΠ΅Ρ‚ Π±Ρ‹Ρ‚ΡŒ прСдставлСна Π² Π²ΠΈΠ΄Π΅ водяного насоса, Π° элСктричСский Ρ‚ΠΎΠΊ β€” Π² Π²ΠΈΠ΄Π΅ ΠΎΠΏΡ€Π΅Π΄Π΅Π»Π΅Π½Π½ΠΎΠ³ΠΎ объСма Π²ΠΎΠ΄Ρ‹. Аналогами Π΄Π²ΡƒΡ… элСктричСских сопротивлСний (Π΄Π²ΡƒΡ… Π»Π°ΠΌΠΏ накаливания) ΡΠ²Π»ΡΡŽΡ‚ΡΡ Π΄Π²Π° водослива Π² ΠΊΠ°Π½Π°Π»Π΅.

Π’ Ρ‚Π°ΠΊΠΎΠΉ ΠΌΠΎΠ΄Π΅Π»ΠΈ ΠΊΠ°ΠΆΠ΄Ρ‹ΠΉ Ρ€Π°Π·, ΠΊΠΎΠ³Π΄Π° Π²ΠΎΠ΄Π° (элСктричСский Ρ‚ΠΎΠΊ) встрСчаСт водослив (сопротивлСниС), ΠΎΠ½Π° ΠΏΠ°Π΄Π°Π΅Ρ‚ Π½Π° Π±ΠΎΠ»Π΅Π΅ Π½ΠΈΠ·ΠΊΠΈΠΉ ΡƒΡ€ΠΎΠ²Π΅Π½ΡŒ (мСньшСС напряТСниС). ОбъСм Π²ΠΎΠ΄Ρ‹ остаСтся Π½Π΅ΠΈΠ·ΠΌΠ΅Π½Π½Ρ‹ΠΌ, ΠΎΠ΄Π½Π°ΠΊΠΎ Π΅Π΅ ΡƒΡ€ΠΎΠ²Π΅Π½ΡŒ (энСргия) ΡƒΠΌΠ΅Π½ΡŒΡˆΠ°Π΅Ρ‚ΡΡ. Π’ΠΎ ΠΆΠ΅ самоС происходит с элСктричСским Ρ‚ΠΎΠΊΠΎΠΌ. Когда элСктричСский Ρ‚ΠΎΠΊ ΠΏΡ€ΠΎΡ…ΠΎΠ΄ΠΈΡ‚ Ρ‡Π΅Ρ€Π΅Π· сопротивлСниС, Π΅Π³ΠΎ энСргия отводится Π² ΠΎΠΊΡ€ΡƒΠΆΠ°ΡŽΡ‰ΡƒΡŽ срСду, Π° напряТСниС ΡƒΠΌΠ΅Π½ΡŒΡˆΠ°Π΅Ρ‚ΡΡ.

ВычислСниС падСния напряТСния

Когда элСктричСский Ρ‚ΠΎΠΊ ΠΏΡ€ΠΎΡ…ΠΎΠ΄ΠΈΡ‚ Ρ‡Π΅Ρ€Π΅Π· сопротивлСниС, Π½Π°ΠΏΡ€ΠΈΠΌΠ΅Ρ€, Ρ‡Π΅Ρ€Π΅Π· Π»Π°ΠΌΠΏΡƒ накаливания, силовоС воздСйствиС Π½Π° заряды (напряТСниС) ΡƒΠΌΠ΅Π½ΡŒΡˆΠ°Π΅Ρ‚ΡΡ. Π­Ρ‚ΠΎ ΡƒΠΌΠ΅Π½ΡŒΡˆΠ΅Π½ΠΈΠ΅ называСтся ΠΏΠ°Π΄Π΅Π½ΠΈΠ΅ΠΌ напряТСния. ИзмСнСниС напряТСния ΠΌΠΎΠΆΠ΅Ρ‚ Π±Ρ‹Ρ‚ΡŒ ΠΎΠΏΡ€Π΅Π΄Π΅Π»Π΅Π½ΠΎ числСнно, ΠΏΡƒΡ‚Π΅ΠΌ умноТСния Π²Π΅Π»ΠΈΡ‡ΠΈΠ½Ρ‹ сопротивлСния Π½Π° силу Ρ‚ΠΎΠΊΠ°.

ЭлСктричСский Ρ‚ΠΎΠΊ ΠΈ ΠΏΠΎΡ‚ΠΎΠΊ элСктронов

Π­Π»Π΅ΠΊΡ‚Ρ€ΠΎΠ½Ρ‹ (синиС ΡˆΠ°Ρ€ΠΈΠΊΠΈ) Ρ‚Π΅ΠΊΡƒΡ‚ ΠΏΠΎ Π½Π°ΠΏΡ€Π°Π²Π»Π΅Π½ΠΈΡŽ ΠΊ ΠΏΠΎΠ»ΠΎΠΆΠΈΡ‚Π΅Π»ΡŒΠ½ΠΎΠΌΡƒ ΠΏΠΎΠ»ΡŽΡΡƒ источника Ρ‚ΠΎΠΊΠ°, Ρ‚.Π΅. навстрСчу элСктричСскому Ρ‚ΠΎΠΊΡƒ, ΠΊΠΎΡ‚ΠΎΡ€Ρ‹ΠΉ двиТСтся ΠΎΡ‚ ΠΏΠΎΠ»ΠΎΠΆΠΈΡ‚Π΅Π»ΡŒΠ½ΠΎΠ³ΠΎ полюса ΠΊ ΠΎΡ‚Ρ€ΠΈΡ†Π°Ρ‚Π΅Π»ΡŒΠ½ΠΎΠΌΡƒ (большая голубая стрСлка). Π‘ΠΈΠ»Π° Ρ‚ΠΎΠΊΠ° зависит ΠΎΡ‚ Ρ‚ΠΎΠ³ΠΎ, сколько элСктронов ΠΏΡ€ΠΎΠΉΠ΄Π΅Ρ‚ Ρ‡Π΅Ρ€Π΅Π· ΠΏΠΎΠΏΠ΅Ρ€Π΅Ρ‡Π½ΠΎΠ΅ сСчСниС ΠΏΡ€ΠΎΠ²ΠΎΠ΄Π½ΠΈΠΊΠ° Π² Π΅Π΄ΠΈΠ½ΠΈΡ†Ρƒ Π²Ρ€Π΅ΠΌΠ΅Π½ΠΈ.

ЭлСктричСский Ρ‚ΠΎΠΊ Π² ΠΏΠ°Ρ€Π°Π»Π»Π΅Π»ΡŒΠ½ΠΎΠΉ Ρ†Π΅ΠΏΠΈ

Π’ ΠΏΠ°Ρ€Π°Π»Π»Π΅Π»ΡŒΠ½ΠΎΠΉ Ρ†Π΅ΠΏΠΈ элСктричСский Ρ‚ΠΎΠΊ (синиС стрСлки), ΠΏΡ€Π΅ΠΆΠ΄Π΅ Ρ‡Π΅ΠΌ Π²Π΅Ρ€Π½ΡƒΡ‚ΡŒΡΡ ΠΊ своСму источнику (красная Π±Π°Ρ‚Π°Ρ€Π΅ΠΉΠΊΠ°), раздСляСтся Π½Π° Π΄Π²Π΅ ΠΎΡ‚Π΄Π΅Π»ΡŒΠ½Ρ‹Π΅ Π²Π΅Ρ‚Π²ΠΈ.

Π’ΠΈΠ΄ Ρ†Π΅ΠΏΠΈ ΠΈ напряТСниС

ΠŸΠΎΡΠ»Π΅Π΄ΠΎΠ²Π°Ρ‚Π΅Π»ΡŒΠ½Π°Ρ Ρ†Π΅ΠΏΡŒ содСрТит Π΄Π²Π° сопротивлСния (R), ΠΊΠΎΡ‚ΠΎΡ€Ρ‹Π΅ ΠΏΠΎΠΎΡ‡Π΅Ρ€Π΅Π΄Π½ΠΎ ΡΠ½ΠΈΠΆΠ°ΡŽΡ‚ напряТСниС (V). ПадСниС напряТСния опрСдСляСтся суммой сопротивлСний.

Π’ ΠΏΠ°Ρ€Π°Π»Π»Π΅Π»ΡŒΠ½ΠΎΠΉ Ρ†Π΅ΠΏΠΈ элСктричСский Ρ‚ΠΎΠΊ ΠΏΡ€ΠΎΡ…ΠΎΠ΄ΠΈΡ‚ ΠΏΠΎ Ρ€Π°Π·Π»ΠΈΡ‡Π½Ρ‹ΠΌ путям. Π’Π°ΠΊΠΎΠ΅ располоТСниС сопротивлСний (R) Π²Ρ‹Π·Ρ‹Π²Π°Π΅Ρ‚ ΠΎΠ΄Π½ΠΎΠ²Ρ€Π΅ΠΌΠ΅Π½Π½ΠΎΠ΅ ΠΏΠ°Π΄Π΅Π½ΠΈΠ΅ напряТСния.

ЭлСктричСский Ρ‚ΠΎΠΊ ΠΈ элСктричСская Ρ†Π΅ΠΏΡŒ

БтатичСскоС элСктричСство. Если ΠΆΠ΅Π»Ρ‚Ρ‹ΠΉ ΡΠ½Ρ‚Π°Ρ€ΡŒ ΠΏΠΎΡ‚Π΅Ρ€Π΅Ρ‚ΡŒ ΡˆΠ΅Ρ€ΡΡ‚ΡŒΡŽ ΠΈΠ»ΠΈ ΠΌΠ΅Ρ…ΠΎΠΌ, Ρ‚ΠΎ ΡΠ½Ρ‚Π°Ρ€ΡŒ ΠΏΡ€ΠΈΠΎΠ±Ρ€Π΅Ρ‚Π°Π΅Ρ‚ свойство Π΄Π»ΠΈΡ‚Π΅Π»ΡŒΠ½ΠΎΠ΅ врСмя ΠΏΡ€ΠΈΡ‚ΡΠ³ΠΈΠ²Π°Ρ‚ΡŒ ,ΠΊ сСбС волосы, Π»ΠΈΡΡ‚ΡŒΡ, соломинки. Π‘ΠΏΠΎΡΠΎΠ±Π½ΠΎΡΡ‚ΡŒ янтаря ,ΠΏΡ€ΠΈΡ‚ΡΠ³ΠΈΠ²Π°Ρ‚ΡŒ ΠΊ сСбС Π΄Ρ€ΡƒΠ³ΠΈΠ΅ вСщСства вызываСтся Π΅Π³ΠΎ зарядом. Под зарядом Ρ‚Π΅Π» ΠΏΠΎΠ΄Ρ€Π°Π·ΡƒΠΌΠ΅Π²Π°ΡŽΡ‚ элСктричСский заряд. ΠŸΡ€ΠΈ ΠΎΠΏΡ€Π΅Π΄Π΅Π»Π΅Π½Π½Ρ‹Ρ… условиях заряд сохраняСтся Π½Π° заряТСнных Ρ‚Π΅Π»Π°Ρ…, поэтому Π΅Π³ΠΎ Π½Π°Π·Ρ‹Π²Π°ΡŽΡ‚ статичСским элСктричСством.

Π’Π΅Π»ΠΈΡ‡ΠΈΠ½Ρ‹ количСства элСктричСства заряТСнных Ρ‚Π΅Π» ΠΈ расстояния ΠΌΠ΅ΠΆΠ΄Ρƒ Π½ΠΈΠΌΠΈ ΠΎΠΊΠ°Π·Ρ‹Π²Π°ΡŽΡ‚ влияниС Π½Π° ΠΈΡ… взаимодСйствиС. ΠŸΡ€Π°Π²ΠΈΠ»Π°, ΠΊΠΎΡ‚ΠΎΡ€Ρ‹ΠΌ ΠΏΠΎΠ΄Ρ‡ΠΈΠ½ΡΡŽΡ‚ΡΡ Ρ‚Π΅Π»Π° ΠΏΡ€ΠΈ взаимодСйствии, Π½Π°Π·Ρ‹Π²Π°ΡŽΡ‚ Π·Π°ΠΊΠΎΠ½ΠΎΠΌ ΠšΡƒΠ»ΠΎΠ½Π°. Он формулируСтся Ρ‚Π°ΠΊ: сила, Π΄Π΅ΠΉΡΡ‚Π²ΡƒΡŽΡ‰Π°Ρ ΠΌΠ΅ΠΆΠ΄Ρƒ двумя заряТСнными Ρ‚Π΅Π»Π°ΠΌΠΈ, прямо ΠΏΡ€ΠΎΠΏΠΎΡ€Ρ†ΠΈΠΎΠ½Π°Π»ΡŒΠ½Π° количСству элСктричСства Π½Π° ΠΊΠ°ΠΆΠ΄ΠΎΠΌ ΠΈΠ· Ρ‚Π΅Π» ΠΈ ΠΎΠ±Ρ€Π°Ρ‚Π½ΠΎ ΠΏΡ€ΠΎΠΏΠΎΡ€Ρ†ΠΈΠΎΠ½Π°Π»ΡŒΠ½Π° ΠΊΠ²Π°Π΄Ρ€Π°Ρ‚Ρƒ расстояния ΠΌΠ΅ΠΆΠ΄Ρƒ зарядами.

ЭлСктричСски заряТСнныС Ρ‚Π΅Π»Π°, Π½Π°Ρ…ΠΎΠ΄ΡΡΡŒ Π½Π° расстоянии Π΄Ρ€ΡƒΠ³ ΠΎΡ‚ Π΄Ρ€ΡƒΠ³Π°, ΠΈΡΠΏΡ‹Ρ‚Ρ‹Π²Π°ΡŽΡ‚ дСйствиС ΠΎΠΏΡ€Π΅Π΄Π΅Π»Π΅Π½Π½ΠΎΠΉ силы. ΠŸΡ€ΠΎΡΡ‚Ρ€Π°Π½ΡΡ‚Π²ΠΎ, Π² ΠΊΠΎΡ‚ΠΎΡ€ΠΎΠΌ Π΄Π΅ΠΉΡΡ‚Π²ΡƒΡŽΡ‚ эти силы, Π½Π°Π·Ρ‹Π²Π°ΡŽΡ‚ элСктричСским силовым ΠΏΠΎΠ»Π΅ΠΌ. Π’Π½ΡƒΡ‚Ρ€ΠΈ элСктричСского поля силы Π΄Π΅ΠΉΡΡ‚Π²ΡƒΡŽΡ‚ Π² ΠΎΠΏΡ€Π΅Π΄Π΅Π»Π΅Π½Π½ΠΎΠΌ Π½Π°ΠΏΡ€Π°Π²Π»Π΅Π½ΠΈΠΈ. Π›ΠΈΠ½ΠΈΠΈ, ΠΏΠΎ ΠΊΠΎΡ‚ΠΎΡ€Ρ‹ΠΌ Π΄Π΅ΠΉΡΡ‚Π²ΡƒΡŽΡ‚ элСктричСскиС силы поля, Π½Π°Π·Ρ‹Π²Π°ΡŽΡ‚ силовыми. Π—Π° ΠΈΡ… Π½Π°ΠΏΡ€Π°Π²Π»Π΅Π½ΠΈΠ΅ Π² любой Ρ‚ΠΎΡ‡ΠΊΠ΅ поля принято Π½Π°ΠΏΡ€Π°Π²Π»Π΅Π½ΠΈΠ΅, Π² ΠΊΠΎΡ‚ΠΎΡ€ΠΎΠΌ Π±ΡƒΠ΄Π΅Ρ‚ Π΄Π²ΠΈΠ³Π°Ρ‚ΡŒΡΡ Π² этом полС ΠΏΠΎΠ»ΠΎΠΆΠΈΡ‚Π΅Π»ΡŒΠ½Ρ‹ΠΉ заряд. Π‘Π»Π΅Π΄ΠΎΠ²Π°Ρ‚Π΅Π»ΡŒΠ½ΠΎ, элСктричСскоС ΠΏΠΎΠ»Π΅ ΠΈΠ·ΠΎΠ»ΠΈΡ€ΠΎΠ²Π°Π½Π½ΠΎΠ³ΠΎ ΠΎΡ‚Ρ€ΠΈΡ†Π°Ρ‚Π΅Π»ΡŒΠ½ΠΎΠ³ΠΎ заряда Π½Π°ΠΏΡ€Π°Π²Π»Π΅Π½ΠΎ ΠΊ заряду (рис. 1), Π° Π»ΠΈΠ½ΠΈΠΈ сил, Π΄Π΅ΠΉΡΡ‚Π²ΡƒΡŽΡ‰ΠΈΡ… ΠΌΠ΅ΠΆΠ΄Ρƒ ΠΏΠΎΠ»ΠΎΠΆΠΈΡ‚Π΅Π»ΡŒΠ½Ρ‹ΠΌ ΠΈ ΠΎΡ‚Ρ€ΠΈΡ†Π°Ρ‚Π΅Π»ΡŒΠ½Ρ‹ΠΌ зарядами, Π½Π°ΠΏΡ€Π°Π²Π»Π΅Π½Ρ‹ Π² сторону ΠΎΡ‚Ρ€ΠΈΡ†Π°Ρ‚Π΅Π»ΡŒΠ½ΠΎΠ³ΠΎ заряда. Π‘ΠΈΠ»ΠΎΠ²Ρ‹Π΅ Π»ΠΈΠ½ΠΈΠΈ ΠΎΠ΄Π½ΠΎΠΈΠΌΠ΅Π½Π½Ρ‹Ρ… зарядов ΠΎΡ‚Ρ‚Π°Π»ΠΊΠΈΠ²Π°ΡŽΡ‚ΡΡ Π΄Ρ€ΡƒΠ³ ΠΎΡ‚ Π΄Ρ€ΡƒΠ³Π° (рис. 2).

Β 

Рис. 1
Рис. 2

ЭлСктричСский Ρ‚ΠΎΠΊ ΠΈ Π½Π°ΠΏΡ€Π°Π²Π»Π΅Π½ΠΈΠ΅ двиТСния элСктронов. ΠŸΡ€ΠΈ ΠΈΠ·ΡƒΡ‡Π΅Π½ΠΈΠΈ Π·Π°ΠΊΠΎΠ½ΠΎΠ² элСктричСского Ρ‚ΠΎΠΊΠ° сначала Π±Ρ‹Π»ΠΎ ΠΏΡ€Π΅Π΄ΠΏΠΎΠ»ΠΎΠΆΠ΅Π½ΠΎ, Ρ‡Ρ‚ΠΎ элСктричСский Ρ‚ΠΎΠΊ Π½Π°ΠΏΡ€Π°Π²Π»Π΅Π½ ΠΎΡ‚ ΠΏΠΎΠ»ΠΎΠΆΠΈΡ‚Π΅Π»ΡŒΠ½ΠΎ ΠΊ ΠΎΡ‚Ρ€ΠΈΡ†Π°Ρ‚Π΅Π»ΡŒΠ½ΠΎ заряТСнным Ρ‚Π΅Π»Π°ΠΌ. Π‘ ΠΏΠΎΠΌΠΎΡ‰ΡŒΡŽ Π±ΠΎΠ»Π΅Π΅ ΠΏΠΎΠ·Π΄Π½ΠΈΡ… исслСдований Π±Ρ‹Π»ΠΎ установлСно, Ρ‡Ρ‚ΠΎ элСктроны пСрСходят ΠΎΡ‚ ΠΎΡ‚Ρ€ΠΈΡ†Π°Ρ‚Π΅Π»ΡŒΠ½ΠΎ заряТСнных ΠΊ ΠΏΠΎΠ»ΠΎΠΆΠΈΡ‚Π΅Π»ΡŒΠ½ΠΎ заряТСнным ΠΈΠ»ΠΈ Π½Π΅ΠΉΡ‚Ρ€Π°Π»ΡŒΠ½Ρ‹ΠΌ Ρ‚Π΅Π»Π°ΠΌ.

Однако ΡƒΠΊΠΎΡ€Π΅Π½ΠΈΠ»ΠΎΡΡŒ ΠΏΠ΅Ρ€Π²ΠΎΠ΅ ΠΏΠΎΠ»ΠΎΠΆΠ΅Π½ΠΈΠ΅, ΠΊΠΎΡ‚ΠΎΡ€ΠΎΠ΅ Π»Π΅Π³Π»ΠΎ Π² основу всСх элСктричСских ΠΈΠ·ΠΌΠ΅Ρ€Π΅Π½ΠΈΠΉ ΠΈ Π² ΡΠ»Π΅ΠΊΡ‚Ρ€ΠΎΡ‚Π΅Ρ…Π½ΠΈΡ‡Π΅ΡΠΊΡƒΡŽ ΠΏΡ€Π°ΠΊΡ‚ΠΈΠΊΡƒ. Но, нСсмотря Π½Π° это, Π² соврСмСнных условиях дСйствуСт ΠΏΡ€Π°Π²ΠΈΠ»ΠΎ, ΠΊΠΎΡ‚ΠΎΡ€ΠΎΠ΅ опрСдСляСт элСктричСский Ρ‚ΠΎΠΊ ΠΊΠ°ΠΊ ΠΏΠΎΡ‚ΠΎΠΊ элСктронов, Π½Π°ΠΏΡ€Π°Π²Π»Π΅Π½Π½Ρ‹ΠΉ ΠΎΡ‚ минуса ΠΊ ΠΏΠ»ΡŽΡΡƒ.

ЭлСктричСский ΠΏΠΎΡ‚Π΅Π½Ρ†ΠΈΠ°Π». Π”Π΅ΠΉΡΡ‚Π²ΡƒΡŽΡ‰ΠΈΠ΅ Π½Π° Ρ‚Π΅Π»Π° силы стрСмятся привСсти ΠΈΡ… Π² Ρ‚Π°ΠΊΠΎΠ΅ ΠΏΠΎΠ»ΠΎΠΆΠ΅Π½ΠΈΠ΅, Π² ΠΊΠΎΡ‚ΠΎΡ€ΠΎΠΌ ΠΏΠΎΡ‚Π΅Π½Ρ†ΠΈΠ°Π»ΡŒΠ½Π°Ρ энСргия Ρ‚Π΅Π» Π±ΡƒΠ΄Π΅Ρ‚ наимСньшСй (Π½Π°ΠΏΡ€ΠΈΠΌΠ΅Ρ€, пролитая Π²ΠΎΠ΄Π° стСкаСт Π² самыС Π½ΠΈΠ·ΠΊΠΈΠ΅ мСста, ΠΏΠ°Ρ€ двиТСтся Π² Ρ‚Ρ€ΡƒΠ±Π΅ ΠΈΠ· Ρ‚ΠΎΡ‡ΠΊΠΈ с мСньшСй ΠΊ Ρ‚ΠΎΡ‡ΠΊΠ΅ с большСй ΠΏΠΎΡ‚Π΅Π½Ρ†ΠΈΠ°Π»ΡŒΠ½ΠΎΠΉ энСргиСй). Для сообщСния ΠΏΠΎΡ‚Π΅Π½Ρ†ΠΈΠ°Π»ΡŒΠ½ΠΎΠΉ энСргии Π²ΠΎΠ΄Π΅ Π΅Π΅ ΠΌΠΎΠΆΠ½ΠΎ ΠΏΠΎΠ΄Π½ΡΡ‚ΡŒ Π½Π° Π½Π΅ΠΊΠΎΡ‚ΠΎΡ€ΡƒΡŽ высоту. Π­Ρ‚ΠΈ полоТСния Ρ€Π°ΡΠΏΡ€ΠΎΡΡ‚Ρ€Π°Π½ΡΡŽΡ‚ΡΡ ΠΈ Π½Π° элСктричСский Ρ‚ΠΎΠΊ.

ЭлСктричСский ΠΏΠΎΡ‚Π΅Π½Ρ†ΠΈΠ°Π» ΠΌΠΎΠΆΠ½ΠΎ ΡΠΎΠ·Π΄Π°Ρ‚ΡŒ, отняв ΠΈΠ»ΠΈ Π΄ΠΎΠ±Π°Π²ΠΈΠ² ΠΊ Π½Π΅ΠΉΡ‚Ρ€Π°Π»ΡŒΠ½ΠΎΠΌΡƒ Ρ‚Π΅Π»Ρƒ элСктроны. Π’ ΠΏΠ΅Ρ€Π²ΠΎΠΌ случаС Ρ‚Π΅Π»ΠΎ ΠΏΡ€ΠΈΠΎΠ±Ρ€Π΅Ρ‚Π°Π΅Ρ‚ ΠΏΠΎΠ»ΠΎΠΆΠΈΡ‚Π΅Π»ΡŒΠ½Ρ‹ΠΉ заряд, Ρ‚. Π΅. ΠΏΠΎΡ‚Π΅Π½Ρ†ΠΈΠ°Π» Ρ‚Π΅Π»Π° возрастаСт (ΡΠΎΠ²Π΅Ρ€ΡˆΠ΅Π½Π° Ρ€Π°Π±ΠΎΡ‚Π° ΠΏΠΎ ΡƒΠ΄Π°Π»Π΅Π½ΠΈΡŽ элСктрона), Π²ΠΎ Π²Ρ‚ΠΎΡ€ΠΎΠΌ β€” ΠΎΡ‚Ρ€ΠΈΡ†Π°Ρ‚Π΅Π»ΡŒΠ½Ρ‹ΠΉ заряд ΠΈ ΠΏΠΎΡ‚Π΅Π½Ρ†ΠΈΠ°Π» Π΅Π³ΠΎ Π±ΡƒΠ΄Π΅Ρ‚ ΠΎΡ‚Ρ€ΠΈΡ†Π°Ρ‚Π΅Π»ΡŒΠ½Ρ‹ΠΌ. ЭлСктричСство ΠΏΠ΅Ρ€Π΅Ρ‚Π΅ΠΊΠ°Π΅Ρ‚ ΠΎΡ‚ Π±ΠΎΠ»Π΅Π΅ высокого ΠΊ Π±ΠΎΠ»Π΅Π΅ Π½ΠΈΠ·ΠΊΠΎΠΌΡƒ ΠΏΠΎΡ‚Π΅Π½Ρ†ΠΈΠ°Π»Ρƒ.

Π Π°Π·Ρ€ΡΠ΄ΠΈΡ‚ΡŒ Ρ‚Π΅Π»ΠΎ ΠΎΡ‚ элСктричСского заряда ΠΌΠΎΠΆΠ½ΠΎ ΠΏΡƒΡ‚Π΅ΠΌ соСдинСния Π΅Π³ΠΎ с Π·Π΅ΠΌΠ»Π΅ΠΉ, Ρ‚. Π΅. зазСмлСния Ρ‚Π΅Π»Π°. ЭлСктричСскиС заряды Ρ‚Π΅Π»Π° вслСдствиС ΠΈΡ… Π²Π·Π°ΠΈΠΌΠ½ΠΎΠ³ΠΎ отталкивания стрСмятся Ρ€Π°Π²Π½ΠΎΠΌΠ΅Ρ€Π½ΠΎ Ρ€Π°ΡΠΏΡ€Π΅Π΄Π΅Π»ΠΈΡ‚ΡŒΡΡ Π½Π° заряТСнном Ρ‚Π΅Π»Π΅ ΠΈ Π·Π΅ΠΌΠ»Π΅. Однако вслСдствиС Ρ‚ΠΎΠ³ΠΎ Ρ‡Ρ‚ΠΎ зСмля нСсравнимо большС заряТСнного Ρ‚Π΅Π»Π°, всС заряды с Π½Π΅Π³ΠΎ ΡƒΠΉΠ΄ΡƒΡ‚ Π² зСмлю ΠΈ Ρ‚Π΅Π»ΠΎ станСт Π½Π΅ΠΉΡ‚Ρ€Π°Π»ΡŒΠ½Ρ‹ΠΌ, Ρ‚. Π΅. элСктричСски бСзопасным.

ЭлСктричСская Ρ†Π΅ΠΏΡŒ постоянного Ρ‚ΠΎΠΊΠ°. ЭлСктричСский Ρ‚ΠΎΠΊ, Π·Π½Π°Ρ‡Π΅Π½ΠΈΠ΅ ΠΊΠΎΡ‚ΠΎΡ€ΠΎΠ³ΠΎ Π½Π΅ измСняСтся Π²ΠΎ Π²Ρ€Π΅ΠΌΠ΅Π½ΠΈ, Π½Π°Π·Ρ‹Π²Π°ΡŽΡ‚ постоянным. Π˜ΡΡ‚ΠΎΡ‡Π½ΠΈΠΊ элСктричСского Ρ‚ΠΎΠΊΠ° с присоСдинСнными ΠΊ Π½Π΅ΠΌΡƒ Π»ΠΈΠ½Π΅ΠΉΠ½Ρ‹ΠΌΠΈ ΠΏΡ€ΠΎΠ²ΠΎΠ΄Π°ΠΌΠΈ ΠΈ ΠΏΠΎΡ‚Ρ€Π΅Π±ΠΈΡ‚Π΅Π»Π΅ΠΌ Ρ‚ΠΎΠΊΠ° ΠΎΠ±Ρ€Π°Π·ΡƒΡŽΡ‚ Π·Π°ΠΌΠΊΠ½ΡƒΡ‚ΡƒΡŽ ΡΠ»Π΅ΠΊΡ‚Ρ€ΠΈΡ‡Π΅ΡΠΊΡƒΡŽ Ρ†Π΅ΠΏΡŒ, ΠΏΠΎ ΠΊΠΎΡ‚ΠΎΡ€ΠΎΠΉ ΠΏΡ€ΠΎΡ‚Π΅ΠΊΠ°Π΅Ρ‚ элСктричСский Ρ‚ΠΎΠΊ. ΠŸΡ€ΠΎΡΡ‚Π΅ΠΉΡˆΠ°Ρ элСктричСская Ρ†Π΅ΠΏΡŒ ΠΈΠΌΠ΅Π΅Ρ‚ источник ΠΈ ΠΏΠΎΡ‚Ρ€Π΅Π±ΠΈΡ‚Π΅Π»ΡŒ элСктричСского Ρ‚ΠΎΠΊΠ° ΠΈΒ Π΄Π²Π° ΡΠΎΠ΅Π΄ΠΈΠ½ΡΡŽΡ‰ΠΈΡ… ΠΈΡ… Π»ΠΈΠ½Π΅ΠΉΠ½Ρ‹Ρ… ΠΏΡ€ΠΎΠ²ΠΎΠ΄Π° (рис. 3). Π’ качСствС источников постоянного элСктричСского Ρ‚ΠΎΠΊΠ° ΠΏΡ€ΠΈΠΌΠ΅Π½ΡΡŽΡ‚ аккумуляторы, Π³Π΅Π½Π΅Ρ€Π°Ρ‚ΠΎΡ€Ρ‹ β€” элСктричСскиС ΠΌΠ°ΡˆΠΈΠ½Ρ‹, ΠΏΡ€ΠΈΠ²ΠΎΠ΄ΠΈΠΌΡ‹Π΅ Π² Π΄Π²ΠΈΠΆΠ΅Π½ΠΈΠ΅ мСханичСскими двигатСлями, Π³Π°Π»ΡŒΠ²Π°Π½ΠΈΡ‡Π΅ΡΠΊΠΈΠ΅ элСмСнты ΠΈ ряд Π΄Ρ€ΡƒΠ³ΠΈΡ… устройств. ΠŸΠΎΡ‚Ρ€Π΅Π±ΠΈΡ‚Π΅Π»ΡΠΌΠΈ элСктричСского Ρ‚ΠΎΠΊΠ° ΠΌΠΎΠ³ΡƒΡ‚ Π±Ρ‹Ρ‚ΡŒ ΡΠ»Π΅ΠΊΡ‚Ρ€ΠΎΠ½Π°Π³Ρ€Π΅Π²Π°Ρ‚Π΅Π»ΡŒΠ½Ρ‹Π΅ ΠΏΡ€ΠΈΠ±ΠΎΡ€Ρ‹, сварочная Π΄ΡƒΠ³Π°, ΠΎΡΠ²Π΅Ρ‚ΠΈΡ‚Π΅Π»ΡŒΠ½Ρ‹Π΅ Π»Π°ΠΌΠΏΠΎΡ‡ΠΊΠΈ ΠΈ Ρ‚.Π΄.

Рис. 3

ΠšΠΎΠ½Π΄Π΅Π½ΡΠ°Ρ‚ΠΎΡ€Ρ‹. ΠŸΡ€ΠΈ ΠΎΠ΄Π½ΠΎΠΌ ΠΈ Ρ‚ΠΎΠΌ ΠΆΠ΅ Π΄Π°Π²Π»Π΅Π½ΠΈΠΈ Π² сосудС большСго объСма ΠΌΠΎΠΆΠ½ΠΎ Π²ΠΌΠ΅ΡΡ‚ΠΈΡ‚ΡŒ большСС количСство Π³Π°Π·Π°. ΠΠ΅ΠΊΠΎΡ‚ΠΎΡ€ΡƒΡŽ аналогию ΠΌΠΎΠΆΠ½ΠΎ пронСсти ΠΈ с элСктричСским зарядом. Π§Π΅ΠΌ большС Ρ€Π°Π·ΠΌΠ΅Ρ€Ρ‹ ΠΏΡ€ΠΎΠ²ΠΎΠ΄Π½ΠΈΠΊΠ°, Ρ‚Π΅ΠΌ большС Π΅Π³ΠΎ Π²ΠΌΠ΅ΡΡ‚ΠΈΠΌΠΎΡΡ‚ΡŒ для элСктричСских зарядов, Ρ‚. Π΅. большС Π΅Π³ΠΎ элСктричСская Π΅ΠΌΠΊΠΎΡΡ‚ΡŒ.

ΠžΠ΄ΠΈΠ½ΠΎΡ‡Π½Ρ‹Π΅ ΠΏΡ€ΠΎΠ²ΠΎΠ΄Π½ΠΈΠΊΠΈ ΠΎΠ±Π»Π°Π΄Π°ΡŽΡ‚ ΠΌΠ°Π»ΠΎΠΉ Π΅ΠΌΠΊΠΎΡΡ‚ΡŒΡŽ. ΠŸΠΎΡΡ‚ΠΎΠΌΡƒ для образования запаса элСктричСских зарядов ΠΏΡ€ΠΈΠΌΠ΅Π½ΡΡŽΡ‚ кондСнсаторы. ΠšΠΎΠ½Π΄Π΅Π½ΡΠ°Ρ‚ΠΎΡ€ΠΎΠΌ Π½Π°Π·Ρ‹Π²Π°ΡŽΡ‚ устройство, ΠΊΠΎΡ‚ΠΎΡ€ΠΎΠ΅ ΠΏΡ€ΠΈ ΡΡ€Π°Π²Π½ΠΈΡ‚Π΅Π»ΡŒΠ½ΠΎ ΠΌΠ°Π»Ρ‹Ρ… Ρ€Π°Π·ΠΌΠ΅Ρ€Π°Ρ… способно Π½Π°ΠΊΠ°ΠΏΠ»ΠΈΠ²Π°Ρ‚ΡŒ большиС элСктричСскиС заряды. Π’ ΠΏΡ€ΠΎΡΡ‚Π΅ΠΉΡˆΠ΅ΠΌ Π²ΠΈΠ΄Π΅ кондСнсатор состоит ΠΈΠ· Π΄Π²ΡƒΡ… мСталличСских пластин, Ρ€Π°Π·Π΄Π΅Π»Π΅Π½Π½Ρ‹Ρ… диэлСктриком (Π²ΠΎΠ·Π΄ΡƒΡ…ΠΎΠΌ, слюдой, ΠΏΠ°Ρ€Π°Ρ„ΠΈΠ½ΠΈΡ€ΠΎΠ²Π°Π½Π½ΠΎΠΉ Π±ΡƒΠΌΠ°Π³ΠΎΠΉ ΠΈ Ρ‚.ΠΏ.). Π’ зависимости ΠΎΡ‚ Π²ΠΈΠ΄Π° диэлСктрика кондСнсатор Π½Π°Π·Ρ‹Π²Π°ΡŽΡ‚ Π²ΠΎΠ·Π΄ΡƒΡˆΠ½Ρ‹ΠΌ, Π±ΡƒΠΌΠ°ΠΆΠ½Ρ‹ΠΌ, ΡΠ»ΡŽΠ΄ΡΠ½Ρ‹ΠΌ ΠΈ Ρ‚.ΠΏ. Одна пластина кондСнсатора заряТаСтся ΠΏΠΎΠ»ΠΎΠΆΠΈΡ‚Π΅Π»ΡŒΠ½Ρ‹ΠΌΠΈ зарядами, Π° другая β€” ΠΎΡ‚Ρ€ΠΈΡ†Π°Ρ‚Π΅Π»ΡŒΠ½Ρ‹ΠΌΠΈ. БильноС Π²Π·Π°ΠΈΠΌΠ½ΠΎΠ΅ притяТСниС ΡƒΠ΄Π΅Ρ€ΠΆΠΈΠ²Π°Π΅Ρ‚ заряды, позволяя Π½Π°ΠΊΠΎΠΏΠΈΡ‚ΡŒ Π² кондСнсаторС большоС количСство зарядов.

Π•ΠΌΠΊΠΎΡΡ‚ΡŒ кондСнсатора зависит ΠΎΡ‚ ΠΏΠ»ΠΎΡ‰Π°Π΄ΠΈ Π΅Π³ΠΎ пластин. ΠšΠΎΠ½Π΄Π΅Π½ΡΠ°Ρ‚ΠΎΡ€, Ρƒ ΠΊΠΎΡ‚ΠΎΡ€ΠΎΠ³ΠΎ пластины ΠΈΠΌΠ΅ΡŽΡ‚ Π±ΠΎΠ»ΡŒΡˆΡƒΡŽ ΠΏΠ»ΠΎΡ‰Π°Π΄ΡŒ, ΠΌΠΎΠΆΠ΅Ρ‚ Π²ΠΌΠ΅ΡΡ‚ΠΈΡ‚ΡŒ большСС количСство зарядов.

Основной Π΅Π΄ΠΈΠ½ΠΈΡ†Π΅ΠΉ измСрСния элСктричСской Смкости слуТит Ρ„Π°Ρ€Π°Π΄Π° (Ρ„). На ΠΏΡ€Π°ΠΊΡ‚ΠΈΠΊΠ΅ ΠΏΡ€ΠΈΠΌΠ΅Π½ΡΡŽΡ‚ Π±ΠΎΠ»Π΅Π΅ ΠΌΠ΅Π»ΠΊΠΈΠ΅ Π΅Π΄ΠΈΠ½ΠΈΡ†Ρ‹: ΠΌΠΈΠΊΡ€ΠΎΡ„Π°Ρ€Π°Π΄Π° (1 ΠΌΠΊΡ„ = 0,000 001 Ρ„), ΠΏΠΈΠΊΠΎΡ„Π°Ρ€Π°Π΄Π° (1 ΠΏΡ„ = 0,000 001 ΠΌΠΊΡ„).

Π’ Ρ‚Π΅Ρ…Π½ΠΈΠΊΠ΅ кондСнсаторы ΠΈΡΠΏΠΎΠ»ΡŒΠ·ΡƒΡŽΡ‚ Π² Ρ€Π°Π·Π»ΠΈΡ‡Π½Ρ‹Ρ… элСктричСских ΠΈ радиосхСмах.

ЭлСктродвиТущая сила источника Ρ‚ΠΎΠΊΠ°. НапряТСниС. Если ΡΠΎΠ΅Π΄ΠΈΠ½ΠΈΡ‚ΡŒ Ρ‚Ρ€ΡƒΠ±ΠΊΠΎΠΉ Π΄Π²Π° сосуда с Ρ€Π°Π·Π»ΠΈΡ‡Π½Ρ‹ΠΌΠΈ уровнями Π²ΠΎΠ΄Ρ‹, Ρ‚ΠΎ Π²ΠΎΠ΄Π° Π±ΡƒΠ΄Π΅Ρ‚ ΠΏΠ΅Ρ€Π΅Ρ…ΠΎΠ΄ΠΈΡ‚ΡŒ Π² сосуд с мСньшим ΡƒΡ€ΠΎΠ²Π½Π΅ΠΌ. Наливая Π²ΠΎΠ΄Ρƒ Π² ΠΎΠ΄ΠΈΠ½ ΠΈΠ· сосудов, ΠΌΠΎΠΆΠ½ΠΎ Π΄ΠΎΠ±ΠΈΡ‚ΡŒΡΡ Ρ‚ΠΎΠ³ΠΎ, Ρ‡Ρ‚ΠΎΠ±Ρ‹ Π²ΠΎΠ΄Π° ΠΏΠΎ Ρ‚Ρ€ΡƒΠ±ΠΊΠ΅ Ρ‚Π΅ΠΊΠ»Π° Π½Π΅ΠΏΡ€Π΅Ρ€Ρ‹Π²Π½ΠΎ. Аналогичная ΠΊΠ°Ρ€Ρ‚ΠΈΠ½Π° Π½Π°Π±Π»ΡŽΠ΄Π°Π΅Ρ‚ΡΡ Π² элСктричСской Ρ†Π΅ΠΏΠΈ. На врСмя прохоТдСния элСктричСского Ρ‚ΠΎΠΊΠ° Π² Ρ†Π΅ΠΏΠΈ Π½Π° ΠΏΠΎΠ»ΡŽΡΠ°Ρ… источника Ρ‚ΠΎΠΊΠ° Π½Π΅ΠΎΠ±Ρ…ΠΎΠ΄ΠΈΠΌΠΎ ΠΏΠΎΠ΄Π΄Π΅Ρ€ΠΆΠΈΠ²Π°Ρ‚ΡŒ Ρ€Π°Π·Π½ΠΎΡΡ‚ΡŒ ΠΏΠΎΡ‚Π΅Π½Ρ†ΠΈΠ°Π»ΠΎΠ².

Π‘ΠΈΠ»Ρƒ, которая ΠΏΠΎΠ΄Π΄Π΅Ρ€ΠΆΠΈΠ²Π°Π΅Ρ‚ Ρ€Π°Π·Π½ΠΎΡΡ‚ΡŒ ΠΏΠΎΡ‚Π΅Π½Ρ†ΠΈΠ°Π»ΠΎΠ², обСспСчивая ΠΏΡ€ΠΎΡ…ΠΎΠΆΠ΄Π΅Π½ΠΈΠ΅ Ρ‚ΠΎΠΊΠ° ΠΏΠΎ элСктричСской Ρ†Π΅ΠΏΠΈ, Π½Π°Π·Ρ‹Π²Π°ΡŽΡ‚ элСктродвиТущСй силой ΠΈ условно ΠΎΠ±ΠΎΠ·Π½Π°Ρ‡Π°ΡŽΡ‚ э. Π΄.с. Π Π°Π·Π½ΠΎΡΡ‚ΡŒ ΠΏΠΎΡ‚Π΅Π½Ρ†ΠΈΠ°Π»ΠΎΠ², Π·Π°Ρ‚Ρ€Π°Ρ‡ΠΈΠ²Π°Π΅ΠΌΡƒΡŽ Π½Π° ΠΏΡ€ΠΎΠ²Π΅Π΄Π΅Π½ΠΈΠ΅ Ρ‚ΠΎΠΊΠ° Ρ‡Π΅Ρ€Π΅Π· ΡΠ»Π΅ΠΊΡ‚Ρ€ΠΈΡ‡Π΅ΡΠΊΡƒΡŽ Ρ†Π΅ΠΏΡŒ, Π½Π°Π·Ρ‹Π²Π°ΡŽΡ‚ напряТСниСм ΠΌΠ΅ΠΆΠ΄Ρƒ ΠΊΠΎΠ½Ρ†Π°ΠΌΠΈ элСктричСской Ρ†Π΅Π»ΠΈ.

НапряТСниС создаСтся источником Ρ‚ΠΎΠΊΠ°. ΠŸΡ€ΠΈ Ρ€Π°Π·ΠΎΠΌΠΊΠ½ΡƒΡ‚ΠΎΠΉ Ρ†Π΅ΠΏΠΈ напряТСниС сущСствуСт Π½Π° ΠΏΠΎΠ»ΡŽΡΠ°Ρ… ΠΈΠ»ΠΈ ΠΊΠ»Π΅ΠΌΠΌΠ°Ρ… источника Ρ‚ΠΎΠΊΠ°. Когда источник Ρ‚ΠΎΠΊΠ° Π²ΠΊΠ»ΡŽΡ‡Π΅Π½ Π² Ρ†Π΅ΠΏΡŒ, напряТСниС появляСтся ΠΈ Π½Π° ΠΎΡ‚Π΄Π΅Π»ΡŒΠ½Ρ‹Ρ… участках Ρ†Π΅ΠΏΠΈ, Ρ‡Ρ‚ΠΎ ΠΈ обусловливаСт Ρ‚ΠΎΠΊ Π² Ρ†Π΅ΠΏΠΈ. НСт напряТСния, Π½Π΅Ρ‚ ΠΈ Ρ‚ΠΎΠΊΠ° Π² Ρ†Π΅ΠΏΠΈ.

ЭлСктричСскоС сопротивлСниС. ΠŸΡ€ΠΈ Π²ΠΎΠ·Π½ΠΈΠΊΠ½ΠΎΠ²Π΅Π½ΠΈΠΈ Π² Ρ†Π΅ΠΏΠΈ элСктричСского Ρ‚ΠΎΠΊΠ° свободныС элСктроны ΠΏΠΎΠ΄ влияниСм элСктричСских сил поля ΠΏΠ΅Ρ€Π΅ΠΌΠ΅Ρ‰Π°ΡŽΡ‚ΡΡ вдоль ΠΏΡ€ΠΎΠ²ΠΎΠ΄Π½ΠΈΠΊΠ°. Π”Π²ΠΈΠΆΠ΅Π½ΠΈΡŽ элСктронов ΠΏΡ€Π΅ΠΏΡΡ‚ΡΡ‚Π²ΡƒΡŽΡ‚ Π°Ρ‚ΠΎΠΌΡ‹ ΠΈ ΠΌΠΎΠ»Π΅ΠΊΡƒΠ»Ρ‹ ΠΏΡ€ΠΎΠ²ΠΎΠ΄Π½ΠΈΠΊΠΎΠ², Π²ΡΡ‚Ρ€Π΅Ρ‡Π°ΡŽΡ‰ΠΈΡ…ΡΡ Π½Π° ΠΏΡƒΡ‚ΠΈ, Ρ‚. Π΅. элСктричСская Ρ†Π΅ΠΏΡŒ ΠΎΠΊΠ°Π·Ρ‹Π²Π°Π΅Ρ‚ сопротивлСниС ΠΏΡ€ΠΎΡ…ΠΎΠΆΠ΄Π΅Π½ΠΈΡŽ элСктричСского Ρ‚ΠΎΠΊΠ°. ЭлСктричСским сопротивлСниСм ΠΏΡ€ΠΎΠ²ΠΎΠ΄Π½ΠΈΠΊΠ° Π½Π°Π·Ρ‹Π²Π°ΡŽΡ‚ свойство Ρ‚Π΅Π»Π° ΠΈΠ»ΠΈ срСды ΠΏΡ€Π΅Π²Ρ€Π°Ρ‚ΠΈΡ‚ΡŒ ΡΠ»Π΅ΠΊΡ‚Ρ€ΠΈΡ‡Π΅ΡΠΊΡƒΡŽ ΡΠ½Π΅Ρ€Π³ΠΈΡŽ Π² Ρ‚Π΅ΠΏΠ»ΠΎΠ²ΡƒΡŽ ΠΏΡ€ΠΈ ΠΏΡ€ΠΎΡ…ΠΎΠΆΠ΄Π΅Π½ΠΈΠΈ ΠΏΠΎ Π½Π΅ΠΌΡƒ элСктричСского Ρ‚ΠΎΠΊΠ°.

Π Π°Π·Π»ΠΈΡ‡Π½Ρ‹Π΅ вСщСства ΠΈΠΌΠ΅ΡŽΡ‚ Ρ€Π°Π·Π½ΠΎΠ΅ количСство элСктронов ΠΈ Ρ€Π°Π·Π½ΠΎΠ΅ располоТСниС Π°Ρ‚ΠΎΠΌΠΎΠ². ΠŸΠΎΡΡ‚ΠΎΠΌΡƒ сопротивлСниС ΠΏΡ€ΠΎΠ²ΠΎΠ΄Π½ΠΈΠΊΠ° зависит ΠΎΡ‚ ΠΌΠ°Ρ‚Π΅Ρ€ΠΈΠ°Π»Π°, ΠΈΠ· ΠΊΠΎΡ‚ΠΎΡ€ΠΎΠ³ΠΎ ΠΎΠ½ ΠΈΠ·Π³ΠΎΡ‚ΠΎΠ²Π»Π΅Π½. Π₯ΠΎΡ€ΠΎΡˆΠΈΠΌΠΈ ΠΏΡ€ΠΎΠ²ΠΎΠ΄Π½ΠΈΠΊΠ°ΠΌΠΈ являСтся сСрСбро, мСдь, алюминий. Π‘ΠΎΠ»ΡŒΡˆΠΈΠΌ сопротивлСниСм ΠΎΠ±Π»Π°Π΄Π°ΡŽΡ‚ никСль, ΠΆΠ΅Π»Π΅Π·ΠΎ, ΡƒΠ³ΠΎΠ»ΡŒ. Наряду с этим сопротивлСниС зависит ΠΎΡ‚ Π΄Π»ΠΈΠ½Ρ‹ ΠΈ ΠΏΠ»ΠΎΡ‰Π°Π΄ΠΈ ΠΏΠΎΠΏΠ΅Ρ€Π΅Ρ‡Π½ΠΎΠ³ΠΎ сСчСния ΠΏΡ€ΠΎΠ²ΠΎΠ΄Π½ΠΈΠΊΠ°. Π§Π΅ΠΌ Π΄Π»ΠΈΠ½Π½Π΅Π΅ ΠΏΡ€ΠΎΠ²ΠΎΠ΄Π½ΠΈΠΊ ΠΏΡ€ΠΈ ΠΎΠ΄Π½ΠΎΠΌ ΠΈ Ρ‚ΠΎΠΌ ΠΆΠ΅ ΠΏΠΎΠΏΠ΅Ρ€Π΅Ρ‡Π½ΠΎΠΌ сСчСнии, Ρ‚Π΅ΠΌ большим ΠΎΠ±Π»Π°Π΄Π°Π΅Ρ‚ ΠΎΠ½ сопротивлСниСм, ΠΈ Π½Π°ΠΎΠ±ΠΎΡ€ΠΎΡ‚: Ρ‡Π΅ΠΌ большС сСчСниС ΠΏΡ€ΠΎΠ²ΠΎΠ΄Π½ΠΈΠΊΠ° ΠΏΡ€ΠΈ ΠΎΠ΄Π½ΠΎΠΉ ΠΈ Ρ‚ΠΎΠΉ ΠΆΠ΅ Π΄Π»ΠΈΠ½Π΅, Ρ‚Π΅ΠΌ мСньшС Π΅Π³ΠΎ сопротивлСниС.

НагрСв ΡƒΠ²Π΅Π»ΠΈΡ‡ΠΈΠ²Π°Π΅Ρ‚ сопротивлСниС Π±ΠΎΠ»ΡŒΡˆΠΈΠ½ΡΡ‚Π²Π° ΠΌΠ΅Ρ‚Π°Π»Π»ΠΎΠ² ΠΈ сплавов. Для чистых ΠΌΠ΅Ρ‚Π°Π»Π»ΠΎΠ² это ΡƒΠ²Π΅Π»ΠΈΡ‡Π΅Π½ΠΈΠ΅ составляСт ΠΎΠΊΠΎΠ»ΠΎ 4% Π½Π° ΠΊΠ°ΠΆΠ΄Ρ‹Π΅ 10Β° ΠΏΠΎΠ²Ρ‹ΡˆΠ΅Π½ΠΈΡ Ρ‚Π΅ΠΌΠΏΠ΅Ρ€Π°Ρ‚ΡƒΡ€Ρ‹. Волько Π½Π΅ΠΊΠΎΡ‚ΠΎΡ€Ρ‹Π΅ ΡΠΏΠ΅Ρ†ΠΈΠ°Π»ΡŒΠ½Ρ‹Π΅ мСталличСскиС сплавы (ΠΌΠ°Π½Π³Π°Π½ΠΈΠ½, константан ΠΈ Π΄Ρ€.) ΠΏΠΎΡ‡Ρ‚ΠΈ Π½Π΅ ΠΌΠ΅Π½ΡΡŽΡ‚ своСго сопротивлСния с ΡƒΠ²Π΅Π»ΠΈΡ‡Π΅Π½ΠΈΠ΅ΠΌ Ρ‚Π΅ΠΌΠΏΠ΅Ρ€Π°Ρ‚ΡƒΡ€Ρ‹.

РСостаты. ΠŸΡ€ΠΈΠ±ΠΎΡ€Ρ‹, ΠΏΡ€ΠΈ ΠΏΠΎΠΌΠΎΡ‰ΠΈ ΠΊΠΎΡ‚ΠΎΡ€Ρ‹Ρ…, мСняя сопротивлСниС, ΠΌΠΎΠΆΠ½ΠΎ Ρ€Π΅Π³ΡƒΠ»ΠΈΡ€ΠΎΠ²Π°Ρ‚ΡŒ силу Ρ‚ΠΎΠΊΠ° Π² Ρ†Π΅ΠΏΠΈ, Π½Π°Π·Ρ‹Π²Π°ΡŽΡ‚ рСостатами. РСостаты Π±Ρ‹Π²Π°ΡŽΡ‚ Π½Π΅ΡΠΊΠΎΠ»ΡŒΠΊΠΈΡ… Π²ΠΈΠ΄ΠΎΠ², Π½Π°ΠΏΡ€ΠΈΠΌΠ΅Ρ€: рСостат со ΡΠΊΠΎΠ»ΡŒΠ·ΡΡ‰ΠΈΠΌ ΠΊΠΎΠ½Ρ‚Π°ΠΊΡ‚ΠΎΠΌ, Ρ€Ρ‹Ρ‡Π°ΠΆΠ½Ρ‹ΠΉ рСостат, Π»Π°ΠΌΠΏΠΎΠ²Ρ‹ΠΉ ΠΈ Π΄Ρ€.

Рис. 4

РСостат со ΡΠΊΠΎΠ»ΡŒΠ·ΡΡ‰ΠΈΠΌ ΠΊΠΎΠ½Ρ‚Π°ΠΊΡ‚ΠΎΠΌ устроСн ΡΠ»Π΅Π΄ΡƒΡŽΡ‰ΠΈΠΌ ΠΎΠ±Ρ€Π°Π·ΠΎΠΌ (рис. 4). ΠŸΡ€ΠΎΠ²ΠΎΠ»ΠΎΠΊΠ° ΠΈΠ· ΠΌΠ΅Ρ‚Π°Π»Π»Π° с высоким ΡƒΠ΄Π΅Π»ΡŒΠ½Ρ‹ΠΌ сопротивлСниСм Π½Π°ΠΌΠΎΡ‚Π°Π½Π° Π½Π° Ρ†ΠΈΠ»ΠΈΠ½Π΄Ρ€, сдСланный ΠΈΠ· изолятора, ΠΊ ΠΊΠΎΠ½Ρ†Π°ΠΌ ΠΏΡ€ΠΎΠ²ΠΎΠ»ΠΎΠΊΠΈ ΠΏΡ€ΠΈΠΊΡ€Π΅ΠΏΠ»Π΅Π½Ρ‹ клСммы для Π²ΠΊΠ»ΡŽΡ‡Π΅Π½ΠΈΡ рСостата Π² Ρ†Π΅ΠΏΡŒ. Π‘Π²Π΅Ρ€Ρ…Ρƒ Ρ†ΠΈΠ»ΠΈΠ½Π΄Ρ€Π° Π½Π° мСталличСском стСрТнС ΠΏΡ€ΠΈΠΊΡ€Π΅ΠΏΠ»Π΅Π½ ΠΏΠΎΠ»Π·ΡƒΠ½, ΠΏΠ»ΠΎΡ‚Π½ΠΎ ΠΊΠ°ΡΠ°ΡŽΡ‰ΠΈΠΉΡΡ Π²ΠΈΡ‚ΠΊΠΎΠ² ΠΏΡ€ΠΎΠ²ΠΎΠ»ΠΎΠΊΠΈ. РСостат Π²ΠΊΠ»ΡŽΡ‡Π°ΡŽΡ‚ Π² Ρ†Π΅ΠΏΡŒ ΠΏΡ€ΠΈ ΠΏΠΎΠΌΠΎΡ‰ΠΈ ΠΎΠ΄Π½ΠΎΠΉ ΠΈΠ· ΠΊΠ»Π΅ΠΌΠΌ Π½Π° ΠΏΡ€ΠΎΠ²ΠΎΠ»ΠΎΠΊΠ΅ рСостата ΠΈ ΠΊΠ»Π΅ΠΌΠΌΡ‹ Π½Π° мСталличСском стСрТнС ΠΏΠΎΠ»Π·ΡƒΠ½Π°. ΠŸΠ΅Ρ€Π΅Π΄Π²ΠΈΠ³Π°Ρ ΠΏΠΎΠ»Π·ΡƒΠ½ Π² Ρ‚Ρƒ ΠΈΠ»ΠΈ Π΄Ρ€ΡƒΠ³ΡƒΡŽ сторону, ΡƒΠ²Π΅Π»ΠΈΡ‡ΠΈΠ²Π°ΡŽΡ‚ ΠΈΠ»ΠΈ ΡƒΠΌΠ΅Π½ΡŒΡˆΠ°ΡŽΡ‚ Π΄Π»ΠΈΠ½Ρƒ Π²ΠΊΠ»ΡŽΡ‡Π΅Π½Π½ΠΎΠΉ ΠΏΡ€ΠΎΠ²ΠΎΠ»ΠΎΠΊΠΈ ΠΈ Ρ‚Π΅ΠΌ самым ΠΈΠ·ΠΌΠ΅Π½ΡΡŽΡ‚ сопротивлСниС Ρ†Π΅ΠΏΠΈ.

РСостат Ρ€Ρ‹Ρ‡Π°ΠΆΠ½ΠΎΠ³ΠΎ Ρ‚ΠΈΠΏΠ°, состоит ΠΈΠ· ряда ΠΏΡ€ΠΎΠ²ΠΎΠ»ΠΎΡ‡Π½Ρ‹Ρ… спиралСй, ΡƒΠΊΡ€Π΅ΠΏΠ»Π΅Π½Π½Ρ‹Ρ… Π½Π° Ρ€Π°ΠΌΠ΅ ΠΈΠ· изолятора. На ΠΎΠ΄Π½ΠΎΠΉ сторонС Ρ€Π°ΠΌΡ‹ ΠΊΠΎΠ½Ρ†Ρ‹ спиралСй соСдинСны с рядом мСталличСских ΠΊΠΎΠ½Ρ‚Π°ΠΊΡ‚ΠΎΠ². ΠœΠ΅Ρ‚Π°Π»Π»ΠΈΡ‡Π΅ΡΠΊΠ°Ρ Ρ€ΡƒΡ‡ΠΊΠ°, Π²Ρ€Π°Ρ‰Π°ΡΡΡŒ Π²ΠΎΠΊΡ€ΡƒΠ³ оси, ΠΌΠΎΠΆΠ΅Ρ‚ ΠΏΠ»ΠΎΡ‚Π½ΠΎ ΠΏΡ€ΠΈΠΆΠΈΠΌΠ°Ρ‚ΡŒΡΡ ΠΊ Ρ‚ΠΎΠΌΡƒ ΠΈΠ»ΠΈ Π΄Ρ€ΡƒΠ³ΠΎΠΌΡƒ ΠΊΠΎΠ½Ρ‚Π°ΠΊΡ‚Ρƒ. Π’ зависимости ΠΎΡ‚ полоТСния Ρ€ΡƒΡ‡ΠΊΠΈ Π² Ρ†Π΅ΠΏΡŒ ΠΌΠΎΠΆΠ΅Ρ‚ Π²ΠΊΠ»ΡŽΡ‡Π°Ρ‚ΡŒΡΡ Ρ€Π°Π·Π»ΠΈΡ‡Π½ΠΎΠ΅ количСство спиралСй.

Π˜Π·ΠΌΠ΅Ρ€Π΅Π½ΠΈΠ΅ Ρ‚ΠΎΠΊΠ°, напряТСния ΠΈ сопротивлСния. ΠžΠΏΡ‹Ρ‚Ρ‹ ΠΏΠΎΠΊΠ°Π·Ρ‹Π²Π°ΡŽΡ‚, Ρ‡Π΅ΠΌ большСС количСство элСктричСства ΠΏΡ€ΠΎΡ‚Π΅ΠΊΠ°Π΅Ρ‚ ΠΏΠΎ ΠΏΡ€ΠΎΠ²ΠΎΠ΄Π½ΠΈΠΊΡƒ Π² ΠΎΠ΄Π½ΠΎ ΠΈ Ρ‚ΠΎ ΠΆΠ΅ врСмя, Ρ‚Π΅ΠΌ сильнСС дСйствиС Ρ‚ΠΎΠΊΠ°. ΠŸΠΎΡΡ‚ΠΎΠΌΡƒ элСктричСский Ρ‚ΠΎΠΊ опрСдСляСтся количСством элСктричСства, ΠΏΡ€ΠΎΡ‚Π΅ΠΊΠ°ΡŽΡ‰Π΅Π³ΠΎ Ρ‡Π΅Ρ€Π΅Π· ΠΏΠΎΠΏΠ΅Ρ€Π΅Ρ‡Π½ΠΎΠ΅ сСчСниС ΠΏΡ€ΠΎΠ²ΠΎΠ΄Π½ΠΈΠΊΠ° Π² Π΅Π΄ΠΈΠ½ΠΈΡ†Ρƒ Π²Ρ€Π΅ΠΌΠ΅Π½ΠΈ. ΠšΠΎΠ»ΠΈΡ‡Π΅ΡΡ‚Π²ΠΎ элСктричСства, ΠΏΡ€ΠΎΡ‚Π΅ΠΊΠ°ΡŽΡ‰Π΅Π³ΠΎ Ρ‡Π΅Ρ€Π΅Π· ΠΏΠΎΠΏΠ΅Ρ€Π΅Ρ‡Π½ΠΎΠ΅ сСчСниС ΠΏΡ€ΠΎΠ²ΠΎΠ΄Π½ΠΈΠΊΠ° Π² 1 сСк, Π½Π°Π·Ρ‹Π²Π°ΡŽΡ‚ силой элСктричСского Ρ‚ΠΎΠΊΠ°. Π—Π° Π΅Π΄ΠΈΠ½ΠΈΡ†Ρƒ силы Ρ‚ΠΎΠΊΠ° принят 1 Π°, Ρ‚. Π΅. сила Ρ‚Π°ΠΊΠΎΠ³ΠΎ Ρ‚ΠΎΠΊΠ°, ΠΏΡ€ΠΈ ΠΊΠΎΡ‚ΠΎΡ€ΠΎΠΌ Π² 1 сСк Ρ‡Π΅Ρ€Π΅Π· ΠΏΠΎΠΏΠ΅Ρ€Π΅Ρ‡Π½ΠΎΠ΅ сСчСниС ΠΏΡ€ΠΎΠ²ΠΎΠ΄Π½ΠΈΠΊΠ° ΠΏΡ€ΠΎΡ…ΠΎΠ΄ΠΈΡ‚ 1 ΠΊΡƒΠ»ΠΎΠ½ элСктричСства. АмпСр обозначаСтся Π±ΡƒΠΊΠ²ΠΎΠΉ Π°. Π•Π΄ΠΈΠ½ΠΈΡ†Π° силы Ρ‚ΠΎΠΊΠ° Π°ΠΌΠΏΠ΅Ρ€ Π½Π°Π·Π²Π°Π½Π° Ρ‚Π°ΠΊ Π² Ρ‡Π΅ΡΡ‚ΡŒ французского ΡƒΡ‡Π΅Π½ΠΎΠ³ΠΎ АмпСра.

Английский Ρ„ΠΈΠ·ΠΈΠΊ Π€Π°Ρ€Π°Π΄Π΅ΠΉ, изучая явлСниС прохоТдСния Ρ‚ΠΎΠΊΠ° Ρ‡Π΅Ρ€Π΅Π· ΠΆΠΈΠ΄ΠΊΠΈΠ΅ ΠΏΡ€ΠΎΠ²ΠΎΠ΄Π½ΠΈΠΊΠΈ, установил, Ρ‡Ρ‚ΠΎ вСсовоС количСство Π²Ρ‹Π΄Π΅Π»ΡΡŽΡ‰ΠΈΡ…ΡΡ ΠΏΡ€ΠΈ этом вСщСств Π½Π° элСктродах прямо ΠΏΡ€ΠΎΠΏΠΎΡ€Ρ†ΠΈΠΎΠ½Π°Π»ΡŒΠ½ΠΎ количСству ΠΏΡ€ΠΎΡˆΠ΅Π΄ΡˆΠ΅Π³ΠΎ Ρ‡Π΅Ρ€Π΅Π· раствор элСктричСства. На основании этого Π±Ρ‹Π»Π° установлСна Π΅Π΄ΠΈΠ½ΠΈΡ†Π° количСства элСктричСства.

Π—Π° Π΅Π΄ΠΈΠ½ΠΈΡ†Ρƒ количСства элСктричСства принято Ρ‚Π°ΠΊΠΎΠ΅ количСство элСктричСства, ΠΏΡ€ΠΈ ΠΏΡ€ΠΎΡ…ΠΎΠΆΠ΄Π΅Π½ΠΈΠΈ ΠΊΠΎΡ‚ΠΎΡ€ΠΎΠ³ΠΎ Ρ‡Π΅Ρ€Π΅Π· раствор сСрСбряной соли выдСляСтся Π½Π° элСктродС 1,118 ΠΌΠ³ сСрСбра. Π­Ρ‚Π° Π΅Π΄ΠΈΠ½ΠΈΡ†Π° называСтся ΠΊΡƒΠ»Π°Π½ΠΎΠΌ.

Π˜ΡΡ…ΠΎΠ΄Ρ ΠΈΠ· опрСдСлСния элСктричСского Ρ‚ΠΎΠΊΠ° ΠΌΠΎΠΆΠ½ΠΎ ΠΎΠΏΡ€Π΅Π΄Π΅Π»ΠΈΡ‚ΡŒ Π΅Π³ΠΎ силу ΠΏΠΎ Ρ„ΠΎΡ€ΠΌΡƒΠ»Π΅


Π³Π΄Π΅

I β€” сила Ρ‚ΠΎΠΊΠ° Π² Ρ†Π΅ΠΏΠΈ;

Q β€” количСство элСктричСства, ΠΏΡ€ΠΎΡ‚Π΅ΠΊΠ°ΡŽΡ‰Π΅Π³ΠΎ >Π² Ρ†Π΅Π½ΠΈ, Π² ΠΊΡƒΠ»ΠΎΠ½Π°Ρ…;

Π’ β€” врСмя прохоТдСния элСктричСства Π² Ρ†Π΅ΠΏΠΈ Π² сСк.

Π’ Ρ‚Π΅Ρ…Π½ΠΈΠΊΠ΅ имССтся Π΅Ρ‰Π΅ ΠΈ Ρ‚Π°ΠΊΠΎΠ΅ понятиС, ΠΊΠ°ΠΊ ΠΏΠ»ΠΎΡ‚Π½ΠΎΡΡ‚ΡŒ Ρ‚ΠΎΠΊΠ°.

ΠŸΠ»ΠΎΡ‚Π½ΠΎΡΡ‚ΡŒΡŽ Ρ‚ΠΎΠΊΠ° Π½Π°Π·Ρ‹Π²Π°ΡŽΡ‚ ΠΎΡ‚Π½ΠΎΡˆΠ΅Π½ΠΈΠ΅ Π²Π΅Π»ΠΈΡ‡ΠΈΠ½Ρ‹ Ρ‚ΠΎΠΊΠ° ΠΊ ΠΏΠ»ΠΎΡ‰Π°Π΄ΠΈ ΠΏΠΎΠΏΠ΅Ρ€Π΅Ρ‡Π½ΠΎΠ³ΠΎ сСчСния ΠΏΡ€ΠΎΠ²ΠΎΠ΄Π½ΠΈΠΊΠ°. ΠžΠ±Ρ‹Ρ‡Π½ΠΎ ΠΏΠ»ΠΎΡ‰Π°Π΄ΡŒ сСчСния ΠΏΡ€ΠΎΠ²ΠΎΠ΄Π½ΠΈΠΊΠΎΠ² приводится Π² ΠΊΠ²Π°Π΄Ρ€Π°Ρ‚Π½Ρ‹Ρ… ΠΌΠΈΠ»Π»ΠΈΠΌΠ΅Ρ‚Ρ€Π°Ρ…, поэтому ΠΏΠ»ΠΎΡ‚Π½ΠΎΡΡ‚ΡŒ Ρ‚ΠΎΠΊΠ° ΠΈΠ·ΠΌΠ΅Ρ€ΡΡŽΡ‚ Π² Π°/ΠΌΠΌ2.

Рассмотрим ΡΠ»Π΅ΠΊΡ‚Ρ€ΠΈΡ‡Π΅ΡΠΊΡƒΡŽ Ρ†Π΅ΠΏΡŒ, ΡΠΎΡΡ‚ΠΎΡΡ‰ΡƒΡŽ ΠΈΠ· источника Ρ‚ΠΎΠΊΠ°, ΠΏΡ€ΠΎΠ²ΠΎΠ΄Π½ΠΈΠΊΠΎΠ² ΠΈ элСктричСской Π»Π°ΠΌΠΏΠΎΡ‡ΠΊΠΈ, соСдинСнных ΠΏΠΎΡΠ»Π΅Π΄ΠΎΠ²Π°Ρ‚Π΅Π»ΡŒΠ½ΠΎ. Π‘ΠΈΠ»Π° Ρ‚ΠΎΠΊΠ° Π½Π° всСх участках этой Ρ†Π΅ΠΏΠΈ ΠΎΠ΄ΠΈΠ½Π°ΠΊΠΎΠ²Π°, Π° Π·Π½Π°Ρ‡ΠΈΡ‚ ΠΈ количСство элСктричСства, ΠΏΡ€ΠΎΡ‚Π΅ΠΊΠ°ΡŽΡ‰Π΅Π³ΠΎ ΠΏΠΎ ΠΏΡ€ΠΎΠ²ΠΎΠ΄Π°ΠΌ ΠΈ волоску Π»Π°ΠΌΠΏΠΎΡ‡ΠΊΠΈ Π² ΠΎΠ΄Π½ΠΎ ΠΈ Ρ‚ΠΎ ΠΆΠ΅ врСмя, ΠΎΠ΄ΠΈΠ½Π°ΠΊΠΎΠ²ΠΎΠ΅. Однако количСство энСргии, Π²Ρ‹Π΄Π΅Π»ΡΡŽΡ‰Π΅ΠΉΡΡ Π½Π° ΠΎΡ‚Π΄Π΅Π»ΡŒΠ½Ρ‹Ρ… участках Ρ†Π΅ΠΏΠΈ, Ρ€Π°Π·Π»ΠΈΡ‡Π½ΠΎ. Π’ этом лСгко ΡƒΠ±Π΅Π΄ΠΈΡ‚ΡŒΡΡ, Ссли ΠΏΡ€ΠΈΡ‚Ρ€ΠΎΠ½ΡƒΡ‚ΡŒΡΡ Ρ€ΡƒΠΊΠΎΠΉ ΠΊ ΠΏΡ€ΠΎΠ²ΠΎΠ΄Π°ΠΌ, подводящим Ρ‚ΠΎΠΊ ΠΊ Π»Π°ΠΌΠ»ΠΏΠΎΡ‡ΠΊΠ΅,β€” ΠΎΠ½ΠΈ Ρ…ΠΎΠ»ΠΎΠ΄Π½Ρ‹Π΅, Π² Ρ‚ΠΎ врСмя ΠΊΠ°ΠΊ волосок Π»Π°ΠΌΠΏΠΎΡ‡ΠΊΠΈ раскалСн. Π’Ρ‹Π΄Π΅Π»Π΅Π½ΠΈΠ΅ Ρ€Π°Π·Π»ΠΈΡ‡Π½Ρ‹Ρ… количСств энСргии Π½Π° Ρ€Π°Π·Π»ΠΈΡ‡Π½Ρ‹Ρ… участках Ρ†Π΅ΠΏΠΈ вызываСтся Ρ‚Π΅ΠΌ, Ρ‡Ρ‚ΠΎ Π½Π° этих участках Ρ†Π΅ΠΏΠΈ сущСствуСт Ρ€Π°Π·Π»ΠΈΡ‡Π½ΠΎΠ΅ напряТСниС.

НапряТСниС Π½Π° Π΄Π°Π½Π½ΠΎΠΌ участкС Ρ†Π΅ΠΏΠΈ ΠΏΠΎΠΊΠ°Π·Ρ‹Π²Π°Π΅Ρ‚, ΠΊΠ°ΠΊΠΎΠ΅ количСство энСргии Π±ΡƒΠ΄Π΅Ρ‚ Π²Ρ‹Π΄Π΅Π»ΠΈΡ‚ΡŒΡΡ Π½Π° Π΄Π°Π½Π½ΠΎΠΌ участкС ΠΏΡ€ΠΈ ΠΏΡ€ΠΎΡ…ΠΎΠΆΠ΄Π΅Π½ΠΈΠΈ ΠΏΠΎ Π½Π΅ΠΌΡƒ Π΅Π΄ΠΈΠ½ΠΈΡ†Ρ‹ количСства элСктричСства.

Π—Π° Π΅Π΄ΠΈΠ½ΠΈΡ†Ρƒ напряТСния ΠΏΡ€ΠΈΠ½ΠΈΠΌΠ°ΡŽΡ‚ Ρ‚Π°ΠΊΠΎΠ΅ напряТСниС, ΠΏΡ€ΠΈ ΠΊΠΎΡ‚ΠΎΡ€ΠΎΠΌ Π½Π° участкС Ρ†Π΅ΠΏΠΈ выдСляСтся 1 Π΄ΠΆΠΎΡƒΠ»ΡŒ энСргии (1 ΠΊΠ³β€’ΠΌ=9,8 дТоуля), Ссли ΠΏΠΎ этому участку ΠΏΡ€ΠΎΡ‚Π΅ΠΊΠ°Π΅Ρ‚ 1 ΠΊΡƒΠ»ΠΎΠ½ элСктричСства. Π•Π΄ΠΈΠ½ΠΈΡ†Ρƒ напряТСния Π½Π°Π·Ρ‹Π²Π°ΡŽΡ‚ Π²ΠΎΠ»ΡŒΡ‚ΠΎΠΌ ΠΈ сокращСнно ΠΎΠ±ΠΎΠ·Π½Π°Ρ‡Π°ΡŽΡ‚ Π±ΡƒΠΊΠ²ΠΎΠΉ Π². Π•Π΄ΠΈΠ½ΠΈΡ†Π° напряТСния Β«Π²ΠΎΠ»ΡŒΡ‚Β» Π½Π°Π·Π²Π°Π½Π° Ρ‚Π°ΠΊ Π² Ρ‡Π΅ΡΡ‚ΡŒ ΠΈΡ‚Π°Π»ΡŒΡΠ½ΡΠΊΠΎΠ³ΠΎ ΡƒΡ‡Π΅Π½ΠΎΠ³ΠΎ Π’ΠΎΠ»ΡŒΡ‚Π°.

Если Π½Π° ΠΊΠ°ΠΊΠΎΠΌ-Π»ΠΈΠ±ΠΎ участкС Ρ†Π΅ΠΏΠΈ напряТСниС Ρ€Π°Π²Π½ΠΎ 1 Π², это Π·Π½Π°Ρ‡ΠΈΡ‚, Ρ‡Ρ‚ΠΎ ΠΏΡ€ΠΈ ΠΏΡ€ΠΎΡ…ΠΎΠΆΠ΄Π΅Π½ΠΈΠΈ ΠΊΠ°ΠΆΠ΄ΠΎΠ³ΠΎ ΠΊΡƒΠ»ΠΎΠ½Π° элСктричСства ΠΏΠΎ этому участку выдСляСтся 1 Π΄ΠΆΠΎΡƒΠ»ΡŒ энСргии.

ΠŸΡ€ΠΈ ΠΈΠ·ΠΌΠ΅Ρ€Π΅Π½ΠΈΠΈ высоких напряТСний ΠΏΡ€ΠΈΠΌΠ΅Π½ΡΡŽΡ‚ Π΅Π΄ΠΈΠ½ΠΈΡ†Ρƒ, Π½Π°Π·Ρ‹Π²Π°Π΅ΠΌΡƒΡŽ ΠΊΠΈΠ»ΠΎΠ²ΠΎΠ»ΡŒΡ‚ΠΎΠΌ ΠΈ ΠΎΠ±ΠΎΠ·Π½Π°Ρ‡Π°Π΅ΠΌΡƒΡŽ сокращСнно ΠΊΠ². ΠšΠΈΠ»ΠΎΠ²ΠΎΠ»ΡŒΡ‚ Π² тысячу Ρ€Π°Π· большС Π²ΠΎΠ»ΡŒΡ‚Π°: 1 ΠΊΠ²=1000 Π². Для измСрСния Π½Π΅Π±ΠΎΠ»ΡŒΡˆΠΈΡ… напряТСний ΠΏΡ€ΠΈΠΌΠ΅Π½ΡΡŽΡ‚ ΠΌΠΈΠ»Π»ΠΈΠ²ΠΎΠ»ΡŒΡ‚ (ΠΌΠ²) β€”Π΅Π΄ΠΈΠ½ΠΈΡ†Ρƒ, Π² тысячу Ρ€Π°Π· ΠΌΠ΅Π½ΡŒΡˆΡƒΡŽ, Ρ‡Π΅ΠΌ Π²ΠΎΠ»ΡŒΡ‚: 1 ΠΌΠ² = 0,001 Π².

Π˜ΡΡ‚ΠΎΡ‡Π½ΠΈΠΊ элСктричСского Ρ‚ΠΎΠΊΠ°, Π²ΠΊΠ»ΡŽΡ‡Π΅Π½Π½Ρ‹ΠΉ Π² ΡΠ»Π΅ΠΊΡ‚Ρ€ΠΈΡ‡Π΅ΡΠΊΡƒΡŽ Ρ†Π΅Π»ΡŒ, расходуСт ΡΠ½Π΅Ρ€Π³ΠΈΡŽ Π½Π° ΠΏΡ€Π΅ΠΎΠ΄ΠΎΠ»Π΅Π½ΠΈΠ΅ сопротивлСния Ρ†Π΅ΠΏΠΈ. Π•Π΄ΠΈΠ½ΠΈΡ†Π΅ΠΉ сопротивлСния Π½Π°Π·Ρ‹Π²Π°ΡŽΡ‚ ΠΎΠΌ Π² Ρ‡Π΅ΡΡ‚ΡŒ Π½Π΅ΠΌΠ΅Ρ†ΠΊΠΎΠ³ΠΎ ΡƒΡ‡Π΅Π½ΠΎΠ³ΠΎ Ома, ΠΎΡ‚ΠΊΡ€Ρ‹Π²ΡˆΠ΅Π³ΠΎ Π·Π°ΠΊΠΎΠ½Ρ‹ элСктричСского Ρ‚ΠΎΠΊΠ°; ΠΎΠΌ β€” элСктричСскоС сопротивлСниС ΠΌΠ΅ΠΆΠ΄Ρƒ двумя Ρ‚ΠΎΡ‡ΠΊΠ°ΠΌΠΈ Π»ΠΈΠ½Π΅ΠΉΠ½ΠΎΠ³ΠΎ ΠΏΡ€ΠΎΠ²ΠΎΠ΄Π½ΠΈΠΊΠ°, Π² ΠΊΠΎΡ‚ΠΎΡ€ΠΎΠΌ Ρ€Π°Π·Π½ΠΎΡΡ‚ΡŒ ΠΏΠΎΡ‚Π΅Π½Ρ†ΠΈΠ°Π»ΠΎΠ² Π² 1 Π² ΠΏΡ€ΠΎΠΈΠ·Π²ΠΎΠ΄ΠΈΡ‚ Ρ‚ΠΎΠΊ Π² 1 Π°. ЭлСктричСскоС сопротивлСниС обозначаСтся двумя Π±ΡƒΠΊΠ²Π°ΠΌΠΈ ΠΎΠΌ.

ΠŸΡ€ΠΈ ΠΈΠ·ΠΌΠ΅Ρ€Π΅Π½ΠΈΠΈ Π±ΠΎΠ»ΡŒΡˆΠΈΡ… сопротивлСний ΠΏΠΎΠ»ΡŒΠ·ΡƒΡŽΡ‚ΡΡ Π·Π½Π°Ρ‡ΠΈΡ‚Π΅Π»ΡŒΠ½ΠΎ большими Π΅Π΄ΠΈΠ½ΠΈΡ†Π°ΠΌΠΈ, Ρ‡Π΅ΠΌ ΠΎΠΌ: ΠΊΠΈΠ»ΠΎΠΎΠΌ (ΠΊΠΎΠΌ) ΠΈ ΠΌΠ΅Π³ΠΎΠΌ (ΠΌΠ³ΠΎΠΌ). 1 ΠΊΠΎΠΌ =1000 ΠΎΠΌ, 1 ΠΌΠ³ΠΎΠΌ= 1 000 000 ΠΎΠΌ.

Бвойства ΠΏΡ€ΠΎΠ²ΠΎΠ΄Π½ΠΈΠΊΠΎΠ² Π² ΠΎΡ‚Π½ΠΎΡˆΠ΅Π½ΠΈΠΈ ΠΈΡ… элСктричСского сопротивлСния ΠΎΡ†Π΅Π½ΠΈΠ²Π°ΡŽΡ‚ ΠΏΠΎ ΡƒΠ΄Π΅Π»ΡŒΠ½ΠΎΠΌΡƒ ΡΠΎΠΏΡ€ΠΎΡ‚ΠΈΠ²Π»Π΅Π½ΠΈΡŽ. Π£Π΄Π΅Π»ΡŒΠ½Ρ‹ΠΌ сопротивлСниСм Π½Π°Π·Ρ‹Π²Π°ΡŽΡ‚ сопротивлСниС ΠΏΡ€ΠΎΠ²ΠΎΠ΄Π½ΠΈΠΊΠ° Π΄Π»ΠΈΠ½ΠΎΠΉ 1 ΠΌ с ΠΏΠΎΠΏΠ΅Ρ€Π΅Ρ‡Π½Ρ‹ΠΌ сСчСниСм Π² 1 ΠΌΠΌ2. УдСльноС сопротивлСниС измСряСтся Ρ‚ΠΎΠΆΠ΅ Π² ΠΎΠΌΠ°Ρ….

Π—Π°ΠΊΠΎΠ½ Ома. Если Π² ΡΠ»Π΅ΠΊΡ‚Ρ€ΠΈΡ‡Π΅ΡΠΊΡƒΡŽ Ρ†Π΅ΠΏΡŒ, ΡΠΎΡΡ‚ΠΎΡΡ‰ΡƒΡŽ ΠΈΠ· Π»Π°ΠΌΠΏΠΎΡ‡ΠΊΠΈ ΠΈ Π°ΠΌΠΏΠ΅Ρ€ΠΌΠ΅Ρ‚Ρ€Π°, Π²ΠΊΠ»ΡŽΡ‡ΠΈΡ‚ΡŒ ΠΎΠ΄ΠΈΠ½ большой Π³Π°Π»ΡŒΠ²Π°Π½ΠΈΡ‡Π΅ΡΠΊΠΈΠΉ элСмСнт, ΠΌΠΎΠΆΠ½ΠΎ Π·Π°ΠΌΠ΅Ρ‚ΠΈΡ‚ΡŒ, Ρ‡Ρ‚ΠΎ ΠΏΠΎ Ρ†Π΅ΠΏΠΈ ΠΈΠ΄Π΅Ρ‚ ΠΎΡ‡Π΅Π½ΡŒ слабый Ρ‚ΠΎΠΊ ΠΈ Π½ΠΈΡ‚ΡŒ Π»Π°ΠΌΠΏΠΎΡ‡ΠΊΠΈ Π½Π΅ накаливаСтся. Как Ρ‚ΠΎΠ»ΡŒΠΊΠΎ Π³Π°Π»ΡŒΠ²Π°Π½ΠΈΡ‡Π΅ΡΠΊΠΈΠΉ элСмСнт Π·Π°ΠΌΠ΅Π½ΠΈΠΌ свСТСй Π±Π°Ρ‚Π°Ρ€Π΅ΠΉΠΊΠΎΠΉ ΠΎΡ‚ ΠΊΠ°Ρ€ΠΌΠ°Π½Π½ΠΎΠ³ΠΎ фонаря, Ρ‚ΠΎΠΊ Π² Ρ†Π΅ΠΏΠΈ увСличиваСтся ΠΈ Π½ΠΈΡ‚ΡŒ Π»Π°ΠΌΠΏΠΎΡ‡ΠΊΠΈ ярко накаливаСтся. Π˜Π·ΠΌΠ΅Ρ€ΠΈΠ² напряТСниС Π½Π° ΠΊΠΎΠ½Ρ†Π°Ρ… Ρ†Π΅ΠΏΠΈ ΠΏΡ€ΠΈ Π²ΠΊΠ»ΡŽΡ‡Π΅Π½ΠΈΠΈ элСмСнта ΠΈ Π±Π°Ρ‚Π°Ρ€Π΅ΠΉΠΊΠΈ, ΡƒΠ²ΠΈΠ΄ΠΈΠΌ, Ρ‡Ρ‚ΠΎ ΠΏΡ€ΠΈ Π²ΠΊΠ»ΡŽΡ‡Π΅Π½ΠΈΠΈ Π±Π°Ρ‚Π°Ρ€Π΅ΠΉΠΊΠΈ напряТСниС Π·Π½Π°Ρ‡ΠΈΡ‚Π΅Π»ΡŒΠ½ΠΎ большС.

ΠžΡ‚ΡΡŽΠ΄Π° слСдуСт, Ρ‡Ρ‚ΠΎ сила Ρ‚ΠΎΠΊΠ° Π² ΠΏΡ€ΠΎΠ²ΠΎΠ΄Π½ΠΈΠΊΠ΅ увСличиваСтся с ΡƒΠ²Π΅Π»ΠΈΡ‡Π΅Π½ΠΈΠ΅ΠΌ напряТСния Π½Π° ΠΊΠΎΠ½Ρ†Π°Ρ… ΠΏΡ€ΠΎΠ²ΠΎΠ΄Π½ΠΈΠΊΠ°. Π’ΠΊΠ»ΡŽΡ‡ΠΈΠ² Π² Ρ†Π΅ΠΏΡŒ вмСсто ΠΎΠ΄Π½ΠΎΠΉ Π΄Π²Π΅ Π»Π°ΠΌΠΏΠΎΡ‡ΠΊΠΈ ΠΏΠΎΡΠ»Π΅Π΄ΠΎΠ²Π°Ρ‚Π΅Π»ΡŒΠ½ΠΎ, ΡƒΠ²Π΅Π»ΠΈΡ‡ΠΈΠ²Π°Π΅ΠΌ сопротивлСниС Ρ†Π΅ΠΏΠΈ Π² Π΄Π²Π° Ρ€Π°Π·Π°. Π’Π΅ΠΏΠ΅Ρ€ΡŒ ΠΌΡ‹ Π²ΠΈΠ΄ΠΈΠΌ, Ρ‡Ρ‚ΠΎ сила Ρ‚ΠΎΠΊΠ° Π² Ρ†Π΅ΠΏΠΈ ΡƒΠΌΠ΅Π½ΡŒΡˆΠΈΠ»Π°ΡΡŒ. Π˜Π·ΡƒΡ‡Π°Ρ Π·Π°Π²ΠΈΡΠΈΠΌΠΎΡΡ‚ΡŒ силы Ρ‚ΠΎΠΊΠ° ΠΎΡ‚ сопротивлСния ΠΈ напряТСния, Π½Π΅ΠΌΠ΅Ρ†ΠΊΠΈΠΉ ΡƒΡ‡Π΅Π½Ρ‹ΠΉ Ом установил, Ρ‡Ρ‚ΠΎ сила Ρ‚ΠΎΠΊΠ° Π² ΠΏΡ€ΠΎΠ²ΠΎΠ΄Π½ΠΈΠΊΠ΅ прямо ΠΏΡ€ΠΎΠΏΠΎΡ€Ρ†ΠΈΠΎΠ½Π°Π»ΡŒΠ½Π° Π½Π°ΠΏΡ€ΡΠΆΠ΅Π½ΠΈΡŽ Π½Π° ΠΊΠΎΠ½Ρ†Π°Ρ… ΠΏΡ€ΠΎΠ²ΠΎΠ΄Π½ΠΈΠΊΠ° ΠΈ ΠΎΠ±Ρ€Π°Ρ‚Π½ΠΎ ΠΏΡ€ΠΎΠΏΠΎΡ€Ρ†ΠΈΠΎΠ½Π°Π»ΡŒΠ½Π° ΡΠΎΠΏΡ€ΠΎΡ‚ΠΈΠ²Π»Π΅Π½ΠΈΡŽ ΠΏΡ€ΠΎΠ²ΠΎΠ΄Π½ΠΈΠΊΠ°. Π­Ρ‚Π° Π·Π°Π²ΠΈΡΠΈΠΌΠΎΡΡ‚ΡŒ ΠΌΠ΅ΠΆΠ΄Ρƒ силой Ρ‚ΠΎΠΊΠ°, напряТСниСм ΠΈ сопротивлСниСм носит Π½Π°Π·Π²Π°Π½ΠΈΠ΅ Π·Π°ΠΊΠΎΠ½Π° Ома, ΠΊΠΎΡ‚ΠΎΡ€Ρ‹ΠΉ являСтся ΠΎΠ΄Π½ΠΈΠΌ ΠΈΠ· основных Π·Π°ΠΊΠΎΠ½ΠΎΠ² элСктричСского Ρ‚ΠΎΠΊΠ°.

Π—Π°ΠΊΠΎΠ½ Ома выраТаСтся ΡΠ»Π΅Π΄ΡƒΡŽΡ‰Π΅ΠΉ Ρ„ΠΎΡ€ΠΌΡƒΠ»ΠΎΠΉ:

Π³Π΄Π΅ I β€” Ρ‚ΠΎΠΊ Π² Π°;

V β€” напряТСниС Π² Π²;

R β€” сопротивлСниС Π² ΠΎΠΌ.

Π—Π°ΠΊΠΎΠ½ Ома распространяСтся Π½Π΅ Ρ‚ΠΎΠ»ΡŒΠΊΠΎ Π½Π° dc. Ρ†Π΅ΠΏΡŒ, Π½ΠΎ ΠΈ Π½Π° любой Π΅Π΅ участок. Π’ΠΎΠΊ Π½Π° любом участкС элСктричСской Ρ†Π΅ΠΏΠΈ Ρ€Π°Π²Π΅Π½ Π½Π°ΠΏΡ€ΡΠΆΠ΅Π½ΠΈΡŽ Π½Π° ΠΊΠΎΠ½Ρ†Π°Ρ… этого участка, Π΄Π΅Π»Π΅Π½Π½ΠΎΠΌΡƒ Π½Π° Π΅Π³ΠΎ сопротивлСниС.

ΠŸΠΎΡΠ»Π΅Π΄ΠΎΠ²Π°Ρ‚Π΅Π»ΡŒΠ½ΠΎΠ΅ соСдинСниС Π² элСктричСской Ρ†Π΅ΠΏΠΈ. Π’ Π±ΠΎΠ»ΡŒΡˆΠΈΠ½ΡΡ‚Π²Π΅ случаСв элСктричСская Ρ†Π΅ΠΏΡŒ состоит ΠΈΠ· Π½Π΅ΡΠΊΠΎΠ»ΡŒΠΊΠΈΡ… ΠΏΠΎΡ‚Ρ€Π΅Π±ΠΈΡ‚Π΅Π»Π΅ΠΉ Ρ‚ΠΎΠΊΠ° (рис. 5). Π‘ΠΎΠ΅Π΄ΠΈΠ½Π΅Π½ΠΈΠ΅ ΠΏΠΎΡ‚Ρ€Π΅Π±ΠΈΡ‚Π΅Π»Π΅ΠΉ Ρ‚ΠΎΠΊΠ°, ΠΏΡ€ΠΈ ΠΊΠΎΡ‚ΠΎΡ€ΠΎΠΌ ΠΊΠΎΠ½Π΅Ρ† ΠΎΠ΄Π½ΠΎΠ³ΠΎ ΠΏΡ€ΠΎΠ²ΠΎΠ΄Π½ΠΈΠΊΠ° соСдинСн с Π½Π°Ρ‡Π°Π»ΠΎΠΌ Π΄Ρ€ΡƒΠ³ΠΎΠ³ΠΎ, ΠΊΠΎΠ½Π΅Ρ† Π΄Ρ€ΡƒΠ³ΠΎΠ³ΠΎ β€” с Π½Π°Ρ‡Π°Π»ΠΎΠΌ Ρ‚Ρ€Π΅Ρ‚ΡŒΠ΅Π³ΠΎ ΠΈ Ρ‚.Π΄., Π½Π°Π·Ρ‹Π²Π°ΡŽΡ‚ ΠΏΠΎΡΠ»Π΅Π΄ΠΎΠ²Π°Ρ‚Π΅Π»ΡŒΠ½Ρ‹ΠΌ.

Рис. 5

Π’Π°ΠΊ ΠΊΠ°ΠΊ сопротивлСниС прямо ΠΏΡ€ΠΎΠΏΠΎΡ€Ρ†ΠΈΠΎΠ½Π°Π»ΡŒΠ½ΠΎ Π΄Π»ΠΈΠ½Π΅ ΠΏΡ€ΠΎΠ²ΠΎΠ΄Π½ΠΈΠΊΠ°, сопротивлСниС Ρ†Π΅ΠΏΠΈ Ρ€Π°Π²Π½ΠΎ суммС сопротивлСний ΠΎΡ‚Π΄Π΅Π»ΡŒΠ½Ρ‹Ρ… ΠΏΡ€ΠΎΠ²ΠΎΠ΄Π½ΠΈΠΊΠΎΠ², ΠΏΠΎΡΠΊΠΎΠ»ΡŒΠΊΡƒ Π²ΠΊΠ»ΡŽΡ‡Π΅Π½ΠΈΠ΅ Π½Π΅ΡΠΊΠΎΠ»ΡŒΠΊΠΈΡ… ΠΏΡ€ΠΎΠ²ΠΎΠ΄Π½ΠΈΠΊΠΎΠ² ΡƒΠ²Π΅Π»ΠΈΡ‡ΠΈΠ²Π°Π΅Ρ‚ Π΄Π»ΠΈΠ½Ρƒ ΠΏΡƒΡ‚ΠΈ Ρ‚ΠΎΠΊΠ°. Π’ΠΎΠΊ Π½Π° ΠΎΡ‚Π΄Π΅Π»ΡŒΠ½Ρ‹Ρ… участках Ρ†Π΅ΠΏΠΈ Π±ΡƒΠ΄Π΅Ρ‚ ΠΎΠ΄ΠΈΠ½Π°ΠΊΠΎΠ²Ρ‹ΠΌ. ΠŸΠΎΡΡ‚ΠΎΠΌΡƒ ΠΏΠ°Π΄Π΅Π½ΠΈΠ΅ напряТСния Π½Π° ΠΊΠ°ΠΆΠ΄ΠΎΠΌ участкС Π±ΡƒΠ΄Π΅Ρ‚ ΠΏΡ€ΠΎΠΏΠΎΡ€Ρ†ΠΈΠΎΠ½Π°Π»ΡŒΠ½ΠΎ ΡΠΎΠΏΡ€ΠΎΡ‚ΠΈΠ²Π»Π΅Π½ΠΈΡŽ Π΄Π°Π½Π½ΠΎΠ³ΠΎ участка.

ΠŸΠ°Ρ€Π°Π»Π»Π΅Π»ΡŒΠ½Ρ‹ΠΌ соСдинСниСм Π² элСктричСской Ρ†Π΅ΠΏΠΈΒ Π½Π°Π·Ρ‹Π²Π°ΡŽΡ‚ Ρ‚Π°ΠΊΠΎΠ΅ соСдинСниС, ΠΊΠΎΠ³Π΄Π°Β Π½Π°Ρ‡Π°Π»Π° всСх ΠΏΡ€ΠΎΠ²ΠΎΠ΄Π½ΠΈΠΊΠΎΠ² соСдинСны Π² ΠΎΠ΄Π½ΠΎΠΉ, Π° ΠΈΡ… ΠΊΠΎΠ½Ρ†Ρ‹ β€” Π² Π΄Ρ€ΡƒΠ³ΠΎΠΉ Ρ‚ΠΎΡ‡ΠΊΠ΅ (рис. 6). ΠŸΡ€ΠΈ ΠΏΠ°Ρ€Π°Π»Π»Π΅Π»ΡŒΠ½ΠΎΠΌ соСдинСнии для прохоТдСния элСктричСского Ρ‚ΠΎΠΊΠ° имССтся нСсколько ΠΏΡƒΡ‚Π΅ΠΉ (рис. 6). Π’ΠΎΠΊ ΠΌΠ΅ΠΆΠ΄Ρƒ ΠΏΠ°Ρ€Π°Π»Π»Π΅Π»ΡŒΠ½ΠΎ соСдинСнными потрСбитСлями распрСдСляСтся ΠΎΠ±Ρ€Π°Ρ‚Π½ΠΎΒ ΠΏΡ€ΠΎΠΏΠΎΡ€Ρ†ΠΈΠΎΠ½Π°Π»ΡŒΠ½ΠΎ сопротивлСниям ΠΏΠΎΡ‚Ρ€Π΅Π±ΠΈΡ‚Π΅Π»Π΅ΠΉ. Если ΠΎΡ‚Π΄Π΅Π»ΡŒΠ½Ρ‹Π΅ ΠΏΠΎΡ‚Ρ€Π΅Π±ΠΈΡ‚Π΅Π»ΠΈ ΠΎΠ±Π»Π°Π΄Π°ΡŽΡ‚ ΠΎΠ΄ΠΈΠ½Π°ΠΊΠΎΠ²Ρ‹ΠΌ сопротивлСниСм, Ρ‚ΠΎΠΊ Ρƒ Π½ΠΈΡ… Π±ΡƒΠ΄Π΅Ρ‚ ΠΎΠ΄ΠΈΠ½Π°ΠΊΠΎΠ²Ρ‹ΠΉ. Π§Π΅ΠΌ мСньшС сопротивлСниС ΠΎΡ‚Π΄Π΅Π»ΡŒΠ½ΠΎΠ³ΠΎ потрСбитСля, Ρ‚Π΅ΠΌ больший Ρ‚ΠΎΠΊ ΠΏΡ€ΠΎΠΉΠ΄Π΅Ρ‚ Ρ‡Π΅Ρ€Π΅Π· Π½Π΅Π³ΠΎ.

Рис.6

Π‘ΡƒΠΌΠΌΠ° Ρ‚ΠΎΠΊΠΎΠ² ΠΎΡ‚Π΄Π΅Π»ΡŒΠ½Ρ‹Ρ… участков Π² ΠΏΠ°Ρ€Π°Π»Π»Π΅Π»ΡŒΠ½ΠΎΠΉ Ρ†Π΅ΠΏΠΈ Ρ€Π°Π²Π½Π° ΠΏΠΎΠ»Π½ΠΎΠΌΡƒ Ρ‚ΠΎΠΊΡƒ Π² Ρ‚ΠΎΡ‡ΠΊΠ΅ развСтвлСния Ρ†Π΅ΠΏΠΈ.

Если Π² ΠΏΠΎΡΠ»Π΅Π΄Π°Π²Π°Ρ‚Π΅Π»ΡŒΠ½ΠΎ соСдинСнной Ρ†Π΅ΠΏΠΈ присоСдинСниС Π½ΠΎΠ²Ρ‹Ρ… ΠΏΠΎΡ‚Ρ€Π΅Π±ΠΈΡ‚Π΅Π»Π΅ΠΉ элСктричСского Ρ‚ΠΎΠΊΠ° ΡƒΠ²Π΅Π»ΠΈΡ‡ΠΈΠ²Π°Π΅Ρ‚ сопротивлСниС Ρ†Π΅ΠΏΠΈ,Β ΠΏΡ€ΠΈ ΠΏΠ°Ρ€Π°Π»Π»Π΅Π»ΡŒΠ½ΠΎΠΌ соСдинСнии ΠΎΠ½ΠΎ ΡƒΠΌΠ΅Π½ΡŒΡˆΠ°Π΅Ρ‚ΡΡ: ΠΏΠΎΠ΄ΠΊΠ»ΡŽΡ‡Π΅Π½Π½ΠΎΠ΅ Π½ΠΎΠ²ΠΎΠ΅ сопротивлСниС ΡƒΠ²Π΅Π»ΠΈΡ‡ΠΈΠ²Π°Π΅Ρ‚ ΠΎΠ±Ρ‰Π΅Π΅ сСчСниС ΠΏΡ€ΠΎΠ²ΠΎΠ΄Π½ΠΈΠΊΠ°, состоящСС ΠΈΠ· суммы сСчСний ΠΏΡ€ΠΎΠ²ΠΎΠ΄Π½ΠΈΠΊΠΎΠ² всСх ΠΏΠΎΡ‚Ρ€Π΅Π±ΠΈΡ‚Π΅Π»Π΅ΠΉ. А ΠΊΠ°ΠΊ извСстно, Ρ‡Π΅ΠΌ большС сСчСниС ΠΏΡ€ΠΎΠ²ΠΎΠ΄Π½ΠΈΠΊΠ° ΠΏΡ€ΠΈ постоянной Π΅Π³ΠΎ Π΄Π»ΠΈΠ½Π΅, Ρ‚Π΅ΠΌ мСньшС сопротивлСниС.

ΠŸΡ€Π΅Π½Π΅Π±Ρ€Π΅Π³Π°Ρ сопротивлСниСм ΡΠΎΠ΅Π΄ΠΈΠ½ΠΈΡ‚Π΅Π»ΡŒΠ½Ρ‹Ρ… ΠΏΡ€ΠΎΠ²ΠΎΠ΄ΠΎΠ², ΠΌΠΎΠΆΠ½ΠΎ ΡΡ‡ΠΈΡ‚Π°Ρ‚ΡŒ, Ρ‡Ρ‚ΠΎ напряТСниС источника Ρ‚ΠΎΠΊΠ° ΠΏΡ€ΠΈΠ»ΠΎΠΆΠ΅Π½ΠΎ ΠΊ ΠΊΠ°ΠΆΠ΄ΠΎΠΌΡƒ ΠΏΠΎΡ‚Ρ€Π΅Π±ΠΈΡ‚Π΅Π»ΡŽ ΠΏΠ°Ρ€Π°Π»Π»Π΅Π»ΡŒΠ½ΠΎΠΉ Ρ†Π΅ΠΏΠΈ. ΠŸΠΎΡΡ‚ΠΎΠΌΡƒ достоинством ΠΏΠ°Ρ€Π°Π»Π»Π΅Π»ΡŒΠ½ΠΎΠ³ΠΎ соСдинСния являСтся Π½Π΅Π·Π°Π²ΠΈΡΠΈΠΌΠΎΡΡ‚ΡŒ Ρ€Π°Π±ΠΎΡ‚Ρ‹ ΠΊΠ°ΠΆΠ΄ΠΎΠ³ΠΎ потрСбитСля Ρ‚ΠΎΠΊΠ°. МоТно ΠΎΡ‚ΠΊΠ»ΡŽΡ‡ΠΈΡ‚ΡŒ любой ΠΏΠΎΡ‚Ρ€Π΅Π±ΠΈΡ‚Π΅Π»ΡŒ, Π½Π΅ прСрывая прохоТдСния Ρ‚ΠΎΠΊΠ° ΠΏΠΎ ΠΎΡΡ‚Π°Π»ΡŒΠ½Ρ‹ΠΌ. ИзмСнив сопротивлСниС ΠΎΠ΄Π½ΠΎΠ³ΠΎ ΠΈΠ· ΠΏΠΎΡ‚Ρ€Π΅Π±ΠΈΡ‚Π΅Π»Π΅ΠΉ, ΠΈΠ·ΠΌΠ΅Π½ΠΈΠΌ Π² Π΅Π³ΠΎ Ρ†Π΅ΠΏΠΈ Ρ‚ΠΎΠΊ. Π£ ΠΎΡΡ‚Π°Π»ΡŒΠ½Ρ‹Ρ… ΠΏΠΎΡ‚Ρ€Π΅Π±ΠΈΡ‚Π΅Π»Π΅ΠΉ Ρ‚ΠΎΠΊ Π½Π΅ измСнится.

Рис. 7

БмСшанноС соСдинСниС Π² элСктричСской Ρ†Π΅ΠΏΠΈ. ΠžΡ‡Π΅Π½ΡŒ часто Π² элСктричСских цСпях встрСчаСтся смСшанноС соСдинСниС. Π‘ΠΌΠ΅ΡˆΠ°Π½Π½Ρ‹ΠΌ соСдинСниСм Π½Π°Π·Ρ‹Π²Π°ΡŽΡ‚ Ρ‚Π°ΠΊΠΎΠ΅ соСдинСниС, Π² ΠΊΠΎΡ‚ΠΎΡ€ΠΎΠΌ имССтся ΠΊΠ°ΠΊ ΠΏΠΎΡΠ»Π΅Π΄ΠΎΠ²Π°Ρ‚Π΅Π»ΡŒΠ½ΠΎΠ΅, Ρ‚Π°ΠΊ ΠΈ ΠΏΠ°Ρ€Π°Π»Π»Π΅Π»ΡŒΠ½ΠΎΠ΅ соСдинСниС ΠΏΠΎΡ‚Ρ€Π΅Π±ΠΈΡ‚Π΅Π»Π΅ΠΉ элСктричСского Ρ‚ΠΎΠΊΠ° (рис. 7). Для опрСдСлСния сопротивлСния Π½Π΅ΡΠΊΠΎΠ»ΡŒΠΊΠΈΡ… ΠΏΡ€ΠΎΠ²ΠΎΠ΄Π½ΠΈΠΊΠΎΠ², соСдинСнных ΠΏΠΎ смСшанной схСмС, находят сначала сопротивлСниС ΠΏΠ°Ρ€Π°Π»Π»Π΅Π»ΡŒΠ½ΠΎ ΠΈΠ»ΠΈ ΠΏΠΎΡΠ»Π΅Π΄ΠΎΠ²Π°Ρ‚Π΅Π»ΡŒΠ½ΠΎ соСдинСнных ΠΏΡ€ΠΎΠ²ΠΎΠ΄Π½ΠΈΠΊΠΎΠ², Π° Π·Π°Ρ‚Π΅ΠΌ Π·Π°ΠΌΠ΅Π½ΡΡŽΡ‚ ΠΈΡ… ΠΎΠ΄Π½ΠΈΠΌ ΠΏΡ€ΠΎΠ²ΠΎΠ΄Π½ΠΈΠΊΠΎΠΌ с сопротивлСниСм, Ρ€Π°Π²Π½Ρ‹ΠΌ Π½Π°ΠΉΠ΄Π΅Π½Π½ΠΎΠΌΡƒ. Π’Π°ΠΊΠΈΠΌ способом ΡƒΠΏΡ€ΠΎΡ‰Π°ΡŽΡ‚ схСму, приводя Π΅Π΅ ΠΊ ΠΎΠ΄Π½ΠΎΠΌΡƒ ΠΏΡ€ΠΎΠ²ΠΎΠ΄Π½ΠΈΠΊΡƒ, сопротивлСниС ΠΊΠΎΡ‚ΠΎΡ€ΠΎΠ³ΠΎ Ρ€Π°Π²Π½ΠΎ ΠΎΠ±Ρ‰Π΅ΠΌΡƒ ΡΠΎΠΏΡ€ΠΎΡ‚ΠΈΠ²Π»Π΅Π½ΠΈΡŽ слоТной Ρ†Π΅ΠΏΠΈ.

Π Π°Π±ΠΎΡ‚Π° ΠΈ ΠΌΠΎΡ‰Π½ΠΎΡΡ‚ΡŒ элСктричСского Ρ‚ΠΎΠΊΠ°. ЭлСктричСский Ρ‚ΠΎΠΊ ΠΌΠΎΠΆΠ΅Ρ‚ ΠΏΡ€ΠΎΠΈΠ·Π²ΠΎΠ΄ΠΈΡ‚ΡŒ Ρ€Π°Π±ΠΎΡ‚Ρƒ. Π‘ΠΏΠΎΡΠΎΠ±Π½ΠΎΡΡ‚ΡŒ Ρ‚Π΅Π»Π° ΠΏΡ€ΠΎΠΈΠ·Π²ΠΎΠ΄ΠΈΡ‚ΡŒ Ρ€Π°Π±ΠΎΡ‚Ρƒ Π½Π°Π·Ρ‹Π²Π°ΡŽΡ‚ энСргиСй этого Ρ‚Π΅Π»Π°. ΠŸΠΎΡΡ€Π΅Π΄ΡΡ‚Π²ΠΎΠΌ элСктричСских ΠΌΠΎΡ‚ΠΎΡ€ΠΎΠ² Ρ‚ΠΎΠΊ ΠΏΡ€ΠΈΠ²ΠΎΠ΄ΠΈΡ‚ Π² Π΄Π²ΠΈΠΆΠ΅Π½ΠΈΠ΅ элСктропоСзда, станки. Π—Π° счСт энСргии элСктричСского Ρ‚ΠΎΠΊΠ° ΡΠΎΠ²Π΅Ρ€ΡˆΠ°Π΅Ρ‚ΡΡ мСханичСская Ρ€Π°Π±ΠΎΡ‚Π°. Если ΠΏΡ€ΠΎΠ²ΠΎΠ΄Π½ΠΈΠΊ, ΠΏΠΎ ΠΊΠΎΡ‚ΠΎΡ€ΠΎΠΌΡƒ ΠΏΡ€ΠΎΡ…ΠΎΠ΄ΠΈΡ‚ Ρ‚ΠΎΠΊ, нагрСваСтся, энСргия Ρ‚ΠΎΠΊΠ° прСвращаСтся Π² Ρ‚Π΅ΠΏΠ»ΠΎΡ‚Ρƒ. ΠŸΡ€ΠΈ Ρ€Π°Π·Π»ΠΈΡ‡Π½Ρ‹Ρ… проявлСниях Ρ‚ΠΎΠΊΠ° Π½Π°Π±Π»ΡŽΠ΄Π°Π΅Ρ‚ΡΡ ΠΏΡ€Π΅Π²Ρ€Π°Ρ‰Π΅Π½ΠΈΠ΅ элСктричСской энСргии Π² Π΄Ρ€ΡƒΠ³ΠΈΠ΅ Π²ΠΈΠ΄Ρ‹ энСргии.

Π’ Π·Π°ΠΌΠΊΠ½ΡƒΡ‚ΠΎΠΉ элСктричСской Ρ†Π΅ΠΏΠΈ ΠΏΡ€ΠΎΡ‚Π΅ΠΊΠ°Π΅Ρ‚ Ρ‚ΠΎΠΊ, ΠΊΠΎΡ‚ΠΎΡ€Ρ‹ΠΉ прСдставляСт Π΄Π²ΠΈΠΆΠ΅Π½ΠΈΠ΅ элСктричСских зарядов. Для пСрСноса зарядов Π² элСктричСской Ρ†Π΅ΠΏΠΈ источник элСктричСской энСргии Π·Π°Ρ‚Ρ€Π°Ρ‡ΠΈΠ²Π°Π΅Ρ‚ ΠΎΠΏΡ€Π΅Π΄Π΅Π»Π΅Π½Π½ΠΎΠ΅ количСство энСргии ΠΈΠ»ΠΈ ΡΠΎΠ²Π΅Ρ€ΡˆΠ°Π΅Ρ‚ Ρ€Π°Π±ΠΎΡ‚Ρƒ, Ρ€Π°Π²Π½ΡƒΡŽ ΠΏΡ€ΠΎΠΈΠ·Π²Π΅Π΄Π΅Π½ΠΈΡŽ напряТСния Ρ†Π΅ΠΏΠΈ Π½Π° пСрСнСсСнноС Ρ‡Π΅Ρ€Π΅Π· Ρ†Π΅ΠΏΡŒ количСство элСктричСства.

Если ΠΏΠΎ участку элСктричСской Ρ†Π΅ΠΏΠΈ ΠΏΡ€ΠΎΡ‚Π΅ΠΊΠ»ΠΎ Q ΠΊΡƒΠ»ΠΎΠ½ΠΎΠ² элСктричСства, Π° напряТСниС Π½Π° Π½Π΅ΠΌ Ρ€Π°Π²Π½ΠΎ V, Ρ‚ΠΎ ΡΠΎΠ²Π΅Ρ€ΡˆΠ΅Π½Π½Π°Ρ Π½Π° Π΄Π°Π½Π½ΠΎΠΌ участкС Ρ†Π΅ΠΏΠΈ Ρ€Π°Π±ΠΎΡ‚Π° А Π±ΡƒΠ΄Π΅Ρ‚ Ρ€Π°Π²Π½Π°:

А = QV дТ.

ΠŸΡ€ΠΈ Ρ‚ΠΎΠΊΠ΅ Ia Π² Ρ‚Π΅Ρ‡Π΅Π½ΠΈΠ΅ Π’ сСкунд Ρ‡Π΅Ρ€Π΅Π· сСчСниС ΠΏΡ€ΠΎΠ²ΠΎΠ΄Π½ΠΈΠΊΠ° ΠΏΡ€ΠΎΡ…ΠΎΠ΄ΠΈΡ‚ IT = QΒ ΠΊΡƒΠ»ΠΎΠ½ΠΎΠ² элСктричСства. Π‘Π»Π΅Π΄ΠΎΠ²Π°Ρ‚Π΅Π»ΡŒΠ½ΠΎ, Ρ€Π°Π±ΠΎΡ‚Π° Ρ‚ΠΎΠΊΠ° Π² IΠ° ΠΏΡ€ΠΈ напряТСнии V Π² Ρ‚Π΅Ρ‡Π΅Π½ΠΈΠ΅ Π’ сСкунд Π±ΡƒΠ΄Π΅Ρ‚ Ρ€Π°Π²Π½Π°:

A = IVT.

Π Π°Π±ΠΎΡ‚Ρƒ Ρ‚ΠΎΠΊΠ° принято ΠΎΡ†Π΅Π½ΠΈΠ²Π°Ρ‚ΡŒ ΠΏΠΎ Π΅Π³ΠΎ мощности. ΠœΠΎΡ‰Π½ΠΎΡΡ‚ΡŒ Ρ‚ΠΎΠΊΠ° числСнно Ρ€Π°Π²Π½Π° Ρ€Π°Π±ΠΎΡ‚Π΅, ΠΊΠΎΡ‚ΠΎΡ€ΡƒΡŽ ΠΏΡ€ΠΎΠΈΠ·Π²ΠΎΠ΄ΠΈΡ‚ Ρ‚ΠΎΠΊ Π² 1 сСк. Π‘Π»Π΅Π΄ΠΎΠ²Π°Ρ‚Π΅Π»ΡŒΠ½ΠΎ, ΠΌΠΎΡ‰Π½ΠΎΡΡ‚ΡŒ Ρ‚ΠΎΠΊΠ° Π±ΡƒΠ΄Π΅Ρ‚ Ρ€Π°Π²Π½Π°:

Π΄ΠΆΠΎΡƒΠ»Π΅ΠΉ Π² 1 сСк.

Π•Π΄ΠΈΠ½ΠΈΡ†Π΅ΠΉ измСрСния мощности слуТит Π²Π°Ρ‚Ρ‚ (Π²Ρ‚). Один Π²Π°Ρ‚Ρ‚ β€” ΠΌΠΎΡ‰Π½ΠΎΡΡ‚ΡŒ Ρ‚ΠΎΠΊΠ° Π² 1 Π° ΠΏΡ€ΠΈ напряТСнии Π² 1 Π². Π‘Π»Π΅Π΄ΠΎΠ²Π°Ρ‚Π΅Π»ΡŒΠ½ΠΎ, с ΡƒΠ²Π΅Π»ΠΈΡ‡Π΅Π½ΠΈΠ΅ΠΌ Ρ‚ΠΎΠΊΠ° ΠΈ напряТСния ΠΌΠΎΡ‰Π½ΠΎΡΡ‚ΡŒ увСличиваСтся. Для опрСдСлСния мощности элСктричСского Ρ‚ΠΎΠΊΠ° Π½Π΅ΠΎΠ±Ρ…ΠΎΠ΄ΠΈΠΌΠΎ напряТСниС Π² Π²ΠΎΠ»ΡŒΡ‚Π°Ρ… ΡƒΠΌΠ½ΠΎΠΆΠΈΡ‚ΡŒ Π½Π° Ρ‚ΠΎΠΊ Π² Π°ΠΌΠΏΠ΅Ρ€Π°Ρ….

Наряду с Π²Π°Ρ‚Ρ‚ΠΎΠΌ для измСрСния мощности часто ΠΏΡ€ΠΈΠΌΠ΅Π½ΡΡŽΡ‚ ΠΊΠΈΠ»ΠΎΠ²Π°Ρ‚Ρ‚ (1 ΠΊΠ²Ρ‚ =1000 Π²Ρ‚), Π³Π΅ΠΊΡ‚ΠΎΠ²Π°Ρ‚Ρ‚ (1 Π³Π²Ρ‚=100 Π²Ρ‚), ΠΌΠΈΠ»Π»ΠΈΠ²Π°Ρ‚Ρ‚ (1 ΠΌΠ²Ρ‚=0,001 Π²Ρ‚) ΠΈ ΠΌΠΈΠΊΡ€ΠΎΠ²Π°Ρ‚Ρ‚ (1 ΠΌΠΊΠ²Ρ‚= 0,000 001 Π²Ρ‚).

Π Π°Π±ΠΎΡ‚Ρƒ элСктричСского Ρ‚ΠΎΠΊΠ° ΠΌΠΎΠΆΠ½ΠΎ ΠΎΠΏΡ€Π΅Π΄Π΅Π»ΠΈΡ‚ΡŒ, Ссли Π΅Π³ΠΎ ΠΌΠΎΡ‰Π½ΠΎΡΡ‚ΡŒ ΡƒΠΌΠ½ΠΎΠΆΠΈΡ‚ΡŒ Π½Π° врСмя прохоТдСния Ρ‚ΠΎΠΊΠ°: ΠΌΠΎΡ‰Π½ΠΎΡΡ‚ΡŒ —это Ρ€Π°Π±ΠΎΡ‚Π° Π² 1 сСк. Π—Π° ΠΎΡΠ½ΠΎΠ²Π½ΡƒΡŽ Π΅Π΄ΠΈΠ½ΠΈΡ†Ρƒ Ρ€Π°Π±ΠΎΡ‚Ρ‹ принята Π²Π°Ρ‚Ρ‚-сСкунда (вт‒сСк), Ρ‚. Π΅. Ρ€Π°Π±ΠΎΡ‚Π° Ρ‚ΠΎΠΊΠ° ΠΌΠΎΡ‰Π½ΠΎΡΡ‚ΡŒΡŽ 1 Π²Ρ‚ Π² Ρ‚Π΅Ρ‡Π΅Π½ΠΈΠ΅ 1 сСк. Π‘ΠΎΠ»Π΅Π΅ ΠΊΡ€ΡƒΠΏΠ½Ρ‹ΠΌΠΈ Π΅Π΄ΠΈΠ½ΠΈΡ†Π°ΠΌΠΈ ΡΠ²Π»ΡΡŽΡ‚ΡΡ Π²Π°Ρ‚Ρ‚-час (1 Π²Ρ‚β€’Ρ‡=3600 вт‒сСк), Π³Π΅ΠΊΡ‚ΠΎΠ²Π°Ρ‚Ρ‚-час (1 Π³Π²Ρ‚β€’Ρ‡ =100 Π²Ρ‚β€’Ρ‡), ΠΊΠΈΠ»ΠΎΠ²Π°Ρ‚Ρ‚-час (1 ΠΊΠ²Ρ‚β€’Ρ‡= 1000 Π²Ρ‚β€’Ρ‡).

Π—Π°ΠΊΠΎΠ½ ЛСнца—ДТоуля. Русский Π°ΠΊΠ°Π΄Π΅ΠΌΠΈΠΊ Π›Π΅Π½Ρ† ΠΈ английский Ρ„ΠΈΠ·ΠΈΠΊ Π”ΠΆΠΎΡƒΠ»ΡŒ, нСзависимо Π΄Ρ€ΡƒΠ³ ΠΎΡ‚ Π΄Ρ€ΡƒΠ³Π°, установили, Ρ‡Ρ‚ΠΎ Π² процСссС прохоТдСния элСктричСского Ρ‚ΠΎΠΊΠ° ΠΏΠΎ ΠΏΡ€ΠΎΠ²ΠΎΠ΄Π½ΠΈΠΊΡƒ количСство Ρ‚Π΅ΠΏΠ»ΠΎΡ‚Ρ‹, выдСляСмоС ΠΏΡ€ΠΎΠ²ΠΎΠ΄Π½ΠΈΠΊΠΎΠΌ, прямо ΠΏΡ€ΠΎΠΏΠΎΡ€Ρ†ΠΈΠΎΠ½Π°Π»ΡŒΠ½ΠΎ ΠΊΠ²Π°Π΄Ρ€Π°Ρ‚Ρƒ силы Ρ‚ΠΎΠΊΠ°, ΡΠΎΠΏΡ€ΠΎΡ‚ΠΈΠ²Π»Π΅Π½ΠΈΡŽ ΠΏΡ€ΠΎΠ²ΠΎΠ΄Π½ΠΈΠΊΠ° ΠΈ Π²Ρ€Π΅ΠΌΠ΅Π½ΠΈ прохоТдСния Ρ‚ΠΎΠΊΠ°. Π­Ρ‚Ρƒ Π·Π°ΠΊΠΎΠ½ΠΎΠΌΠ΅Ρ€Π½ΠΎΡΡ‚ΡŒ Π½Π°Π·Ρ‹Π²Π°ΡŽΡ‚ Π·Π°ΠΊΠΎΠΌΠΎΠΌ Π›Π΅Π½Ρ†Π° β€” ДТоуля ΠΈ Π²Ρ‹Ρ€Π°ΠΆΠ°ΡŽΡ‚ Ρ„ΠΎΡ€ΠΌΡƒΠ»ΠΎΠΉ

Q = 0,24I2Rt,

Π΄Π΅ Q β€” количСство Ρ‚Π΅ΠΏΠ»ΠΎΡ‚Ρ‹ Π² ΠΊΠ°Π»;

0,24 β€” коэффициСнт ΠΏΡ€ΠΎΠΏΠΎΡ€Ρ†ΠΈΠΎΠ½Π°Π»ΡŒΠ½ΠΎΡΡ‚ΠΈ, ΠΎΠ±ΡƒΡΠ»ΠΎΠ²Π»ΠΈΠ²Π°ΡŽΡ‰ΠΈΠΉ, Ρ‡Ρ‚ΠΎΠ±Ρ‹ Ρ‚ΠΎΠΊ Π±Ρ‹Π» Π²Ρ‹Ρ€Π°ΠΆΠ΅Π½ Π² Π°, напряТСниС Π² Π², Π° сопротивлСниС β€” Π² ΠΎΠΌ;

I β€” Ρ‚ΠΎΠΊ Π² Π°;

R β€” сопротивлСниС ΠΏΡ€ΠΎΠ²ΠΎΠ΄Π½ΠΈΠΊΠ° Π² ΠΎΠΌ;

t β€” врСмя, Π² Ρ‚Π΅Ρ‡Π΅Π½ΠΈΠ΅ ΠΊΠΎΡ‚ΠΎΡ€ΠΎΠ³ΠΎ Ρ‚ΠΎΠΊ ΠΏΡ€ΠΎΡ‚Π΅ΠΊΠ°Π» ΠΏΠΎ ΠΏΡ€ΠΎΠ²ΠΎΠ΄Π½ΠΈΠΊΡƒ, Π² сСк.

ЭлСктричСская Π΄ΡƒΠ³Π°. Если ΡΠ±Π»ΠΈΠ·ΠΈΡ‚ΡŒ ΠΊΠΎΠ½Ρ†Ρ‹ Π΄Π²ΡƒΡ… ΠΏΡ€ΠΎΠ²ΠΎΠ΄Π½ΠΈΠΊΠΎΠ², присоСдинСнных ΠΊ источнику элСктричСского Ρ‚ΠΎΠΊΠ°, ΠΌΠ΅ΠΆΠ΄Ρƒ Π½ΠΈΠΌΠΈ образуСтся искра. РазвСдя ΠΊΠΎΠ½Ρ†Ρ‹, вмСсто искры ΠΏΠΎΠ»ΡƒΡ‡ΠΈΠΌ ΡΠ»Π΅ΠΊΡ‚Ρ€ΠΈΡ‡Π΅ΡΠΊΡƒΡŽ Π΄ΡƒΠ³Ρƒ, ΡΠΎΠ·Π΄Π°ΡŽΡ‰ΡƒΡŽ ΡΠΈΠ»ΡŒΠ½Ρ‹ΠΉ ΠΈ ΠΎΡΠ»Π΅ΠΏΠΈΡ‚Π΅Π»ΡŒΠ½Ρ‹ΠΉ свСт. Если ΠΊ ΠΊΠΎΠ½Ρ†Π°ΠΌ ΠΏΡ€ΠΎΠ²ΠΎΠ΄Π½ΠΈΠΊΠΎΠ² ΠΏΡ€ΠΈΡΠΎΠ΅Π΄ΠΈΠ½ΠΈΡ‚ΡŒ ΡƒΠ³ΠΎΠ»ΡŒΠ½Ρ‹Π΅ стСрТни, ΠΌΠ΅ΠΆΠ΄Ρƒ Π½ΠΈΠΌΠΈ Ρ‚Π°ΠΊΠΆΠ΅ Π²ΠΎΠ·Π½ΠΈΠΊΠ½Π΅Ρ‚ элСктричСская Π΄ΡƒΠ³Π°. Π’ΠΎΠ·Π½ΠΈΠΊΠ½ΠΎΠ²Π΅Π½ΠΈΠ΅ Π΄ΡƒΠ³ΠΈ ΠΎΠ±ΡŠΡΡΠ½ΡΠ΅Ρ‚ΡΡ ΡΠ»Π΅Π΄ΡƒΡŽΡ‰ΠΈΠΌ ΠΎΠ±Ρ€Π°Π·ΠΎΠΌ.

Π‘ ΠΏΠΎΠ²Ρ‹ΡˆΠ΅Π½ΠΈΠ΅ΠΌ Ρ‚Π΅ΠΌΠΏΠ΅Ρ€Π°Ρ‚ΡƒΡ€Ρ‹ ΡƒΠ³ΠΎΠ»ΡŒΠ½Ρ‹Ρ… стСрТнСй увСличиваСтся ΡΠΊΠΎΡ€ΠΎΡΡ‚ΡŒ двиТСния элСктронов, находящихся Π² ΡƒΠ³Π»Π΅. ΠŸΡ€ΠΈ сильном Π½Π°Π³Ρ€Π΅Π²Π΅ ΡΠΊΠΎΡ€ΠΎΡΡ‚ΡŒ двиТСния свободных элСктронов возрастаСт Π½Π°ΡΡ‚ΠΎΠ»ΡŒΠΊΠΎ, Ρ‡Ρ‚ΠΎ ΠΏΡ€ΠΈ Ρ€Π°Π·Π΄Π²ΠΈΠΆΠ΅Π½ΠΈΠΈ ΡƒΠ³Π»Π΅ΠΉ элСктроны ΠΈΠ· стСрТнСй Π²Ρ‹Π»Π΅Ρ‚Π°ΡŽΡ‚ Π² мСТэлСктродноС пространство. Π’ Ρ€Π΅Π·ΡƒΠ»ΡŒΡ‚Π°Ρ‚Π΅ дСйствия Π²Ρ‹Π»Π΅Ρ‚Π΅Π²ΡˆΠΈΡ… элСктронов Π½Π° Π½Π΅ΠΉΡ‚Ρ€Π°Π»ΡŒΠ½Ρ‹Π΅ Π°Ρ‚ΠΎΠΌΡ‹ ΠΈ интСнсивного излучСния свСта Π½Π°Π³Ρ€Π΅Ρ‚Ρ‹ΠΌΠΈ ΠΊΠΎΠ½Ρ†Π°ΠΌΠΈ элСктродов Π²ΠΎΠ·Π΄ΡƒΡ… ΠΌΠ΅ΠΆΠ΄Ρƒ элСктродами пСрСстаСт Π±Ρ‹Ρ‚ΡŒ элСктричСски Π½Π΅ΠΉΡ‚Ρ€Π°Π»ΡŒΠ½Ρ‹ΠΌ, Ρ‚. Π΅. ΠΌΠ΅ΠΆΠ΄Ρƒ ΠΊΠΎΠ½Ρ†Π°ΠΌΠΈ Ρ€Π°Π·Π΄Π²ΠΈΠ½ΡƒΡ‚Ρ‹Ρ… элСктродов создаСтся Π³Π°Π·ΠΎΠ²Ρ‹ΠΉ ΠΏΡ€ΠΎΠΌΠ΅ΠΆΡƒΡ‚ΠΎΠΊ,Β Ρ…ΠΎΡ€ΠΎΡˆΠΎ проводящий элСктричСский Ρ‚ΠΎΠΊ, ΠΈ Π²ΠΎΠ·Π½ΠΈΠΊΠ°Π΅Ρ‚ элСктричСский разряд.

Π‘ΠΏΠΎΡΠΎΠ±Π½ΠΎΡΡ‚ΡŒ Ρ‚ΠΎΠΊΠ° ΡΠΎΠ·Π΄Π°Π²Π°Ρ‚ΡŒ ΡΠ»Π΅ΠΊΡ‚Ρ€ΠΈΡ‡Π΅ΡΠΊΡƒΡŽ Π΄ΡƒΠ³Ρƒ с успСхом ΠΈΡΠΏΠΎΠ»ΡŒΠ·ΡƒΡŽΡ‚ ΠΏΡ€ΠΈ сваркС. Π—Π°ΠΌΠ΅Π½ΠΈΠ² ΠΎΠ΄ΠΈΠ½ ΠΈΠ· ΡƒΠ³ΠΎΠ»ΡŒΠ½Ρ‹Ρ… элСктродов свариваСмым ΠΈΠ·Π΄Π΅Π»ΠΈΠ΅ΠΌ, ΠΏΠΎΠ»ΡƒΡ‡ΠΈΠΌ ΡΠ»Π΅ΠΊΡ‚Ρ€ΠΈΡ‡Π΅ΡΠΊΡƒΡŽ Π΄ΡƒΠ³Ρƒ, Π³ΠΎΡ€ΡΡ‰ΡƒΡŽ ΠΌΠ΅ΠΆΠ΄Ρƒ этим ΠΈΠ·Π΄Π΅Π»ΠΈΠ΅ΠΌ ΠΈ Π²Ρ‚ΠΎΡ€Ρ‹ΠΌ ΡƒΠ³ΠΎΠ»ΡŒΠ½Ρ‹ΠΌ элСктродом. Однако Π² настоящСС врСмя наибольшСС ΠΏΡ€ΠΈΠΌΠ΅Π½Π΅Π½ΠΈΠ΅ ΠΏΠΎΠ»ΡƒΡ‡ΠΈΠ» способ сварки мСталличСским элСктродом. Π’ этом случаС вмСсто ΡƒΠ³ΠΎΠ»ΡŒΠ½ΠΎΠ³ΠΎ элСктрода ΠΏΡ€ΠΈΠΌΠ΅Π½ΡΡŽΡ‚ мСталличСский. Бварочная Π΄ΡƒΠ³Π° Π³ΠΎΡ€ΠΈΡ‚ ΠΌΠ΅ΠΆΠ΄Ρƒ свариваСмым ΠΈΠ·Π΄Π΅Π»ΠΈΠ΅ΠΌ ΠΈ мСталличСским элСктродом. ПослС расплавлСния мСталличСского элСктрода ΠΎΠ½ замСняСтся Π½ΠΎΠ²Ρ‹ΠΌ.

ΠšΠΎΡ€ΠΎΡ‚ΠΊΠΎΠ΅ Π·Π°ΠΌΡ‹ΠΊΠ°Π½ΠΈΠ΅. Аварийный Ρ€Π΅ΠΆΠΈΠΌ Ρ€Π°Π±ΠΎΡ‚Ρ‹ элСктричСской Ρ†Π΅ΠΏΠΈ, ΠΊΠΎΠ³Π΄Π° вслСдствиС ΡƒΠΌΠ΅Π½ΡŒΡˆΠ΅Π½ΠΈΡ Π΅Π΅ сопротивлСния Ρ‚ΠΎΠΊ Π² Π½Π΅ΠΉ Ρ€Π΅Π·ΠΊΠΎ увСличиваСтся ΠΏΡ€ΠΎΡ‚ΠΈΠ² Π½ΠΎΡ€ΠΌΠ°Π»ΡŒΠ½ΠΎΠ³ΠΎ, Π½Π°Π·Ρ‹Π²Π°ΡŽΡ‚ ΠΊΠΎΡ€ΠΎΡ‚ΠΊΠΈΠΌ Π·Π°ΠΌΡ‹ΠΊΠ°Π½ΠΈΠ΅ΠΌ. ΠšΠΎΡ€ΠΎΡ‚ΠΊΠΎΠ΅ Π·Π°ΠΌΡ‹ΠΊΠ°Π½ΠΈΠ΅ получаСтся, Ссли Π² ΡΠ»Π΅ΠΊΡ‚Ρ€ΠΈΡ‡Π΅ΡΠΊΡƒΡŽ Ρ†Π΅ΠΏΡŒ Π²ΠΊΠ»ΡŽΡ‡Π°Π΅Ρ‚ΡΡ ΠΏΡ€ΠΎΠ²ΠΎΠ΄Π½ΠΈΠΊ ΠΈΠ»ΠΈ ΠΏΡ€ΠΈΠ±ΠΎΡ€ ΠΈ Ρ‚.ΠΏ. с ΠΎΡ‡Π΅Π½ΡŒ нСбольшим сопротивлСниСм ΠΏΠΎ ΡΡ€Π°Π²Π½Π΅Π½ΠΈΡŽ с сопротивлСниСм Ρ†Π΅ΠΏΠΈ. ВслСдствиС нСбольшого сопротивлСния ΠΏΠΎ Ρ†Π΅ΠΏΠΈ ΠΏΠΎΠΉΠ΄Π΅Ρ‚ Ρ‚ΠΎΠΊ, Π½Π°ΠΌΠ½ΠΎΠ³ΠΎ ΠΏΡ€Π΅Π²Ρ‹ΡˆΠ°ΡŽΡ‰ΠΈΠΉ Ρ‚ΠΎΡ‚, Π½Π° ΠΊΠΎΡ‚ΠΎΡ€Ρ‹ΠΉ рассчитана Ρ†Π΅ΠΏΡŒ. Π’Π°ΠΊΠΎΠΉ Ρ‚ΠΎΠΊ Π²Ρ‹Π·ΠΎΠ²Π΅Ρ‚ Π²Ρ‹Π΄Π΅Π»Π΅Π½ΠΈΠ΅ большого количСства Ρ‚Π΅ΠΏΠ»Π°, Ρ‡Ρ‚ΠΎ ΠΏΡ€ΠΈΠ²Π΅Π΄Π΅Ρ‚ ΠΊ ΠΎΠ±ΡƒΠ³Π»ΠΈΠ²Π°Π½ΠΈΡŽ ΠΈ ΡΠ³ΠΎΡ€Π°Π½ΠΈΡŽ изоляции ΠΏΡ€ΠΎΠ²ΠΎΠ΄ΠΎΠ², Ρ€Π°ΡΠΏΠ»Π°Π²Π»Π΅Π½ΠΈΡŽ ΠΌΠ°Ρ‚Π΅Ρ€ΠΈΠ°Π»Π° ΠΏΡ€ΠΎΠ²ΠΎΠ΄ΠΎΠ², ΠΏΠΎΡ€Ρ‡Π΅ ΡΠ»Π΅ΠΊΡ‚Ρ€ΠΎΠΈΠ·ΠΌΠ΅Ρ€ΠΈΡ‚Π΅Π»ΡŒΠ½Ρ‹Ρ… ΠΏΡ€ΠΈΠ±ΠΎΡ€ΠΎΠ², оплавлСнию ΠΊΠΎΠ½Ρ‚Π°ΠΊΡ‚ΠΎΠ² Π²Ρ‹ΠΊΠ»ΡŽΡ‡Π°Ρ‚Π΅Π»Π΅ΠΉ, Π½ΠΎΠΆΠ΅ΠΉ Ρ€ΡƒΠ±ΠΈΠ»ΡŒΠ½ΠΈΠΊΠΎΠ² ΠΈ Ρ‚.ΠΏ. ΠœΠΎΠΆΠ΅Ρ‚ Π±Ρ‹Ρ‚ΡŒ ΠΏΠΎΠ²Ρ€Π΅ΠΆΠ΄Π΅Π½ Π΄Π°ΠΆΠ΅ источник элСктричСского Ρ‚ΠΎΠΊΠ°. ΠŸΠΎΡΡ‚ΠΎΠΌΡƒ (Π²Π²ΠΈΠ΄Ρƒ опасных Ρ€Π°Π·Ρ€ΡƒΡˆΠΈΡ‚Π΅Π»ΡŒΠ½Ρ‹Ρ… послСдствий ΠΊΠΎΡ€ΠΎΡ‚ΠΊΠΎΠ³ΠΎ замыкания Π½Π΅ΠΎΠ±Ρ…ΠΎΠ΄ΠΈΠΌΠΎ ΡΠΎΠ±Π»ΡŽΠ΄Π°Ρ‚ΡŒ ΠΎΠΏΡ€Π΅Π΄Π΅Π»Π΅Π½Π½Ρ‹Π΅ условия ΠΏΡ€ΠΈ ΠΌΠΎΠ½Ρ‚Π°ΠΆΠ΅ ΠΈ эксплуатации элСктричСских установок.

ПлавкиС ΠΏΡ€Π΅Π΄ΠΎΡ…Ρ€Π°Π½ΠΈΡ‚Π΅Π»ΠΈ. Для Ρ‚ΠΎΠ³ΠΎ Ρ‡Ρ‚ΠΎΠ±Ρ‹ ΠΈΠ·Π±Π΅ΠΆΠ°Ρ‚ΡŒ Π²Π½Π΅Π·Π°ΠΏΠ½ΠΎΠ³ΠΎ ΠΈ опасного увСличСния Ρ‚ΠΎΠΊΠ° Π² элСктричСской Ρ†Π΅ΠΏΠΈ ΠΏΡ€ΠΈ ΠΊΠΎΡ€ΠΎΡ‚ΠΊΠΎΠΌ Π·Π°ΠΌΡ‹ΠΊΠ°Π½ΠΈΠΈ, Ρ†Π΅ΠΏΡŒ Π·Π°Ρ‰ΠΈΡ‰Π°ΡŽΡ‚ ΠΏΠ»Π°Π²ΠΊΠΈΠΌΠΈ прСдохранитСлями. ΠŸΡ€Π΅Π΄ΠΎΡ…Ρ€Π°Π½ΠΈΡ‚Π΅Π»ΡŒ прСдставляСт собой Π»Π΅Π³ΠΊΠΎΠΏΠ»Π°Π²ΠΊΡƒΡŽ ΠΏΡ€ΠΎΠ²ΠΎΠ»ΠΎΠΊΡƒ, Π²ΠΊΠ»ΡŽΡ‡Π΅Π½Π½ΡƒΡŽ Π² Ρ†Π΅ΠΏΡŒ ΠΏΠΎΡΠ»Π΅Π΄ΠΎΠ²Π°Ρ‚Π΅Π»ΡŒΠ½ΠΎ. ΠŸΡ€ΠΈ ΡƒΠ²Π΅Π»ΠΈΡ‡Π΅Π½ΠΈΠΈ Ρ‚ΠΎΠΊΠ° свСрх ΠΎΠΏΡ€Π΅Π΄Π΅Π»Π΅Π½Π½ΠΎΠΉ Π²Π΅Π»ΠΈΡ‡ΠΈΠ½Ρ‹ ΠΏΡ€ΠΎΠ²ΠΎΠ»ΠΎΡ‡ΠΊΠ° прСдохранитСля нагрСваСтся ΠΈ плавится, элСктричСская Ρ†Π΅ΠΏΡŒ автоматичСски разрываСтся ΠΈ Ρ‚ΠΎΠΊ Π² Π½Π΅ΠΉ прСкращаСтся. ПлавкиС вставки для Ρ€Π°Π·Π½Ρ‹Ρ… сСчСний Π·Π°Ρ‰ΠΈΡ‰Π°Π΅ΠΌΡ‹Ρ… ΠΏΡ€ΠΎΠ²ΠΎΠ΄ΠΎΠ² ΠΈ для Ρ€Π°Π·Π½Ρ‹Ρ… ΠΏΠΎΡ‚Ρ€Π΅Π±ΠΈΡ‚Π΅Π»Π΅ΠΉ энСргии бСрутся Ρ€Π°Π·Π»ΠΈΡ‡Π½Ρ‹Π΅. ПлавкиС ΠΏΡ€Π΅Π΄ΠΎΡ…Ρ€Π°Π½ΠΈΡ‚Π΅Π»ΠΈ ΠΌΠΎΠ³ΡƒΡ‚ Π²Ρ‹ΠΏΠΎΠ»Π½ΠΈΡ‚ΡŒ свою Π·Π°Π΄Π°Ρ‡Ρƒ ΠΏΡ€ΠΈ условии, Ρ‡Ρ‚ΠΎ ΠΎΠ½ΠΈ ΠΏΡ€Π°Π²ΠΈΠ»ΡŒΠ½ΠΎ Π²Ρ‹Π±Ρ€Π°Π½Ρ‹.

Рис. 8

По своСй конструкции ΠΏΡ€Π΅Π΄ΠΎΡ…Ρ€Π°Π½ΠΈΡ‚Π΅Π»ΠΈ дСлят Π½Π° ΠΏΡ€ΠΎΠ±ΠΎΡ‡Π½Ρ‹Π΅ (рис. 8,Π°), пластинчатыС (рис. 8,Π±) ΠΈ Ρ‚Ρ€ΡƒΠ±Ρ‡Π°Ρ‚Ρ‹Π΅ (рис. 8,Π²), Π’ ΠΏΡ€ΠΎΠ±ΠΎΡ‡Π½Ρ‹Ρ… прСдохранитСлях плавкая ΠΏΡ€ΠΎΠ²ΠΎΠ»ΠΎΠΊΠ° помСщаСтся Π²Π½ΡƒΡ‚Ρ€ΠΈ Ρ„Π°Ρ€Ρ„ΠΎΡ€ΠΎΠ²ΠΎΠΉ ΠΏΡ€ΠΎΠ±ΠΊΠΈ ΠΈ укрСпляСтся Π² Π΅Π΅ основании, ΠΊ ΠΊΠΎΡ‚ΠΎΡ€ΠΎΠΌΡƒ ΠΏΠΎΠ΄Π²Π΅Π΄Π΅Π½Ρ‹ ΠΏΡ€ΠΎΠ²ΠΎΠ΄Π° Ρ€Π°Π·ΠΌΡ‹ΠΊΠ°Π΅ΠΌΠΎΠΉ Ρ†Π΅ΠΏΠΈ. Π’ пластинчатых прСдохранитСлях плавкая вставка с ΠΏΠΎΠΌΠΎΡ‰ΡŒΡŽ Π½Π°ΠΊΠΎΠ½Π΅Ρ‡Π½ΠΈΠΊΠΎΠ² ΠΈ Π²ΠΈΠ½Ρ‚ΠΎΠ² ΡƒΠΊΡ€Π΅ΠΏΠ»Π΅Π½Π° Π½Π° ΠΈΠ·ΠΎΠ»ΠΈΡ€ΡƒΡŽΡ‰Π΅ΠΌ основании. ΠŸΡ€ΠΎΠ²ΠΎΠ΄Π° Ρ€Π°Π·ΠΌΡ‹ΠΊΠ°Π΅ΠΌΠΎΠΉ Ρ†Π΅ΠΏΠΈ подводят ΠΊ Π²ΠΈΠ½Ρ‚Π°ΠΌ. Π’ Ρ‚Ρ€ΡƒΠ±Ρ‡Π°Ρ‚Ρ‹Ρ… прСдохранитСлях плавкая Ρ‡Π°ΡΡ‚ΡŒ ΠΏΠΎΠΌΠ΅Ρ‰Π΅Π½Π° Π²Π½ΡƒΡ‚Ρ€ΠΈ Π»Π΅Π³ΠΊΠΎ-ΡΡŠΠ΅ΠΌΠ½Ρ‹Ρ… Ρ„Π°Ρ€Ρ„ΠΎΡ€ΠΎΠ²Ρ‹Ρ… Ρ‚Ρ€ΡƒΠ±ΠΎΠΊ.

Π’ цСпях с большим Ρ‚ΠΎΠΊΠΎΠΌ ΠΈ напряТСниСм ΠΏΠ»Π°Π²ΠΊΠΈΠ΅ ΠΏΡ€Π΅Π΄ΠΎΡ…Ρ€Π°Π½ΠΈΡ‚Π΅Π»ΠΈ ΠΏΡ€ΠΈΠΌΠ΅Π½ΡΡŽΡ‚ Ρ€Π΅Π΄ΠΊΠΎ. Π’ этих случаях ΡƒΡΡ‚Ρ€Π°ΠΈΠ²Π°ΡŽΡ‚ Π΄Ρ€ΡƒΠ³ΡƒΡŽ Π°Π²Ρ‚ΠΎΠΌΠ°Ρ‚ΠΈΡ‡Π΅ΡΠΊΡƒΡŽ Π·Π°Ρ‰ΠΈΡ‚Ρƒ.

ЭлСктричСский Ρ‚ΠΎΠΊ — Π€ΠΈΠ·ΠΈΠΊΠ° — ВСория, тСсты, Ρ„ΠΎΡ€ΠΌΡƒΠ»Ρ‹ ΠΈ Π·Π°Π΄Π°Ρ‡ΠΈ

ОглавлСниС:

Β 

ΠžΡΠ½ΠΎΠ²Π½Ρ‹Π΅ тСорСтичСскиС свСдСния

ЭлСктричСский Ρ‚ΠΎΠΊ. Π‘ΠΈΠ»Π° Ρ‚ΠΎΠΊΠ°. Π‘ΠΎΠΏΡ€ΠΎΡ‚ΠΈΠ²Π»Π΅Π½ΠΈΠ΅

К оглавлСнию…

Π’ ΠΏΡ€ΠΎΠ²ΠΎΠ΄Π½ΠΈΠΊΠ°Ρ… ΠΏΡ€ΠΈ ΠΎΠΏΡ€Π΅Π΄Π΅Π»Π΅Π½Π½Ρ‹Ρ… условиях ΠΌΠΎΠΆΠ΅Ρ‚ Π²ΠΎΠ·Π½ΠΈΠΊΠ½ΡƒΡ‚ΡŒ Π½Π΅ΠΏΡ€Π΅Ρ€Ρ‹Π²Π½ΠΎΠ΅ упорядочСнноС Π΄Π²ΠΈΠΆΠ΅Π½ΠΈΠ΅ свободных носитСлСй элСктричСского заряда. Π’Π°ΠΊΠΎΠ΅ Π΄Π²ΠΈΠΆΠ΅Π½ΠΈΠ΅ называСтся элСктричСским Ρ‚ΠΎΠΊΠΎΠΌ. Π—Π° Π½Π°ΠΏΡ€Π°Π²Π»Π΅Π½ΠΈΠ΅ элСктричСского Ρ‚ΠΎΠΊΠ° принято Π½Π°ΠΏΡ€Π°Π²Π»Π΅Π½ΠΈΠ΅ двиТСния ΠΏΠΎΠ»ΠΎΠΆΠΈΡ‚Π΅Π»ΡŒΠ½Ρ‹Ρ… свободных зарядов, хотя Π² Π±ΠΎΠ»ΡŒΡˆΠΈΠ½ΡΡ‚Π²Π΅ случаС двиТутся элСктроны – ΠΎΡ‚Ρ€ΠΈΡ†Π°Ρ‚Π΅Π»ΡŒΠ½ΠΎ заряТСнныС частицы.

ΠšΠΎΠ»ΠΈΡ‡Π΅ΡΡ‚Π²Π΅Π½Π½ΠΎΠΉ ΠΌΠ΅Ρ€ΠΎΠΉ элСктричСского Ρ‚ΠΎΠΊΠ° слуТит сила Ρ‚ΠΎΠΊΠ° I – скалярная физичСская Π²Π΅Π»ΠΈΡ‡ΠΈΠ½Π°, равная ΠΎΡ‚Π½ΠΎΡˆΠ΅Π½ΠΈΡŽ заряда q, пСрСносимого Ρ‡Π΅Ρ€Π΅Π· ΠΏΠΎΠΏΠ΅Ρ€Π΅Ρ‡Π½ΠΎΠ΅ сСчСниС ΠΏΡ€ΠΎΠ²ΠΎΠ΄Π½ΠΈΠΊΠ° Π·Π° ΠΈΠ½Ρ‚Π΅Ρ€Π²Π°Π» Π²Ρ€Π΅ΠΌΠ΅Π½ΠΈ t, ΠΊ этому ΠΈΠ½Ρ‚Π΅Ρ€Π²Π°Π»Ρƒ Π²Ρ€Π΅ΠΌΠ΅Π½ΠΈ:

Π€ΠΎΡ€ΠΌΡƒΠ»Π° Π‘ΠΈΠ»Π° Ρ‚ΠΎΠΊΠ°

Если Ρ‚ΠΎΠΊ Π½Π΅ постоянный, Ρ‚ΠΎ для нахоТдСния количСства ΠΏΡ€ΠΎΡˆΠ΅Π΄ΡˆΠ΅Π³ΠΎ Ρ‡Π΅Ρ€Π΅Π· ΠΏΡ€ΠΎΠ²ΠΎΠ΄Π½ΠΈΠΊ заряда Ρ€Π°ΡΡΡ‡ΠΈΡ‚Ρ‹Π²Π°ΡŽΡ‚ ΠΏΠ»ΠΎΡ‰Π°Π΄ΡŒ Ρ„ΠΈΠ³ΡƒΡ€Ρ‹ ΠΏΠΎΠ΄ Π³Ρ€Π°Ρ„ΠΈΠΊΠΎΠΌ зависимости силы Ρ‚ΠΎΠΊΠ° ΠΎΡ‚ Π²Ρ€Π΅ΠΌΠ΅Π½ΠΈ.

Если сила Ρ‚ΠΎΠΊΠ° ΠΈ Π΅Π³ΠΎ Π½Π°ΠΏΡ€Π°Π²Π»Π΅Π½ΠΈΠ΅ Π½Π΅ ΠΈΠ·ΠΌΠ΅Π½ΡΡŽΡ‚ΡΡ со Π²Ρ€Π΅ΠΌΠ΅Π½Π΅ΠΌ, Ρ‚ΠΎ Ρ‚Π°ΠΊΠΎΠΉ Ρ‚ΠΎΠΊ называСтся постоянным. Π‘ΠΈΠ»Π° Ρ‚ΠΎΠΊΠ° измСряСтся Π°ΠΌΠΏΠ΅Ρ€ΠΌΠ΅Ρ‚Ρ€ΠΎΠΌ, ΠΊΠΎΡ‚ΠΎΡ€Ρ‹ΠΉ Π²ΠΊΠ»ΡŽΡ‡Π°Π΅Ρ‚ΡΡ Π² Ρ†Π΅ΠΏΡŒ ΠΏΠΎΡΠ»Π΅Π΄ΠΎΠ²Π°Ρ‚Π΅Π»ΡŒΠ½ΠΎ. Π’ ΠœΠ΅ΠΆΠ΄ΡƒΠ½Π°Ρ€ΠΎΠ΄Π½ΠΎΠΉ систСмС Π΅Π΄ΠΈΠ½ΠΈΡ† БИ сила Ρ‚ΠΎΠΊΠ° измСряСтся Π² Π°ΠΌΠΏΠ΅Ρ€Π°Ρ… [А]. 1 А = 1 Кл/с.

БрСдняя сила Ρ‚ΠΎΠΊΠ° находится ΠΊΠ°ΠΊ ΠΎΡ‚Π½ΠΎΡˆΠ΅Π½ΠΈΠ΅ всСго заряда ΠΊΠΎ всСму Π²Ρ€Π΅ΠΌΠ΅Π½ΠΈ (Ρ‚.Π΅. ΠΏΠΎ Ρ‚ΠΎΠΌΡƒ ΠΆΠ΅ ΠΏΡ€ΠΈΠ½Ρ†ΠΈΠΏΡƒ, Ρ‡Ρ‚ΠΎ ΠΈ срСдняя ΡΠΊΠΎΡ€ΠΎΡΡ‚ΡŒ ΠΈΠ»ΠΈ любая другая срСдняя Π²Π΅Π»ΠΈΡ‡ΠΈΠ½Π° Π² Ρ„ΠΈΠ·ΠΈΠΊΠ΅):

БрСдняя сила Ρ‚ΠΎΠΊΠ°

Если ΠΆΠ΅ Ρ‚ΠΎΠΊ Ρ€Π°Π²Π½ΠΎΠΌΠ΅Ρ€Π½ΠΎ мСняСтся с Ρ‚Π΅Ρ‡Π΅Π½ΠΈΠ΅ΠΌ Π²Ρ€Π΅ΠΌΠ΅Π½ΠΈ ΠΎΡ‚ значСния I1 Π΄ΠΎ значСния I2, Ρ‚ΠΎ ΠΌΠΎΠΆΠ½ΠΎ Π·Π½Π°Ρ‡Π΅Π½ΠΈΠ΅ срСднСго Ρ‚ΠΎΠΊΠ° ΠΌΠΎΠΆΠ½ΠΎ Π½Π°ΠΉΡ‚ΠΈ ΠΊΠ°ΠΊ срСднСарифмСтичСскоС ΠΊΡ€Π°ΠΉΠ½ΠΈΡ… Π·Π½Π°Ρ‡Π΅Π½ΠΈΠΉ:

БрСдняя сила Ρ‚ΠΎΠΊΠ°

ΠŸΠ»ΠΎΡ‚Π½ΠΎΡΡ‚ΡŒ Ρ‚ΠΎΠΊΠ° – сила Ρ‚ΠΎΠΊΠ°, приходящаяся Π½Π° Π΅Π΄ΠΈΠ½ΠΈΡ†Ρƒ ΠΏΠΎΠΏΠ΅Ρ€Π΅Ρ‡Π½ΠΎΠ³ΠΎ сСчСния ΠΏΡ€ΠΎΠ²ΠΎΠ΄Π½ΠΈΠΊΠ°, рассчитываСтся ΠΏΠΎ Ρ„ΠΎΡ€ΠΌΡƒΠ»Π΅:

Π€ΠΎΡ€ΠΌΡƒΠ»Π° ΠŸΠ»ΠΎΡ‚Π½ΠΎΡΡ‚ΡŒ Ρ‚ΠΎΠΊΠ°

ΠŸΡ€ΠΈ ΠΏΡ€ΠΎΡ…ΠΎΠΆΠ΄Π΅Π½ΠΈΠΈ Ρ‚ΠΎΠΊΠ° ΠΏΠΎ ΠΏΡ€ΠΎΠ²ΠΎΠ΄Π½ΠΈΠΊΡƒ Ρ‚ΠΎΠΊ испытываСт сопротивлСниС со стороны ΠΏΡ€ΠΎΠ²ΠΎΠ΄Π½ΠΈΠΊΠ°. ΠŸΡ€ΠΈΡ‡ΠΈΠ½Π° сопротивлСния – взаимодСйствиС зарядов с Π°Ρ‚ΠΎΠΌΠ°ΠΌΠΈ вСщСства ΠΏΡ€ΠΎΠ²ΠΎΠ΄Π½ΠΈΠΊΠ° ΠΈ ΠΌΠ΅ΠΆΠ΄Ρƒ собой. Π•Π΄ΠΈΠ½ΠΈΡ†Π° измСрСния сопротивлСния 1 Ом. Π‘ΠΎΠΏΡ€ΠΎΡ‚ΠΈΠ²Π»Π΅Π½ΠΈΠ΅ ΠΏΡ€ΠΎΠ²ΠΎΠ΄Π½ΠΈΠΊΠ° R опрСдСляСтся ΠΏΠΎ Ρ„ΠΎΡ€ΠΌΡƒΠ»Π΅:

Π€ΠΎΡ€ΠΌΡƒΠ»Π° Π‘ΠΎΠΏΡ€ΠΎΡ‚ΠΈΠ²Π»Π΅Π½ΠΈΠ΅ ΠΏΡ€ΠΎΠ²ΠΎΠ΄Π½ΠΈΠΊΠ°

Π³Π΄Π΅: l – Π΄Π»ΠΈΠ½Π° ΠΏΡ€ΠΎΠ²ΠΎΠ΄Π½ΠΈΠΊΠ°, S – ΠΏΠ»ΠΎΡ‰Π°Π΄ΡŒ Π΅Π³ΠΎ ΠΏΠΎΠΏΠ΅Ρ€Π΅Ρ‡Π½ΠΎΠ³ΠΎ сСчСния, ρ – ΡƒΠ΄Π΅Π»ΡŒΠ½ΠΎΠ΅ сопротивлСниС ΠΌΠ°Ρ‚Π΅Ρ€ΠΈΠ°Π»Π° ΠΏΡ€ΠΎΠ²ΠΎΠ΄Π½ΠΈΠΊΠ° (Π±ΡƒΠ΄ΡŒΡ‚Π΅ Π²Π½ΠΈΠΌΠ°Ρ‚Π΅Π»ΡŒΠ½Ρ‹ ΠΈ Π½Π΅ ΠΏΠ΅Ρ€Π΅ΠΏΡƒΡ‚Π°ΠΉΡ‚Π΅ послСднюю Π²Π΅Π»ΠΈΡ‡ΠΈΠ½Ρƒ с ΠΏΠ»ΠΎΡ‚Π½ΠΎΡΡ‚ΡŒΡŽ вСщСства), ΠΊΠΎΡ‚ΠΎΡ€ΠΎΠ΅ Ρ…Π°Ρ€Π°ΠΊΡ‚Π΅Ρ€ΠΈΠ·ΡƒΠ΅Ρ‚ ΡΠΏΠΎΡΠΎΠ±Π½ΠΎΡΡ‚ΡŒ ΠΌΠ°Ρ‚Π΅Ρ€ΠΈΠ°Π»Π° ΠΏΡ€ΠΎΠ²ΠΎΠ΄Π½ΠΈΠΊΠ° ΠΏΡ€ΠΎΡ‚ΠΈΠ²ΠΎΠ΄Π΅ΠΉΡΡ‚Π²ΠΎΠ²Π°Ρ‚ΡŒ ΠΏΡ€ΠΎΡ…ΠΎΠΆΠ΄Π΅Π½ΠΈΡŽ Ρ‚ΠΎΠΊΠ°. Π’ΠΎ Π΅ΡΡ‚ΡŒ это такая ΠΆΠ΅ характСристика вСщСства, ΠΊΠ°ΠΊ ΠΈ ΠΌΠ½ΠΎΠ³ΠΈΠ΅ Π΄Ρ€ΡƒΠ³ΠΈΠ΅: ΡƒΠ΄Π΅Π»ΡŒΠ½Π°Ρ Ρ‚Π΅ΠΏΠ»ΠΎΠ΅ΠΌΠΊΠΎΡΡ‚ΡŒ, ΠΏΠ»ΠΎΡ‚Π½ΠΎΡΡ‚ΡŒ, Ρ‚Π΅ΠΌΠΏΠ΅Ρ€Π°Ρ‚ΡƒΡ€Π° плавлСния ΠΈ Ρ‚.Π΄. Π•Π΄ΠΈΠ½ΠΈΡ†Π° измСрСния ΡƒΠ΄Π΅Π»ΡŒΠ½ΠΎΠ³ΠΎ сопротивлСния 1 Ом·м. УдСльноС сопротивлСниС вСщСства – табличная Π²Π΅Π»ΠΈΡ‡ΠΈΠ½Π°.

Π‘ΠΎΠΏΡ€ΠΎΡ‚ΠΈΠ²Π»Π΅Π½ΠΈΠ΅ ΠΏΡ€ΠΎΠ²ΠΎΠ΄Π½ΠΈΠΊΠ° зависит ΠΈ ΠΎΡ‚ Π΅Π³ΠΎ Ρ‚Π΅ΠΌΠΏΠ΅Ρ€Π°Ρ‚ΡƒΡ€Ρ‹:

Π€ΠΎΡ€ΠΌΡƒΠ»Π° Π—Π°Π²ΠΈΡΠΈΠΌΠΎΡΡ‚ΡŒ сопротивлСния ΠΏΡ€ΠΎΠ²ΠΎΠ΄Π½ΠΈΠΊΠ° ΠΎΡ‚ Ρ‚Π΅ΠΌΠΏΠ΅Ρ€Π°Ρ‚ΡƒΡ€Ρ‹

Π³Π΄Π΅: R0 – сопротивлСниС ΠΏΡ€ΠΎΠ²ΠΎΠ΄Π½ΠΈΠΊΠ° ΠΏΡ€ΠΈ 0Β°Π‘, t – Ρ‚Π΅ΠΌΠΏΠ΅Ρ€Π°Ρ‚ΡƒΡ€Π°, выраТСнная Π² градусах ЦСльсия, α – Ρ‚Π΅ΠΌΠΏΠ΅Ρ€Π°Ρ‚ΡƒΡ€Π½Ρ‹ΠΉ коэффициСнт сопротивлСния. Он Ρ€Π°Π²Π΅Π½ ΠΎΡ‚Π½ΠΎΡΠΈΡ‚Π΅Π»ΡŒΠ½ΠΎΠΌΡƒ измСнСнию сопротивлСния, ΠΏΡ€ΠΈ ΡƒΠ²Π΅Π»ΠΈΡ‡Π΅Π½ΠΈΠΈ Ρ‚Π΅ΠΌΠΏΠ΅Ρ€Π°Ρ‚ΡƒΡ€Ρ‹ Π½Π° 1Β°Π‘. Для ΠΌΠ΅Ρ‚Π°Π»Π»ΠΎΠ² ΠΎΠ½ всСгда большС нуля, для элСктролитов Π½Π°ΠΎΠ±ΠΎΡ€ΠΎΡ‚, всСгда мСньшС нуля.

Π”ΠΈΠΎΠ΄ Π² Ρ†Π΅ΠΏΠΈ постоянного Ρ‚ΠΎΠΊΠ°

Π”ΠΈΠΎΠ΄ – это Π½Π΅Π»ΠΈΠ½Π΅ΠΉΠ½Ρ‹ΠΉ элСмСнт Ρ†Π΅ΠΏΠΈ, сопротивлСниС ΠΊΠΎΡ‚ΠΎΡ€ΠΎΠ³ΠΎ зависит ΠΎΡ‚ направлСния протСкания Ρ‚ΠΎΠΊΠ°. ΠžΠ±ΠΎΠ·Π½Π°Ρ‡Π°Π΅Ρ‚ΡΡ Π΄ΠΈΠΎΠ΄ ΡΠ»Π΅Π΄ΡƒΡŽΡ‰ΠΈΠΌ ΠΎΠ±Ρ€Π°Π·ΠΎΠΌ:

ΠžΠ±ΠΎΠ·Π½Π°Ρ‡Π΅Π½ΠΈΠ΅ Π΄ΠΈΠΎΠ΄Π°

Π‘Ρ‚Ρ€Π΅Π»ΠΊΠ° Π² схСматичСском ΠΎΠ±ΠΎΠ·Π½Π°Ρ‡Π΅Π½ΠΈΠΈ Π΄ΠΈΠΎΠ΄Π° ΠΏΠΎΠΊΠ°Π·Ρ‹Π²Π°Π΅Ρ‚, Π² ΠΊΠ°ΠΊΠΎΠΌ Π½Π°ΠΏΡ€Π°Π²Π»Π΅Π½ΠΈΠΈ ΠΎΠ½ пропускаСт Ρ‚ΠΎΠΊ. Π’ этом случаС Π΅Π³ΠΎ сопротивлСниС Ρ€Π°Π²Π½ΠΎ Π½ΡƒΠ»ΡŽ, ΠΈ Π΄ΠΈΠΎΠ΄ ΠΌΠΎΠΆΠ½ΠΎ Π·Π°ΠΌΠ΅Π½ΠΈΡ‚ΡŒ просто Π½Π° ΠΏΡ€ΠΎΠ²ΠΎΠ΄Π½ΠΈΠΊ с Π½ΡƒΠ»Π΅Π²Ρ‹ΠΌ сопротивлСниСм. Если Ρ‚ΠΎΠΊ Ρ‚Π΅Ρ‡Π΅Ρ‚ Ρ‡Π΅Ρ€Π΅Π· Π΄ΠΈΠΎΠ΄ Π² ΠΏΡ€ΠΎΡ‚ΠΈΠ²ΠΎΠΏΠΎΠ»ΠΎΠΆΠ½ΠΎΠΌ Π½Π°ΠΏΡ€Π°Π²Π»Π΅Π½ΠΈΠΈ, Ρ‚ΠΎ Π΄ΠΈΠΎΠ΄ ΠΎΠ±Π»Π°Π΄Π°Π΅Ρ‚ бСсконСчно большим сопротивлСниСм, Ρ‚ΠΎ Π΅ΡΡ‚ΡŒ Π½Π΅ пропускаСт Ρ‚ΠΎΠΊ совсСм, ΠΈ являСтся Ρ€Π°Π·Ρ€Ρ‹Π²ΠΎΠΌ Π² Ρ†Π΅ΠΏΠΈ. Π’ΠΎΠ³Π΄Π° участок Ρ†Π΅ΠΏΠΈ с Π΄ΠΈΠΎΠ΄ΠΎΠΌ ΠΌΠΎΠΆΠ½ΠΎ просто Π²Ρ‹Ρ‡Π΅Ρ€ΠΊΠ½ΡƒΡ‚ΡŒ, Ρ‚Π°ΠΊ ΠΊΠ°ΠΊ Ρ‚ΠΎΠΊ ΠΏΠΎ Π½Π΅ΠΌΡƒ Π½Π΅ ΠΈΠ΄Π΅Ρ‚.

Β 

Π—Π°ΠΊΠΎΠ½ Ома. ΠŸΠΎΡΠ»Π΅Π΄ΠΎΠ²Π°Ρ‚Π΅Π»ΡŒΠ½ΠΎΠ΅ ΠΈ ΠΏΠ°Ρ€Π°Π»Π»Π΅Π»ΡŒΠ½ΠΎΠ΅ соСдинСниС ΠΏΡ€ΠΎΠ²ΠΎΠ΄Π½ΠΈΠΊΠΎΠ²

К оглавлСнию…

НСмСцкий Ρ„ΠΈΠ·ΠΈΠΊ Π“.Ом Π² 1826 Π³ΠΎΠ΄Ρƒ ΡΠΊΡΠΏΠ΅Ρ€ΠΈΠΌΠ΅Π½Ρ‚Π°Π»ΡŒΠ½ΠΎ установил, Ρ‡Ρ‚ΠΎ сила Ρ‚ΠΎΠΊΠ° I, Ρ‚Π΅ΠΊΡƒΡ‰Π΅Π³ΠΎ ΠΏΠΎ ΠΎΠ΄Π½ΠΎΡ€ΠΎΠ΄Π½ΠΎΠΌΡƒ мСталличСскому ΠΏΡ€ΠΎΠ²ΠΎΠ΄Π½ΠΈΠΊΡƒ (Ρ‚ΠΎ Π΅ΡΡ‚ΡŒ ΠΏΡ€ΠΎΠ²ΠΎΠ΄Π½ΠΈΠΊΡƒ, Π² ΠΊΠΎΡ‚ΠΎΡ€ΠΎΠΌ Π½Π΅ Π΄Π΅ΠΉΡΡ‚Π²ΡƒΡŽΡ‚ сторонниС силы) сопротивлСниСм R, ΠΏΡ€ΠΎΠΏΠΎΡ€Ρ†ΠΈΠΎΠ½Π°Π»ΡŒΠ½Π° Π½Π°ΠΏΡ€ΡΠΆΠ΅Π½ΠΈΡŽ U Π½Π° ΠΊΠΎΠ½Ρ†Π°Ρ… ΠΏΡ€ΠΎΠ²ΠΎΠ΄Π½ΠΈΠΊΠ°:

Π€ΠΎΡ€ΠΌΡƒΠ»Π° Π—Π°ΠΊΠΎΠ½ Ома

Π’Π΅Π»ΠΈΡ‡ΠΈΠ½Ρƒ R принято Π½Π°Π·Ρ‹Π²Π°Ρ‚ΡŒ элСктричСским сопротивлСниСм. ΠŸΡ€ΠΎΠ²ΠΎΠ΄Π½ΠΈΠΊ, ΠΎΠ±Π»Π°Π΄Π°ΡŽΡ‰ΠΈΠΉ элСктричСским сопротивлСниСм, называСтся рСзистором. Π­Ρ‚ΠΎ ΡΠΎΠΎΡ‚Π½ΠΎΡˆΠ΅Π½ΠΈΠ΅ Π²Ρ‹Ρ€Π°ΠΆΠ°Π΅Ρ‚ Π·Π°ΠΊΠΎΠ½ Ома для ΠΎΠ΄Π½ΠΎΡ€ΠΎΠ΄Π½ΠΎΠ³ΠΎ участка Ρ†Π΅ΠΏΠΈ: сила Ρ‚ΠΎΠΊΠ° Π² ΠΏΡ€ΠΎΠ²ΠΎΠ΄Π½ΠΈΠΊΠ΅ прямо ΠΏΡ€ΠΎΠΏΠΎΡ€Ρ†ΠΈΠΎΠ½Π°Π»ΡŒΠ½Π° ΠΏΡ€ΠΈΠ»ΠΎΠΆΠ΅Π½Π½ΠΎΠΌΡƒ Π½Π°ΠΏΡ€ΡΠΆΠ΅Π½ΠΈΡŽ ΠΈ ΠΎΠ±Ρ€Π°Ρ‚Π½ΠΎ ΠΏΡ€ΠΎΠΏΠΎΡ€Ρ†ΠΈΠΎΠ½Π°Π»ΡŒΠ½Π° ΡΠΎΠΏΡ€ΠΎΡ‚ΠΈΠ²Π»Π΅Π½ΠΈΡŽ ΠΏΡ€ΠΎΠ²ΠΎΠ΄Π½ΠΈΠΊΠ°.

ΠŸΡ€ΠΎΠ²ΠΎΠ΄Π½ΠΈΠΊΠΈ, ΠΏΠΎΠ΄Ρ‡ΠΈΠ½ΡΡŽΡ‰ΠΈΠ΅ΡΡ Π·Π°ΠΊΠΎΠ½Ρƒ Ома, Π½Π°Π·Ρ‹Π²Π°ΡŽΡ‚ΡΡ Π»ΠΈΠ½Π΅ΠΉΠ½Ρ‹ΠΌΠΈ. ГрафичСская Π·Π°Π²ΠΈΡΠΈΠΌΠΎΡΡ‚ΡŒ силы Ρ‚ΠΎΠΊΠ° I ΠΎΡ‚ напряТСния U (Ρ‚Π°ΠΊΠΈΠ΅ Π³Ρ€Π°Ρ„ΠΈΠΊΠΈ Π½Π°Π·Ρ‹Π²Π°ΡŽΡ‚ΡΡ Π²ΠΎΠ»ΡŒΡ‚-Π°ΠΌΠΏΠ΅Ρ€Π½Ρ‹ΠΌΠΈ характСристиками, сокращСнно ВАΠ₯) изобраТаСтся прямой Π»ΠΈΠ½ΠΈΠ΅ΠΉ, проходящСй Ρ‡Π΅Ρ€Π΅Π· Π½Π°Ρ‡Π°Π»ΠΎ ΠΊΠΎΠΎΡ€Π΄ΠΈΠ½Π°Ρ‚. Π‘Π»Π΅Π΄ΡƒΠ΅Ρ‚ ΠΎΡ‚ΠΌΠ΅Ρ‚ΠΈΡ‚ΡŒ, Ρ‡Ρ‚ΠΎ сущСствуСт ΠΌΠ½ΠΎΠ³ΠΎ ΠΌΠ°Ρ‚Π΅Ρ€ΠΈΠ°Π»ΠΎΠ² ΠΈ устройств, Π½Π΅ ΠΏΠΎΠ΄Ρ‡ΠΈΠ½ΡΡŽΡ‰ΠΈΡ…ΡΡ Π·Π°ΠΊΠΎΠ½Ρƒ Ома, Π½Π°ΠΏΡ€ΠΈΠΌΠ΅Ρ€, ΠΏΠΎΠ»ΡƒΠΏΡ€ΠΎΠ²ΠΎΠ΄Π½ΠΈΠΊΠΎΠ²Ρ‹ΠΉ Π΄ΠΈΠΎΠ΄ ΠΈΠ»ΠΈ газоразрядная Π»Π°ΠΌΠΏΠ°. Π”Π°ΠΆΠ΅ Ρƒ мСталличСских ΠΏΡ€ΠΎΠ²ΠΎΠ΄Π½ΠΈΠΊΠΎΠ² ΠΏΡ€ΠΈ достаточно Π±ΠΎΠ»ΡŒΡˆΠΈΡ… Ρ‚ΠΎΠΊΠ°Ρ… Π½Π°Π±Π»ΡŽΠ΄Π°Π΅Ρ‚ΡΡ ΠΎΡ‚ΠΊΠ»ΠΎΠ½Π΅Π½ΠΈΠ΅ ΠΎΡ‚ Π»ΠΈΠ½Π΅ΠΉΠ½ΠΎΠ³ΠΎ Π·Π°ΠΊΠΎΠ½Π° Ома, Ρ‚Π°ΠΊ ΠΊΠ°ΠΊ элСктричСскоС сопротивлСниС мСталличСских ΠΏΡ€ΠΎΠ²ΠΎΠ΄Π½ΠΈΠΊΠΎΠ² растСт с ростом Ρ‚Π΅ΠΌΠΏΠ΅Ρ€Π°Ρ‚ΡƒΡ€Ρ‹.

ΠŸΡ€ΠΎΠ²ΠΎΠ΄Π½ΠΈΠΊΠΈ Π² элСктричСских цСпях ΠΌΠΎΠΆΠ½ΠΎ ΡΠΎΠ΅Π΄ΠΈΠ½ΡΡ‚ΡŒ двумя способами: ΠΏΠΎΡΠ»Π΅Π΄ΠΎΠ²Π°Ρ‚Π΅Π»ΡŒΠ½ΠΎ ΠΈ ΠΏΠ°Ρ€Π°Π»Π»Π΅Π»ΡŒΠ½ΠΎ. Π£ ΠΊΠ°ΠΆΠ΄ΠΎΠ³ΠΎ способа Π΅ΡΡ‚ΡŒ свои закономСрности.

1. ЗакономСрности ΠΏΠΎΡΠ»Π΅Π΄ΠΎΠ²Π°Ρ‚Π΅Π»ΡŒΠ½ΠΎΠ³ΠΎ соСдинСния:

Π€ΠΎΡ€ΠΌΡƒΠ»Π° ЗакономСрности ΠΏΠΎΡΠ»Π΅Π΄ΠΎΠ²Π°Ρ‚Π΅Π»ΡŒΠ½ΠΎΠ³ΠΎ соСдинСния

Π€ΠΎΡ€ΠΌΡƒΠ»Π° для ΠΎΠ±Ρ‰Π΅Π³ΠΎ сопротивлСния ΠΏΠΎΡΠ»Π΅Π΄ΠΎΠ²Π°Ρ‚Π΅Π»ΡŒΠ½ΠΎ соСдинСнных рСзисторов справСдлива для любого числа ΠΏΡ€ΠΎΠ²ΠΎΠ΄Π½ΠΈΠΊΠΎΠ². Если ΠΆΠ΅ Π² Ρ†Π΅ΠΏΡŒ ΠΏΠΎΡΠ»Π΅Π΄ΠΎΠ²Π°Ρ‚Π΅Π»ΡŒΠ½ΠΎ Π²ΠΊΠ»ΡŽΡ‡Π΅Π½ΠΎ n ΠΎΠ΄ΠΈΠ½Π°ΠΊΠΎΠ²Ρ‹Ρ… сопротивлСний R, Ρ‚ΠΎ ΠΎΠ±Ρ‰Π΅Π΅ сопротивлСниС R0 находится ΠΏΠΎ Ρ„ΠΎΡ€ΠΌΡƒΠ»Π΅:

ΠžΠ±Ρ‰Π΅Π΅ сопротивлСниС n ΠΏΠΎΡΠ»Π΅Π΄ΠΎΠ²Π°Ρ‚Π΅Π»ΡŒΠ½ΠΎ соСдинСнных рСзисторов

2. ЗакономСрности ΠΏΠ°Ρ€Π°Π»Π»Π΅Π»ΡŒΠ½ΠΎΠ³ΠΎ соСдинСния:

Π€ΠΎΡ€ΠΌΡƒΠ»Π° ЗакономСрности ΠΏΠ°Ρ€Π°Π»Π»Π΅Π»ΡŒΠ½ΠΎΠ³ΠΎ соСдинСния

Π€ΠΎΡ€ΠΌΡƒΠ»Π° для ΠΎΠ±Ρ‰Π΅Π³ΠΎ сопротивлСния ΠΏΠ°Ρ€Π°Π»Π»Π΅Π»ΡŒΠ½ΠΎ соСдинСнных рСзисторов справСдлива для любого числа ΠΏΡ€ΠΎΠ²ΠΎΠ΄Π½ΠΈΠΊΠΎΠ². Если ΠΆΠ΅ Π² Ρ†Π΅ΠΏΡŒ ΠΏΠ°Ρ€Π°Π»Π»Π΅Π»ΡŒΠ½ΠΎ Π²ΠΊΠ»ΡŽΡ‡Π΅Π½ΠΎ n ΠΎΠ΄ΠΈΠ½Π°ΠΊΠΎΠ²Ρ‹Ρ… сопротивлСний R, Ρ‚ΠΎ ΠΎΠ±Ρ‰Π΅Π΅ сопротивлСниС R0 находится ΠΏΠΎ Ρ„ΠΎΡ€ΠΌΡƒΠ»Π΅:

ΠžΠ±Ρ‰Π΅Π΅ сопротивлСниС n ΠΏΠ°Ρ€Π°Π»Π»Π΅Π»ΡŒΠ½ΠΎ соСдинСнных рСзисторов

Π­Π»Π΅ΠΊΡ‚Ρ€ΠΎΠΈΠ·ΠΌΠ΅Ρ€ΠΈΡ‚Π΅Π»ΡŒΠ½Ρ‹Π΅ ΠΏΡ€ΠΈΠ±ΠΎΡ€Ρ‹

Для измСрСния напряТСний ΠΈ Ρ‚ΠΎΠΊΠΎΠ² Π² элСктричСских цСпях постоянного Ρ‚ΠΎΠΊΠ° ΠΈΡΠΏΠΎΠ»ΡŒΠ·ΡƒΡŽΡ‚ΡΡ ΡΠΏΠ΅Ρ†ΠΈΠ°Π»ΡŒΠ½Ρ‹Π΅ ΠΏΡ€ΠΈΠ±ΠΎΡ€Ρ‹ – Π²ΠΎΠ»ΡŒΡ‚ΠΌΠ΅Ρ‚Ρ€Ρ‹ ΠΈ Π°ΠΌΠΏΠ΅Ρ€ΠΌΠ΅Ρ‚Ρ€Ρ‹.

Π’ΠΎΠ»ΡŒΡ‚ΠΌΠ΅Ρ‚Ρ€ ΠΏΡ€Π΅Π΄Π½Π°Π·Π½Π°Ρ‡Π΅Π½ для измСрСния разности ΠΏΠΎΡ‚Π΅Π½Ρ†ΠΈΠ°Π»ΠΎΠ², ΠΏΡ€ΠΈΠ»ΠΎΠΆΠ΅Π½Π½ΠΎΠΉ ΠΊ Π΅Π³ΠΎ ΠΊΠ»Π΅ΠΌΠΌΠ°ΠΌ. Он ΠΏΠΎΠ΄ΠΊΠ»ΡŽΡ‡Π°Π΅Ρ‚ΡΡ ΠΏΠ°Ρ€Π°Π»Π»Π΅Π»ΡŒΠ½ΠΎ участку Ρ†Π΅ΠΏΠΈ, Π½Π° ΠΊΠΎΡ‚ΠΎΡ€ΠΎΠΌ производится ΠΈΠ·ΠΌΠ΅Ρ€Π΅Π½ΠΈΠ΅ разности ΠΏΠΎΡ‚Π΅Π½Ρ†ΠΈΠ°Π»ΠΎΠ². Π›ΡŽΠ±ΠΎΠΉ Π²ΠΎΠ»ΡŒΡ‚ΠΌΠ΅Ρ‚Ρ€ ΠΎΠ±Π»Π°Π΄Π°Π΅Ρ‚ Π½Π΅ΠΊΠΎΡ‚ΠΎΡ€Ρ‹ΠΌ Π²Π½ΡƒΡ‚Ρ€Π΅Π½Π½ΠΈΠΌ сопротивлСниСм RB. Для Ρ‚ΠΎΠ³ΠΎ Ρ‡Ρ‚ΠΎΠ±Ρ‹ Π²ΠΎΠ»ΡŒΡ‚ΠΌΠ΅Ρ‚Ρ€ Π½Π΅ вносил Π·Π°ΠΌΠ΅Ρ‚Π½ΠΎΠ³ΠΎ пСрСраспрСдСлСния Ρ‚ΠΎΠΊΠΎΠ² ΠΏΡ€ΠΈ ΠΏΠΎΠ΄ΠΊΠ»ΡŽΡ‡Π΅Π½ΠΈΠΈ ΠΊ измСряСмой Ρ†Π΅ΠΏΠΈ, Π΅Π³ΠΎ Π²Π½ΡƒΡ‚Ρ€Π΅Π½Π½Π΅Π΅ сопротивлСниС Π΄ΠΎΠ»ΠΆΠ½ΠΎ Π±Ρ‹Ρ‚ΡŒ Π²Π΅Π»ΠΈΠΊΠΎ ΠΏΠΎ ΡΡ€Π°Π²Π½Π΅Π½ΠΈΡŽ с сопротивлСниСм Ρ‚ΠΎΠ³ΠΎ участка Ρ†Π΅ΠΏΠΈ, ΠΊ ΠΊΠΎΡ‚ΠΎΡ€ΠΎΠΌΡƒ ΠΎΠ½ ΠΏΠΎΠ΄ΠΊΠ»ΡŽΡ‡Π΅Π½.

АмпСрмСтр ΠΏΡ€Π΅Π΄Π½Π°Π·Π½Π°Ρ‡Π΅Π½ для измСрСния силы Ρ‚ΠΎΠΊΠ° Π² Ρ†Π΅ΠΏΠΈ. АмпСрмСтр Π²ΠΊΠ»ΡŽΡ‡Π°Π΅Ρ‚ΡΡ ΠΏΠΎΡΠ»Π΅Π΄ΠΎΠ²Π°Ρ‚Π΅Π»ΡŒΠ½ΠΎ Π² Ρ€Π°Π·Ρ€Ρ‹Π² элСктричСской Ρ†Π΅ΠΏΠΈ, Ρ‡Ρ‚ΠΎΠ±Ρ‹ Ρ‡Π΅Ρ€Π΅Π· Π½Π΅Π³ΠΎ ΠΏΡ€ΠΎΡ…ΠΎΠ΄ΠΈΠ» вСсь измСряСмый Ρ‚ΠΎΠΊ. АмпСрмСтр Ρ‚Π°ΠΊΠΆΠ΅ ΠΎΠ±Π»Π°Π΄Π°Π΅Ρ‚ Π½Π΅ΠΊΠΎΡ‚ΠΎΡ€Ρ‹ΠΌ Π²Π½ΡƒΡ‚Ρ€Π΅Π½Π½ΠΈΠΌ сопротивлСниСм RA. Π’ ΠΎΡ‚Π»ΠΈΡ‡ΠΈΠ΅ ΠΎΡ‚ Π²ΠΎΠ»ΡŒΡ‚ΠΌΠ΅Ρ‚Ρ€Π°, Π²Π½ΡƒΡ‚Ρ€Π΅Π½Π½Π΅Π΅ сопротивлСниС Π°ΠΌΠΏΠ΅Ρ€ΠΌΠ΅Ρ‚Ρ€Π° Π΄ΠΎΠ»ΠΆΠ½ΠΎ Π±Ρ‹Ρ‚ΡŒ достаточно ΠΌΠ°Π»Ρ‹ΠΌ ΠΏΠΎ ΡΡ€Π°Π²Π½Π΅Π½ΠΈΡŽ с ΠΏΠΎΠ»Π½Ρ‹ΠΌ сопротивлСниСм всСй Ρ†Π΅ΠΏΠΈ.

Β 

Π­Π”Π‘. Π—Π°ΠΊΠΎΠ½ Ома для ΠΏΠΎΠ»Π½ΠΎΠΉ Ρ†Π΅ΠΏΠΈ

К оглавлСнию…

Для сущСствования постоянного Ρ‚ΠΎΠΊΠ° Π½Π΅ΠΎΠ±Ρ…ΠΎΠ΄ΠΈΠΌΠΎ Π½Π°Π»ΠΈΡ‡ΠΈΠ΅ Π² элСктричСской Π·Π°ΠΌΠΊΠ½ΡƒΡ‚ΠΎΠΉ Ρ†Π΅ΠΏΠΈ устройства, способного ΡΠΎΠ·Π΄Π°Π²Π°Ρ‚ΡŒ ΠΈ ΠΏΠΎΠ΄Π΄Π΅Ρ€ΠΆΠΈΠ²Π°Ρ‚ΡŒ разности ΠΏΠΎΡ‚Π΅Π½Ρ†ΠΈΠ°Π»ΠΎΠ² Π½Π° участках Ρ†Π΅ΠΏΠΈ Π·Π° счСт Ρ€Π°Π±ΠΎΡ‚Ρ‹ сил нСэлСктростатичСского происхоТдСния. Π’Π°ΠΊΠΈΠ΅ устройства Π½Π°Π·Ρ‹Π²Π°ΡŽΡ‚ΡΡ источниками постоянного Ρ‚ΠΎΠΊΠ°. Π‘ΠΈΠ»Ρ‹ нСэлСктростатичСского происхоТдСния, Π΄Π΅ΠΉΡΡ‚Π²ΡƒΡŽΡ‰ΠΈΠ΅ Π½Π° свободныС носитСли заряда со стороны источников Ρ‚ΠΎΠΊΠ°, Π½Π°Π·Ρ‹Π²Π°ΡŽΡ‚ΡΡ сторонними силами.

ΠŸΡ€ΠΈΡ€ΠΎΠ΄Π° сторонних сил ΠΌΠΎΠΆΠ΅Ρ‚ Π±Ρ‹Ρ‚ΡŒ Ρ€Π°Π·Π»ΠΈΡ‡Π½ΠΎΠΉ. Π’ Π³Π°Π»ΡŒΠ²Π°Π½ΠΈΡ‡Π΅ΡΠΊΠΈΡ… элСмСнтах ΠΈΠ»ΠΈ аккумуляторах ΠΎΠ½ΠΈ Π²ΠΎΠ·Π½ΠΈΠΊΠ°ΡŽΡ‚ Π² Ρ€Π΅Π·ΡƒΠ»ΡŒΡ‚Π°Ρ‚Π΅ элСктрохимичСских процСссов, Π² Π³Π΅Π½Π΅Ρ€Π°Ρ‚ΠΎΡ€Π°Ρ… постоянного Ρ‚ΠΎΠΊΠ° сторонниС силы Π²ΠΎΠ·Π½ΠΈΠΊΠ°ΡŽΡ‚ ΠΏΡ€ΠΈ Π΄Π²ΠΈΠΆΠ΅Π½ΠΈΠΈ ΠΏΡ€ΠΎΠ²ΠΎΠ΄Π½ΠΈΠΊΠΎΠ² Π² ΠΌΠ°Π³Π½ΠΈΡ‚Π½ΠΎΠΌ ΠΏΠΎΠ»Π΅. Под дСйствиСм сторонних сил элСктричСскиС заряды двиТутся Π²Π½ΡƒΡ‚Ρ€ΠΈ источника Ρ‚ΠΎΠΊΠ° ΠΏΡ€ΠΎΡ‚ΠΈΠ² сил элСктростатичСского поля, благодаря Ρ‡Π΅ΠΌΡƒ Π² Π·Π°ΠΌΠΊΠ½ΡƒΡ‚ΠΎΠΉ Ρ†Π΅ΠΏΠΈ ΠΌΠΎΠΆΠ΅Ρ‚ ΠΏΠΎΠ΄Π΄Π΅Ρ€ΠΆΠΈΠ²Π°Ρ‚ΡŒΡΡ постоянный элСктричСский Ρ‚ΠΎΠΊ.

ΠŸΡ€ΠΈ ΠΏΠ΅Ρ€Π΅ΠΌΠ΅Ρ‰Π΅Π½ΠΈΠΈ элСктричСских зарядов ΠΏΠΎ Ρ†Π΅ΠΏΠΈ постоянного Ρ‚ΠΎΠΊΠ° сторонниС силы, Π΄Π΅ΠΉΡΡ‚Π²ΡƒΡŽΡ‰ΠΈΠ΅ Π²Π½ΡƒΡ‚Ρ€ΠΈ источников Ρ‚ΠΎΠΊΠ°, ΡΠΎΠ²Π΅Ρ€ΡˆΠ°ΡŽΡ‚ Ρ€Π°Π±ΠΎΡ‚Ρƒ. ЀизичСская Π²Π΅Π»ΠΈΡ‡ΠΈΠ½Π°, равная ΠΎΡ‚Π½ΠΎΡˆΠ΅Π½ΠΈΡŽ Ρ€Π°Π±ΠΎΡ‚Ρ‹ Aст сторонних сил ΠΏΡ€ΠΈ ΠΏΠ΅Ρ€Π΅ΠΌΠ΅Ρ‰Π΅Π½ΠΈΠΈ заряда q ΠΎΡ‚ ΠΎΡ‚Ρ€ΠΈΡ†Π°Ρ‚Π΅Π»ΡŒΠ½ΠΎΠ³ΠΎ полюса источника Ρ‚ΠΎΠΊΠ° ΠΊ ΠΏΠΎΠ»ΠΎΠΆΠΈΡ‚Π΅Π»ΡŒΠ½ΠΎΠΌΡƒ ΠΊ Π²Π΅Π»ΠΈΡ‡ΠΈΠ½Π΅ этого заряда, называСтся элСктродвиТущСй силой источника (Π­Π”Π‘):

Π€ΠΎΡ€ΠΌΡƒΠ»Π° ЭлСктродвиТущая сила источника Ρ‚ΠΎΠΊΠ° (Π­Π”Π‘)

Π’Π°ΠΊΠΈΠΌ ΠΎΠ±Ρ€Π°Π·ΠΎΠΌ, Π­Π”Π‘ опрСдСляСтся Ρ€Π°Π±ΠΎΡ‚ΠΎΠΉ, ΡΠΎΠ²Π΅Ρ€ΡˆΠ°Π΅ΠΌΠΎΠΉ сторонними силами ΠΏΡ€ΠΈ ΠΏΠ΅Ρ€Π΅ΠΌΠ΅Ρ‰Π΅Π½ΠΈΠΈ Π΅Π΄ΠΈΠ½ΠΈΡ‡Π½ΠΎΠ³ΠΎ ΠΏΠΎΠ»ΠΎΠΆΠΈΡ‚Π΅Π»ΡŒΠ½ΠΎΠ³ΠΎ заряда. ЭлСктродвиТущая сила, ΠΊΠ°ΠΊ ΠΈ Ρ€Π°Π·Π½ΠΎΡΡ‚ΡŒ ΠΏΠΎΡ‚Π΅Π½Ρ†ΠΈΠ°Π»ΠΎΠ², измСряСтся Π²Β Π²ΠΎΠ»ΡŒΡ‚Π°Ρ… (Π’).

Π—Π°ΠΊΠΎΠ½ Ома для ΠΏΠΎΠ»Π½ΠΎΠΉ (Π·Π°ΠΌΠΊΠ½ΡƒΡ‚ΠΎΠΉ) Ρ†Π΅ΠΏΠΈ: сила Ρ‚ΠΎΠΊΠ° Π² Π·Π°ΠΌΠΊΠ½ΡƒΡ‚ΠΎΠΉ Ρ†Π΅ΠΏΠΈ Ρ€Π°Π²Π½Π° элСктродвиТущСй силС источника, Π΄Π΅Π»Π΅Π½Π½ΠΎΠΉ Π½Π° ΠΎΠ±Ρ‰Π΅Π΅ (Π²Π½ΡƒΡ‚Ρ€Π΅Π½Π½Π΅Π΅ + внСшнСС) сопротивлСниС Ρ†Π΅ΠΏΠΈ:

Π€ΠΎΡ€ΠΌΡƒΠ»Π° Π—Π°ΠΊΠΎΠ½ Ома для ΠΏΠΎΠ»Π½ΠΎΠΉ Ρ†Π΅ΠΏΠΈ

Π‘ΠΎΠΏΡ€ΠΎΡ‚ΠΈΠ²Π»Π΅Π½ΠΈΠ΅ r – Π²Π½ΡƒΡ‚Ρ€Π΅Π½Π½Π΅Π΅ (собствСнноС) сопротивлСниС источника Ρ‚ΠΎΠΊΠ° (зависит ΠΎΡ‚ Π²Π½ΡƒΡ‚Ρ€Π΅Π½Π½Π΅Π³ΠΎ строСния источника). Π‘ΠΎΠΏΡ€ΠΎΡ‚ΠΈΠ²Π»Π΅Π½ΠΈΠ΅ R – сопротивлСниС Π½Π°Π³Ρ€ΡƒΠ·ΠΊΠΈ (внСшнСС сопротивлСниС Ρ†Π΅ΠΏΠΈ).

ПадСниС напряТСния Π²ΠΎ внСшнСй Ρ†Π΅ΠΏΠΈ ΠΏΡ€ΠΈ этом Ρ€Π°Π²Π½ΠΎ (Π΅Π³ΠΎ Π΅Ρ‰Π΅ Π½Π°Π·Ρ‹Π²Π°ΡŽΡ‚ напряТСниСм Π½Π° ΠΊΠ»Π΅ΠΌΠΌΠ°Ρ… источника):

Π€ΠΎΡ€ΠΌΡƒΠ»Π° ПадСниС напряТСния Π²ΠΎ внСшнСй Ρ†Π΅ΠΏΠΈ НапряТСниС Π½Π° ΠΊΠ»Π΅ΠΌΠΌΠ°Ρ… источника

Π’Π°ΠΆΠ½ΠΎ ΠΏΠΎΠ½ΡΡ‚ΡŒ ΠΈ Π·Π°ΠΏΠΎΠΌΠ½ΠΈΡ‚ΡŒ: Π­Π”Π‘ ΠΈ Π²Π½ΡƒΡ‚Ρ€Π΅Π½Π½Π΅Π΅ сопротивлСниС источника Ρ‚ΠΎΠΊΠ° Π½Π΅ ΠΌΠ΅Π½ΡΡŽΡ‚ΡΡ, ΠΏΡ€ΠΈ ΠΏΠΎΠ΄ΠΊΠ»ΡŽΡ‡Π΅Π½ΠΈΠΈ Ρ€Π°Π·Π½Ρ‹Ρ… Π½Π°Π³Ρ€ΡƒΠ·ΠΎΠΊ.

Если сопротивлСниС Π½Π°Π³Ρ€ΡƒΠ·ΠΊΠΈ Ρ€Π°Π²Π½ΠΎ Π½ΡƒΠ»ΡŽ (источник замыкаСтся сам Π½Π° сСбя) ΠΈΠ»ΠΈ ΠΌΠ½ΠΎΠ³ΠΎ мСньшС сопротивлСния источника, Ρ‚ΠΎ Ρ‚ΠΎΠ³Π΄Π° Π² Ρ†Π΅ΠΏΠΈ ΠΏΠΎΡ‚Π΅Ρ‡Π΅Ρ‚ Ρ‚ΠΎΠΊ ΠΊΠΎΡ€ΠΎΡ‚ΠΊΠΎΠ³ΠΎ замыкания:

Π€ΠΎΡ€ΠΌΡƒΠ»Π° Π‘ΠΈΠ»Π° Ρ‚ΠΎΠΊΠ° ΠΊΠΎΡ€ΠΎΡ‚ΠΊΠΎΠ³ΠΎ замыкания

Π‘ΠΈΠ»Π° Ρ‚ΠΎΠΊΠ° ΠΊΠΎΡ€ΠΎΡ‚ΠΊΠΎΠ³ΠΎ замыкания – максимальная сила Ρ‚ΠΎΠΊΠ°, ΠΊΠΎΡ‚ΠΎΡ€ΡƒΡŽ ΠΌΠΎΠΆΠ½ΠΎ ΠΏΠΎΠ»ΡƒΡ‡ΠΈΡ‚ΡŒ ΠΎΡ‚ Π΄Π°Π½Π½ΠΎΠ³ΠΎ источника с элСктродвиТущСй силой Ρ и Π²Π½ΡƒΡ‚Ρ€Π΅Π½Π½ΠΈΠΌ сопротивлСниСм r. Π£ источников с ΠΌΠ°Π»Ρ‹ΠΌ Π²Π½ΡƒΡ‚Ρ€Π΅Π½Π½ΠΈΠΌ сопротивлСниСм Ρ‚ΠΎΠΊ ΠΊΠΎΡ€ΠΎΡ‚ΠΊΠΎΠ³ΠΎ замыкания ΠΌΠΎΠΆΠ΅Ρ‚ Π±Ρ‹Ρ‚ΡŒ ΠΎΡ‡Π΅Π½ΡŒ Π²Π΅Π»ΠΈΠΊ, ΠΈ Π²Ρ‹Π·Ρ‹Π²Π°Ρ‚ΡŒ Ρ€Π°Π·Ρ€ΡƒΡˆΠ΅Π½ΠΈΠ΅ элСктричСской Ρ†Π΅ΠΏΠΈ ΠΈΠ»ΠΈ источника. НапримСр, Ρƒ свинцовых аккумуляторов, ΠΈΡΠΏΠΎΠ»ΡŒΠ·ΡƒΠ΅ΠΌΡ‹Ρ… Π² автомобилях, сила Ρ‚ΠΎΠΊΠ° ΠΊΠΎΡ€ΠΎΡ‚ΠΊΠΎΠ³ΠΎ замыкания ΠΌΠΎΠΆΠ΅Ρ‚ ΡΠΎΡΡ‚Π°Π²Π»ΡΡ‚ΡŒ нСсколько сотСн Π°ΠΌΠΏΠ΅Ρ€. ОсобСнно опасны ΠΊΠΎΡ€ΠΎΡ‚ΠΊΠΈΠ΅ замыкания Π² ΠΎΡΠ²Π΅Ρ‚ΠΈΡ‚Π΅Π»ΡŒΠ½Ρ‹Ρ… сСтях, ΠΏΠΈΡ‚Π°Π΅ΠΌΡ‹Ρ… ΠΎΡ‚ подстанций (тысячи Π°ΠΌΠΏΠ΅Ρ€). Π§Ρ‚ΠΎΠ±Ρ‹ ΠΈΠ·Π±Π΅ΠΆΠ°Ρ‚ΡŒ Ρ€Π°Π·Ρ€ΡƒΡˆΠΈΡ‚Π΅Π»ΡŒΠ½ΠΎΠ³ΠΎ дСйствия Ρ‚Π°ΠΊΠΈΡ… Π±ΠΎΠ»ΡŒΡˆΠΈΡ… Ρ‚ΠΎΠΊΠΎΠ², Π² Ρ†Π΅ΠΏΡŒ Π²ΠΊΠ»ΡŽΡ‡Π°ΡŽΡ‚ΡΡ ΠΏΡ€Π΅Π΄ΠΎΡ…Ρ€Π°Π½ΠΈΡ‚Π΅Π»ΠΈ ΠΈΠ»ΠΈ ΡΠΏΠ΅Ρ†ΠΈΠ°Π»ΡŒΠ½Ρ‹Π΅ Π°Π²Ρ‚ΠΎΠΌΠ°Ρ‚Ρ‹ Π·Π°Ρ‰ΠΈΡ‚Ρ‹ сСтСй.

НСсколько источников Π­Π”Π‘ Π² Ρ†Π΅ΠΏΠΈ

Если Π² Ρ†Π΅ΠΏΠΈ присутствуСт нСсколько Π­Π”Π‘ ΠΏΠΎΠ΄ΠΊΠ»ΡŽΡ‡Π΅Π½Π½Ρ‹Ρ… ΠΏΠΎΡΠ»Π΅Π΄ΠΎΠ²Π°Ρ‚Π΅Π»ΡŒΠ½ΠΎ, Ρ‚ΠΎ:

1. ΠŸΡ€ΠΈ ΠΏΡ€Π°Π²ΠΈΠ»ΡŒΠ½ΠΎΠΌ (ΠΏΠΎΠ»ΠΎΠΆΠΈΡ‚Π΅Π»ΡŒΠ½Ρ‹ΠΉ полюс ΠΎΠ΄Π½ΠΎΠ³ΠΎ источника присоСдиняСтся ΠΊ ΠΎΡ‚Ρ€ΠΈΡ†Π°Ρ‚Π΅Π»ΡŒΠ½ΠΎΠΌΡƒ Π΄Ρ€ΡƒΠ³ΠΎΠ³ΠΎ) ΠΏΠΎΠ΄ΠΊΠ»ΡŽΡ‡Π΅Π½ΠΈΠΈ источников ΠΎΠ±Ρ‰Π΅Π΅ Π­Π”Π‘ всСх источников ΠΈ ΠΈΡ… Π²Π½ΡƒΡ‚Ρ€Π΅Π½Π½Π΅Π΅ сопротивлСниС ΠΌΠΎΠΆΠ΅Ρ‚ Π±Ρ‹Ρ‚ΡŒ Π½Π°ΠΉΠ΄Π΅Π½ΠΎ ΠΏΠΎ Ρ„ΠΎΡ€ΠΌΡƒΠ»Π°ΠΌ:

ΠŸΠΎΡΠ»Π΅Π΄ΠΎΠ²Π°Ρ‚Π΅Π»ΡŒΠ½ΠΎΠ΅ ΠΏΠΎΠ΄ΠΊΠ»ΡŽΡ‡Π΅Π½ΠΈΠ΅ Π­Π”Π‘

НапримСр, Ρ‚Π°ΠΊΠΎΠ΅ ΠΏΠΎΠ΄ΠΊΠ»ΡŽΡ‡Π΅Π½ΠΈΠ΅ источников осущСствляСтся Π² ΠΏΡƒΠ»ΡŒΡ‚Π°Ρ… дистанционного управлСния, Ρ„ΠΎΡ‚ΠΎΠ°ΠΏΠΏΠ°Ρ€Π°Ρ‚Π°Ρ… ΠΈ Π΄Ρ€ΡƒΠ³ΠΈΡ… Π±Ρ‹Ρ‚ΠΎΠ²Ρ‹Ρ… ΠΏΡ€ΠΈΠ±ΠΎΡ€Π°Ρ…, Ρ€Π°Π±ΠΎΡ‚Π°ΡŽΡ‰ΠΈΡ… ΠΎΡ‚ Π½Π΅ΡΠΊΠΎΠ»ΡŒΠΊΠΈΡ… Π±Π°Ρ‚Π°Ρ€Π΅Π΅ΠΊ.

2. ΠŸΡ€ΠΈ Π½Π΅ΠΏΡ€Π°Π²ΠΈΠ»ΡŒΠ½ΠΎΠΌ (источники ΡΠΎΠ΅Π΄ΠΈΠ½ΡΡŽΡ‚ΡΡ ΠΎΠ΄ΠΈΠ½Π°ΠΊΠΎΠ²Ρ‹ΠΌΠΈ полюсами) ΠΏΠΎΠ΄ΠΊΠ»ΡŽΡ‡Π΅Π½ΠΈΠΈ источников ΠΈΡ… ΠΎΠ±Ρ‰Π΅Π΅ Π­Π”Π‘ ΠΈ сопротивлСниС рассчитываСтся ΠΏΠΎ Ρ„ΠΎΡ€ΠΌΡƒΠ»Π°ΠΌ:

ΠŸΠΎΡΠ»Π΅Π΄ΠΎΠ²Π°Ρ‚Π΅Π»ΡŒΠ½ΠΎΠ΅ ΠΏΠΎΠ΄ΠΊΠ»ΡŽΡ‡Π΅Π½ΠΈΠ΅ Π­Π”Π‘

Π’ ΠΎΠ±ΠΎΠΈΡ… случаях ΠΎΠ±Ρ‰Π΅Π΅ сопротивлСниС источников увСличиваСтся.

ΠŸΡ€ΠΈ ΠΏΠ°Ρ€Π°Π»Π»Π΅Π»ΡŒΠ½ΠΎΠΌ ΠΏΠΎΠ΄ΠΊΠ»ΡŽΡ‡Π΅Π½ΠΈΠΈ ΠΈΠΌΠ΅Π΅Ρ‚ смысл ΡΠΎΠ΅Π΄ΠΈΠ½ΡΡ‚ΡŒ источники Ρ‚ΠΎΠ»ΡŒΠΊΠΎ c ΠΎΠ΄ΠΈΠ½Π°ΠΊΠΎΠ²ΠΎΠΉ Π­Π”Π‘, ΠΈΠ½Π°Ρ‡Π΅ источники Π±ΡƒΠ΄ΡƒΡ‚ Ρ€Π°Π·Ρ€ΡΠΆΠ°Ρ‚ΡŒΡΡ Π΄Ρ€ΡƒΠ³ Π½Π° Π΄Ρ€ΡƒΠ³Π°. Π’Π°ΠΊΠΈΠΌ ΠΎΠ±Ρ€Π°Π·ΠΎΠΌ суммарноС Π­Π”Π‘ Π±ΡƒΠ΄Π΅Ρ‚ Ρ‚Π°ΠΊΠΈΠΌ ΠΆΠ΅, ΠΊΠ°ΠΊ ΠΈ Π­Π”Π‘ ΠΊΠ°ΠΆΠ΄ΠΎΠ³ΠΎ источника, Ρ‚ΠΎ Π΅ΡΡ‚ΡŒ ΠΏΡ€ΠΈ ΠΏΠ°Ρ€Π°Π»Π»Π΅Π»ΡŒΠ½ΠΎΠΌ соСдинСнии ΠΌΡ‹ Π½Π΅ ΠΏΠΎΠ»ΡƒΡ‡ΠΈΠΌ Π±Π°Ρ‚Π°Ρ€Π΅ΡŽ с большим Π­Π”Π‘. ΠŸΡ€ΠΈ этом ΡƒΠΌΠ΅Π½ΡŒΡˆΠ°Π΅Ρ‚ΡΡ Π²Π½ΡƒΡ‚Ρ€Π΅Π½Π½Π΅Π΅ сопротивлСниС Π±Π°Ρ‚Π°Ρ€Π΅ΠΈ источников, Ρ‡Ρ‚ΠΎ позволяСт ΠΏΠΎΠ»ΡƒΡ‡Π°Ρ‚ΡŒ Π±ΠΎΠ»ΡŒΡˆΡƒΡŽ силу Ρ‚ΠΎΠΊΠ° ΠΈ ΠΌΠΎΡ‰Π½ΠΎΡΡ‚ΡŒ Π² Ρ†Π΅ΠΏΠΈ:

ΠŸΠ°Ρ€Π°Π»Π»Π΅Π»ΡŒΠ½ΠΎΠ΅ ΠΏΠΎΠ΄ΠΊΠ»ΡŽΡ‡Π΅Π½ΠΈΠ΅ Π­Π”Π‘

Π’ этом ΠΈ состоит смысл ΠΏΠ°Ρ€Π°Π»Π»Π΅Π»ΡŒΠ½ΠΎΠ³ΠΎ соСдинСния источников. Π’ любом случаС ΠΏΡ€ΠΈ Ρ€Π΅ΡˆΠ΅Π½ΠΈΠΈ Π·Π°Π΄Π°Ρ‡ сначала Π½Π°Π΄ΠΎ Π½Π°ΠΉΡ‚ΠΈ ΡΡƒΠΌΠΌΠ°Ρ€Π½ΡƒΡŽ Π­Π”Π‘ ΠΈ ΠΏΠΎΠ»Π½ΠΎΠ΅ Π²Π½ΡƒΡ‚Ρ€Π΅Π½Π½Π΅Π΅ сопротивлСниС ΠΏΠΎΠ»ΡƒΡ‡ΠΈΠ²ΡˆΠ΅Π³ΠΎΡΡ источника, Π° Π·Π°Ρ‚Π΅ΠΌ Π·Π°ΠΏΠΈΡΠ°Ρ‚ΡŒ Π·Π°ΠΊΠΎΠ½ Ома для ΠΏΠΎΠ»Π½ΠΎΠΉ Ρ†Π΅ΠΏΠΈ.

Β 

Π Π°Π±ΠΎΡ‚Π° ΠΈ ΠΌΠΎΡ‰Π½ΠΎΡΡ‚ΡŒ Ρ‚ΠΎΠΊΠ°. Π—Π°ΠΊΠΎΠ½ ДТоуля-Π›Π΅Π½Ρ†Π°

К оглавлСнию…

Π Π°Π±ΠΎΡ‚Π° A элСктричСского Ρ‚ΠΎΠΊΠ° I, ΠΏΡ€ΠΎΡ‚Π΅ΠΊΠ°ΡŽΡ‰Π΅Π³ΠΎ ΠΏΠΎ Π½Π΅ΠΏΠΎΠ΄Π²ΠΈΠΆΠ½ΠΎΠΌΡƒ ΠΏΡ€ΠΎΠ²ΠΎΠ΄Π½ΠΈΠΊΡƒ с сопротивлСниСм R, прСобразуСтся Π² Ρ‚Π΅ΠΏΠ»ΠΎΡ‚ΡƒΒ Q, Π²Ρ‹Π΄Π΅Π»ΡΡŽΡ‰Π΅Π΅ΡΡ Π½Π° ΠΏΡ€ΠΎΠ²ΠΎΠ΄Π½ΠΈΠΊΠ΅. Π­Ρ‚Ρƒ Ρ€Π°Π±ΠΎΡ‚Ρƒ ΠΌΠΎΠΆΠ½ΠΎ Ρ€Π°ΡΡΡ‡ΠΈΡ‚Π°Ρ‚ΡŒ ΠΏΠΎ ΠΎΠ΄Π½ΠΎΠΉ ΠΈΠ· Ρ„ΠΎΡ€ΠΌΡƒΠ» (с ΡƒΡ‡Π΅Ρ‚ΠΎΠΌ Π·Π°ΠΊΠΎΠ½Π° Ома всС ΠΎΠ½ΠΈ ΡΠ»Π΅Π΄ΡƒΡŽΡ‚ Π΄Ρ€ΡƒΠ³ ΠΈΠ· Π΄Ρ€ΡƒΠ³Π°):

Π€ΠΎΡ€ΠΌΡƒΠ»Π° Π Π°Π±ΠΎΡ‚Π° элСктричСского Ρ‚ΠΎΠΊΠ° Π—Π°ΠΊΠΎΠ½ ДТоуля-Π›Π΅Π½Ρ†Π°

Π—Π°ΠΊΠΎΠ½ прСобразования Ρ€Π°Π±ΠΎΡ‚Ρ‹ Ρ‚ΠΎΠΊΠ° Π² Ρ‚Π΅ΠΏΠ»ΠΎ Π±Ρ‹Π» ΡΠΊΡΠΏΠ΅Ρ€ΠΈΠΌΠ΅Π½Ρ‚Π°Π»ΡŒΠ½ΠΎ установлСн нСзависимо Π΄Ρ€ΡƒΠ³ ΠΎΡ‚ Π΄Ρ€ΡƒΠ³Π° Π”ΠΆ.Π”ΠΆΠΎΡƒΠ»Π΅ΠΌ ΠΈ Π­.Π›Π΅Π½Ρ†Π΅ΠΌ ΠΈ носит Π½Π°Π·Π²Π°Π½ΠΈΠ΅ Π·Π°ΠΊΠΎΠ½Π° ДТоуля–ЛСнца. ΠœΠΎΡ‰Π½ΠΎΡΡ‚ΡŒ элСктричСского Ρ‚ΠΎΠΊΠ° Ρ€Π°Π²Π½Π° ΠΎΡ‚Π½ΠΎΡˆΠ΅Π½ΠΈΡŽ Ρ€Π°Π±ΠΎΡ‚Ρ‹ Ρ‚ΠΎΠΊΠ° A ΠΊ ΠΈΠ½Ρ‚Π΅Ρ€Π²Π°Π»Ρƒ Π²Ρ€Π΅ΠΌΠ΅Π½ΠΈ Ξ”t, Π·Π° ΠΊΠΎΡ‚ΠΎΡ€ΠΎΠ΅ эта Ρ€Π°Π±ΠΎΡ‚Π° Π±Ρ‹Π»Π° ΡΠΎΠ²Π΅Ρ€ΡˆΠ΅Π½Π°, поэтому ΠΎΠ½Π° ΠΌΠΎΠΆΠ΅Ρ‚ Π±Ρ‹Ρ‚ΡŒ рассчитана ΠΏΠΎ ΡΠ»Π΅Π΄ΡƒΡŽΡ‰ΠΈΠΌ Ρ„ΠΎΡ€ΠΌΡƒΠ»Π°ΠΌ:

Π€ΠΎΡ€ΠΌΡƒΠ»Π° ΠœΠΎΡ‰Π½ΠΎΡΡ‚ΡŒ элСктричСского Ρ‚ΠΎΠΊΠ°

Π Π°Π±ΠΎΡ‚Π° элСктричСского Ρ‚ΠΎΠΊΠ° Π² БИ, ΠΊΠ°ΠΊ ΠΎΠ±Ρ‹Ρ‡Π½ΠΎ, выраТаСтся Π² дТоулях (Π”ΠΆ), ΠΌΠΎΡ‰Π½ΠΎΡΡ‚ΡŒ – Π² Π²Π°Ρ‚Ρ‚Π°Ρ… (Π’Ρ‚).

Β 

ЭнСргобаланс Π·Π°ΠΌΠΊΠ½ΡƒΡ‚ΠΎΠΉ Ρ†Π΅ΠΏΠΈ

К оглавлСнию…

Рассмотрим Ρ‚Π΅ΠΏΠ΅Ρ€ΡŒ ΠΏΠΎΠ»Π½ΡƒΡŽ Ρ†Π΅ΠΏΡŒ постоянного Ρ‚ΠΎΠΊΠ°, ΡΠΎΡΡ‚ΠΎΡΡ‰ΡƒΡŽ ΠΈΠ· источника с элСктродвиТущСй силой Ρ и Π²Π½ΡƒΡ‚Ρ€Π΅Π½Π½ΠΈΠΌ сопротивлСниСм r ΠΈ внСшнСго ΠΎΠ΄Π½ΠΎΡ€ΠΎΠ΄Π½ΠΎΠ³ΠΎ участка с сопротивлСниСм R. Π’ этом случаС полСзная ΠΌΠΎΡ‰Π½ΠΎΡΡ‚ΡŒ ΠΈΠ»ΠΈ ΠΌΠΎΡ‰Π½ΠΎΡΡ‚ΡŒ, выдСляСмая Π²ΠΎ внСшнСй Ρ†Π΅ΠΏΠΈ:

Π€ΠΎΡ€ΠΌΡƒΠ»Π° ΠœΠΎΡ‰Π½ΠΎΡΡ‚ΡŒ, выдСляСмая Π²ΠΎ внСшнСй Ρ†Π΅ΠΏΠΈ

Максимально возмоТная полСзная ΠΌΠΎΡ‰Π½ΠΎΡΡ‚ΡŒ источника достигаСтся, Ссли R = r ΠΈ Ρ€Π°Π²Π½Π°:

Π€ΠΎΡ€ΠΌΡƒΠ»Π° Максимально возмоТная полСзная ΠΌΠΎΡ‰Π½ΠΎΡΡ‚ΡŒ источника

Если ΠΏΡ€ΠΈ ΠΏΠΎΠ΄ΠΊΠ»ΡŽΡ‡Π΅Π½ΠΈΠΈ ΠΊ ΠΎΠ΄Π½ΠΎΠΌΡƒ ΠΈ Ρ‚ΠΎΠΌΡƒ ΠΆΠ΅ источнику Ρ‚ΠΎΠΊΠ° Ρ€Π°Π·Π½Ρ‹Ρ… сопротивлСний R1 ΠΈ R2 Π½Π° Π½ΠΈΡ… Π²Ρ‹Π΄Π΅Π»ΡΡŽΡ‚ΡΡ Ρ€Π°Π²Π½Ρ‹Π΅ мощности Ρ‚ΠΎ Π²Π½ΡƒΡ‚Ρ€Π΅Π½Π½Π΅Π΅ сопротивлСниС этого источника Ρ‚ΠΎΠΊΠ° ΠΌΠΎΠΆΠ΅Ρ‚ Π±Ρ‹Ρ‚ΡŒ Π½Π°ΠΉΠ΄Π΅Π½ΠΎ ΠΏΠΎ Ρ„ΠΎΡ€ΠΌΡƒΠ»Π΅:

Π€ΠΎΡ€ΠΌΡƒΠ»Π° Π’Π½ΡƒΡ‚Ρ€Π΅Π½Π½Π΅Π΅ сопротивлСниС источника Ρ‚ΠΎΠΊΠ° ΠΏΡ€ΠΈ Ρ€Π°Π²Π½Ρ‹Ρ… мощностях

ΠœΠΎΡ‰Π½ΠΎΡΡ‚ΡŒ ΠΏΠΎΡ‚Π΅Ρ€ΡŒ ΠΈΠ»ΠΈ ΠΌΠΎΡ‰Π½ΠΎΡΡ‚ΡŒ Π²Π½ΡƒΡ‚Ρ€ΠΈ источника Ρ‚ΠΎΠΊΠ°:

Π€ΠΎΡ€ΠΌΡƒΠ»Π° ΠœΠΎΡ‰Π½ΠΎΡΡ‚ΡŒ Π²Π½ΡƒΡ‚Ρ€ΠΈ источника Ρ‚ΠΎΠΊΠ°

Полная ΠΌΠΎΡ‰Π½ΠΎΡΡ‚ΡŒ, развиваСмая источником Ρ‚ΠΎΠΊΠ°:

Π€ΠΎΡ€ΠΌΡƒΠ»Π° Полная ΠΌΠΎΡ‰Π½ΠΎΡΡ‚ΡŒ, развиваСмая источником Ρ‚ΠΎΠΊΠ°

ΠšΠŸΠ” источника Ρ‚ΠΎΠΊΠ°:

Π€ΠΎΡ€ΠΌΡƒΠ»Π° ΠšΠŸΠ” источника Ρ‚ΠΎΠΊΠ°

Β 

Π­Π»Π΅ΠΊΡ‚Ρ€ΠΎΠ»ΠΈΠ·

К оглавлСнию…

Π­Π»Π΅ΠΊΡ‚Ρ€ΠΎΠ»ΠΈΡ‚Π°ΠΌΠΈ принято Π½Π°Π·Ρ‹Π²Π°Ρ‚ΡŒ проводящиС срСды, Π² ΠΊΠΎΡ‚ΠΎΡ€Ρ‹Ρ… ΠΏΡ€ΠΎΡ‚Π΅ΠΊΠ°Π½ΠΈΠ΅ элСктричСского Ρ‚ΠΎΠΊΠ° сопровоТдаСтся пСрСносом вСщСства. НоситСлями свободных зарядов Π² элСктролитах ΡΠ²Π»ΡΡŽΡ‚ΡΡ ΠΏΠΎΠ»ΠΎΠΆΠΈΡ‚Π΅Π»ΡŒΠ½ΠΎ ΠΈ ΠΎΡ‚Ρ€ΠΈΡ†Π°Ρ‚Π΅Π»ΡŒΠ½ΠΎ заряТСнныС ΠΈΠΎΠ½Ρ‹. К элСктролитам относятся ΠΌΠ½ΠΎΠ³ΠΈΠ΅ соСдинСния ΠΌΠ΅Ρ‚Π°Π»Π»ΠΎΠ² с ΠΌΠ΅Ρ‚Π°Π»Π»ΠΎΠΈΠ΄Π°ΠΌΠΈ Π² расплавлСнном состоянии, Π° Ρ‚Π°ΠΊΠΆΠ΅ Π½Π΅ΠΊΠΎΡ‚ΠΎΡ€Ρ‹Π΅ Ρ‚Π²Π΅Ρ€Π΄Ρ‹Π΅ вСщСства. Однако основными прСдставитСлями элСктролитов, ΡˆΠΈΡ€ΠΎΠΊΠΎ ΠΈΡΠΏΠΎΠ»ΡŒΠ·ΡƒΠ΅ΠΌΡ‹ΠΌΠΈ Π² Ρ‚Π΅Ρ…Π½ΠΈΠΊΠ΅, ΡΠ²Π»ΡΡŽΡ‚ΡΡ Π²ΠΎΠ΄Π½Ρ‹Π΅ растворы нСорганичСских кислот, солСй ΠΈ оснований.

ΠŸΡ€ΠΎΡ…ΠΎΠΆΠ΄Π΅Π½ΠΈΠ΅ элСктричСского Ρ‚ΠΎΠΊΠ° Ρ‡Π΅Ρ€Π΅Π· элСктролит сопровоТдаСтся Π²Ρ‹Π΄Π΅Π»Π΅Π½ΠΈΠ΅ΠΌ вСщСства Π½Π° элСктродах. Π­Ρ‚ΠΎ явлСниС ΠΏΠΎΠ»ΡƒΡ‡ΠΈΠ»ΠΎ Π½Π°Π·Π²Π°Π½ΠΈΠ΅ элСктролиза.

ЭлСктричСский Ρ‚ΠΎΠΊ Π² элСктролитах прСдставляСт собой ΠΏΠ΅Ρ€Π΅ΠΌΠ΅Ρ‰Π΅Π½ΠΈΠ΅ ΠΈΠΎΠ½ΠΎΠ² ΠΎΠ±ΠΎΠΈΡ… Π·Π½Π°ΠΊΠΎΠ² Π² ΠΏΡ€ΠΎΡ‚ΠΈΠ²ΠΎΠΏΠΎΠ»ΠΎΠΆΠ½Ρ‹Ρ… направлСниях. ΠŸΠΎΠ»ΠΎΠΆΠΈΡ‚Π΅Π»ΡŒΠ½Ρ‹Π΅ ΠΈΠΎΠ½Ρ‹ двиТутся ΠΊ ΠΎΡ‚Ρ€ΠΈΡ†Π°Ρ‚Π΅Π»ΡŒΠ½ΠΎΠΌΡƒ элСктроду (ΠΊΠ°Ρ‚ΠΎΠ΄Ρƒ), ΠΎΡ‚Ρ€ΠΈΡ†Π°Ρ‚Π΅Π»ΡŒΠ½Ρ‹Π΅ ΠΈΠΎΠ½Ρ‹ – ΠΊ ΠΏΠΎΠ»ΠΎΠΆΠΈΡ‚Π΅Π»ΡŒΠ½ΠΎΠΌΡƒ элСктроду (Π°Π½ΠΎΠ΄Ρƒ). Π˜ΠΎΠ½Ρ‹ ΠΎΠ±ΠΎΠΈΡ… Π·Π½Π°ΠΊΠΎΠ² ΠΏΠΎΡΠ²Π»ΡΡŽΡ‚ΡΡ Π² Π²ΠΎΠ΄Π½Ρ‹Ρ… растворах солСй, кислот ΠΈ Ρ‰Π΅Π»ΠΎΡ‡Π΅ΠΉ Π² Ρ€Π΅Π·ΡƒΠ»ΡŒΡ‚Π°Ρ‚Π΅ расщСплСния части Π½Π΅ΠΉΡ‚Ρ€Π°Π»ΡŒΠ½Ρ‹Ρ… ΠΌΠΎΠ»Π΅ΠΊΡƒΠ». Π­Ρ‚ΠΎ явлСниС называСтся элСктролитичСской диссоциациСй.

Π—Π°ΠΊΠΎΠ½ элСктролиза Π±Ρ‹Π» ΡΠΊΡΠΏΠ΅Ρ€ΠΈΠΌΠ΅Π½Ρ‚Π°Π»ΡŒΠ½ΠΎ установлСн английским Ρ„ΠΈΠ·ΠΈΠΊΠΎΠΌ М.Π€Π°Ρ€Π°Π΄Π΅Π΅ΠΌ Π² 1833 Π³ΠΎΠ΄Ρƒ. Π—Π°ΠΊΠΎΠ½ ЀарадСя опрСдСляСт количСства ΠΏΠ΅Ρ€Π²ΠΈΡ‡Π½Ρ‹Ρ… ΠΏΡ€ΠΎΠ΄ΡƒΠΊΡ‚ΠΎΠ², Π²Ρ‹Π΄Π΅Π»ΡΡŽΡ‰ΠΈΡ…ΡΡ Π½Π° элСктродах ΠΏΡ€ΠΈ элСктролизС. Π˜Ρ‚Π°ΠΊ, масса m вСщСства, Π²Ρ‹Π΄Π΅Π»ΠΈΠ²ΡˆΠ΅Π³ΠΎΡΡ Π½Π° элСктродС, прямо ΠΏΡ€ΠΎΠΏΠΎΡ€Ρ†ΠΈΠΎΠ½Π°Π»ΡŒΠ½Π° заряду Q, ΠΏΡ€ΠΎΡˆΠ΅Π΄ΡˆΠ΅ΠΌΡƒ Ρ‡Π΅Ρ€Π΅Π· элСктролит:

Π€ΠΎΡ€ΠΌΡƒΠ»Π° Π­Π»Π΅ΠΊΡ‚Ρ€ΠΎΠ»ΠΈΠ·

Π’Π΅Π»ΠΈΡ‡ΠΈΠ½Ρƒ k Π½Π°Π·Ρ‹Π²Π°ΡŽΡ‚ элСктрохимичСским эквивалСнтом. Он ΠΌΠΎΠΆΠ΅Ρ‚ Π±Ρ‹Ρ‚ΡŒ рассчитан ΠΏΠΎ Ρ„ΠΎΡ€ΠΌΡƒΠ»Π΅:

Π€ΠΎΡ€ΠΌΡƒΠ»Π° ЭлСктрохимичСский эквивалСнт

Π³Π΄Π΅: n – Π²Π°Π»Π΅Π½Ρ‚Π½ΠΎΡΡ‚ΡŒ вСщСства, NA – постоянная Авогадро, M – молярная масса вСщСства, Π΅ – элСмСнтарный заряд. Иногда Ρ‚Π°ΠΊΠΆΠ΅ вводят ΡΠ»Π΅Π΄ΡƒΡŽΡ‰Π΅Π΅ ΠΎΠ±ΠΎΠ·Π½Π°Ρ‡Π΅Π½ΠΈΠ΅ для постоянной ЀарадСя:

Π€ΠΎΡ€ΠΌΡƒΠ»Π° ΠŸΠΎΡΡ‚ΠΎΡΠ½Π½Π°Ρ ЀарадСя

Β 

ЭлСктричСский Ρ‚ΠΎΠΊ Π² Π³Π°Π·Π°Ρ… ΠΈ Π² Π²Π°ΠΊΡƒΡƒΠΌΠ΅

К оглавлСнию…

ЭлСктричСский Ρ‚ΠΎΠΊ Π² Π³Π°Π·Π°Ρ…

Π’ ΠΎΠ±Ρ‹Ρ‡Π½Ρ‹Ρ… условиях Π³Π°Π·Ρ‹ Π½Π΅ проводят элСктричСский Ρ‚ΠΎΠΊ. Π­Ρ‚ΠΎ ΠΎΠ±ΡŠΡΡΠ½ΡΠ΅Ρ‚ΡΡ элСктричСской Π½Π΅ΠΉΡ‚Ρ€Π°Π»ΡŒΠ½ΠΎΡΡ‚ΡŒΡŽ ΠΌΠΎΠ»Π΅ΠΊΡƒΠ» Π³Π°Π·ΠΎΠ² ΠΈ, ΡΠ»Π΅Π΄ΠΎΠ²Π°Ρ‚Π΅Π»ΡŒΠ½ΠΎ, отсутствиСм носитСлСй элСктричСских зарядов. Для Ρ‚ΠΎΠ³ΠΎ Ρ‡Ρ‚ΠΎΠ±Ρ‹ Π³Π°Π· стал ΠΏΡ€ΠΎΠ²ΠΎΠ΄Π½ΠΈΠΊΠΎΠΌ, ΠΎΡ‚ ΠΌΠΎΠ»Π΅ΠΊΡƒΠ» Π½Π΅ΠΎΠ±Ρ…ΠΎΠ΄ΠΈΠΌΠΎ ΠΎΡ‚ΠΎΡ€Π²Π°Ρ‚ΡŒ ΠΎΠ΄ΠΈΠ½ ΠΈΠ»ΠΈ нСсколько элСктронов. Π’ΠΎΠ³Π΄Π° появятся свободныС носитСля зарядов — элСктроны ΠΈ ΠΏΠΎΠ»ΠΎΠΆΠΈΡ‚Π΅Π»ΡŒΠ½Ρ‹Π΅ ΠΈΠΎΠ½Ρ‹. Π­Ρ‚ΠΎΡ‚ процСсс называСтся ΠΈΠΎΠ½ΠΈΠ·Π°Ρ†ΠΈΠ΅ΠΉ Π³Π°Π·ΠΎΠ².

Π˜ΠΎΠ½ΠΈΠ·ΠΈΡ€ΠΎΠ²Π°Ρ‚ΡŒ ΠΌΠΎΠ»Π΅ΠΊΡƒΠ»Ρ‹ Π³Π°Π·Π° ΠΌΠΎΠΆΠ½ΠΎ внСшним воздСйствиСм — ΠΈΠΎΠ½ΠΈΠ·Π°Ρ‚ΠΎΡ€ΠΎΠΌ. Π˜ΠΎΠ½ΠΈΠ·Π°Ρ‚ΠΎΡ€Π°ΠΌΠΈ ΠΌΠΎΠΆΠ΅Ρ‚ Π±Ρ‹Ρ‚ΡŒ: ΠΏΠΎΡ‚ΠΎΠΊ свСта, рСнтгСновскиС Π»ΡƒΡ‡ΠΈ, ΠΏΠΎΡ‚ΠΎΠΊ элСктронов ΠΈΠ»ΠΈ Ξ±-частиц. ΠœΠΎΠ»Π΅ΠΊΡƒΠ»Ρ‹ Π³Π°Π·Π° Ρ‚Π°ΠΊΠΆΠ΅ ΠΈΠΎΠ½ΠΈΠ·ΠΈΡ€ΡƒΡŽΡ‚ΡΡ ΠΏΡ€ΠΈ высокой Ρ‚Π΅ΠΌΠΏΠ΅Ρ€Π°Ρ‚ΡƒΡ€Π΅. Π˜ΠΎΠ½ΠΈΠ·Π°Ρ†ΠΈΡ ΠΏΡ€ΠΈΠ²ΠΎΠ΄ΠΈΡ‚ ΠΊ возникновСнию Π² Π³Π°Π·Π°Ρ… свободных носитСлСй зарядов — элСктронов, ΠΏΠΎΠ»ΠΎΠΆΠΈΡ‚Π΅Π»ΡŒΠ½Ρ‹Ρ… ΠΈΠΎΠ½ΠΎΠ², ΠΎΡ‚Ρ€ΠΈΡ†Π°Ρ‚Π΅Π»ΡŒΠ½Ρ‹Ρ… ΠΈΠΎΠ½ΠΎΠ² (элСктрон, объСдинившийся с Π½Π΅ΠΉΡ‚Ρ€Π°Π»ΡŒΠ½ΠΎΠΉ ΠΌΠΎΠ»Π΅ΠΊΡƒΠ»ΠΎΠΉ).

Если ΡΠΎΠ·Π΄Π°Ρ‚ΡŒ Π² пространствС, занятом ΠΈΠΎΠ½ΠΈΠ·ΠΈΡ€ΠΎΠ²Π°Π½Π½Ρ‹ΠΌ Π³Π°Π·ΠΎΠΌ, элСктричСскоС ΠΏΠΎΠ»Π΅, Ρ‚ΠΎ носитСли элСктричСских зарядов ΠΏΡ€ΠΈΠ΄ΡƒΡ‚ Π² упорядочСнноС Π΄Π²ΠΈΠΆΠ΅Π½ΠΈΠ΅ – Ρ‚Π°ΠΊ Π²ΠΎΠ·Π½ΠΈΠΊΠ°Π΅Ρ‚ элСктричСский Ρ‚ΠΎΠΊ Π² Π³Π°Π·Π°Ρ…. Если ΠΈΠΎΠ½ΠΈΠ·Π°Ρ‚ΠΎΡ€ пСрСстаСт Π΄Π΅ΠΉΡΡ‚Π²ΠΎΠ²Π°Ρ‚ΡŒ, Ρ‚ΠΎ Π³Π°Π· снова становится Π½Π΅ΠΉΡ‚Ρ€Π°Π»ΡŒΠ½Ρ‹ΠΌ, Ρ‚Π°ΠΊ ΠΊΠ°ΠΊ Π² Π½Π΅ΠΌ происходит рСкомбинация – ΠΎΠ±Ρ€Π°Π·ΠΎΠ²Π°Π½ΠΈΠ΅ Π½Π΅ΠΉΡ‚Ρ€Π°Π»ΡŒΠ½Ρ‹Ρ… Π°Ρ‚ΠΎΠΌΠΎΠ² ΠΈΠΎΠ½Π°ΠΌΠΈ ΠΈ элСктронами.

ЭлСктричСский Ρ‚ΠΎΠΊ Π² Π²Π°ΠΊΡƒΡƒΠΌΠ΅

Π’Π°ΠΊΡƒΡƒΠΌΠΎΠΌ называСтся такая ΡΡ‚Π΅ΠΏΠ΅Π½ΡŒ разрСТСния Π³Π°Π·Π°, ΠΏΡ€ΠΈ ΠΊΠΎΡ‚ΠΎΡ€ΠΎΠΌ ΠΌΠΎΠΆΠ½ΠΎ ΠΏΡ€Π΅Π½Π΅Π±Ρ€Π΅Ρ‡ΡŒ соударСниСм ΠΌΠ΅ΠΆΠ΄Ρƒ Π΅Π³ΠΎ ΠΌΠΎΠ»Π΅ΠΊΡƒΠ»Π°ΠΌΠΈ ΠΈ ΡΡ‡ΠΈΡ‚Π°Ρ‚ΡŒ, Ρ‡Ρ‚ΠΎ срСдняя Π΄Π»ΠΈΠ½Π° свободного ΠΏΡ€ΠΎΠ±Π΅Π³Π° ΠΏΡ€Π΅Π²Ρ‹ΡˆΠ°Π΅Ρ‚ Π»ΠΈΠ½Π΅ΠΉΠ½Ρ‹Π΅ Ρ€Π°Π·ΠΌΠ΅Ρ€Ρ‹ сосуда, Π² ΠΊΠΎΡ‚ΠΎΡ€ΠΎΠΌ Π³Π°Π· находится.

ЭлСктричСским Ρ‚ΠΎΠΊΠΎΠΌ Π² Π²Π°ΠΊΡƒΡƒΠΌΠ΅ Π½Π°Π·Ρ‹Π²Π°ΡŽΡ‚ ΠΏΡ€ΠΎΠ²ΠΎΠ΄ΠΈΠΌΠΎΡΡ‚ΡŒ мСТэлСктродного ΠΏΡ€ΠΎΠΌΠ΅ΠΆΡƒΡ‚ΠΊΠ° Π² состоянии Π²Π°ΠΊΡƒΡƒΠΌΠ°. ΠœΠΎΠ»Π΅ΠΊΡƒΠ» Π³Π°Π·Π° ΠΏΡ€ΠΈ этом ΡΡ‚ΠΎΠ»ΡŒ ΠΌΠ°Π»ΠΎ, Ρ‡Ρ‚ΠΎ процСссы ΠΈΡ… ΠΈΠΎΠ½ΠΈΠ·Π°Ρ†ΠΈΠΈ Π½Π΅ ΠΌΠΎΠ³ΡƒΡ‚ ΠΎΠ±Π΅ΡΠΏΠ΅Ρ‡ΠΈΡ‚ΡŒ Ρ‚Π°ΠΊΠΎΠ³ΠΎ числа элСктронов ΠΈ ΠΈΠΎΠ½ΠΎΠ², ΠΊΠΎΡ‚ΠΎΡ€Ρ‹Π΅ Π½Π΅ΠΎΠ±Ρ…ΠΎΠ΄ΠΈΠΌΡ‹ для ΠΈΠΎΠ½ΠΈΠ·Π°Ρ†ΠΈΠΈ. ΠŸΡ€ΠΎΠ²ΠΎΠ΄ΠΈΠΌΠΎΡΡ‚ΡŒ мСТэлСктродного ΠΏΡ€ΠΎΠΌΠ΅ΠΆΡƒΡ‚ΠΊΠ° Π² Π²Π°ΠΊΡƒΡƒΠΌΠ΅ ΠΌΠΎΠΆΠ΅Ρ‚ Π±Ρ‹Ρ‚ΡŒ обСспСчСна лишь с ΠΏΠΎΠΌΠΎΡ‰ΡŒΡŽ заряТСнных частиц, Π²ΠΎΠ·Π½ΠΈΠΊΡˆΠΈΡ… Π·Π° счСт эмиссионных явлСний Π½Π° элСктродах.

ΠžΡ‚Π²Π΅Ρ‚Ρ‹@Mail.Ru: Π² ΠΊΠ°ΠΊΠΎΠΌ Π½Π°ΠΏΡ€Π°Π²Π»Π΅Π½ΠΈΠΈ ΠΏΡ€ΠΎΡ‚Π΅ΠΊΠ°Π΅Ρ‚ Ρ‚ΠΎΠΊ Π² Ρ†Π΅ΠΏΠΈ

Π½Π°ΠΏΡ€Π°Π²Π»Π΅Π½ΠΈΠ΅ Ρ‚ΠΎΠΊΠ° — ΡƒΡΠ»ΠΎΠ²Π½ΠΎΡΡ‚ΡŒ, принятая для рисования схСм ΠΈ Π½Π΅ Π±ΠΎΠ»Π΅Π΅ Ρ‚ΠΎΠ³ΠΎ. ΠŸΡ€ΠΈΠ½ΡΡ‚ΠΎ Ρ€ΠΈΡΠΎΠ²Π°Ρ‚ΡŒ ΠΎΡ‚ + ΠΊ -. Если ΠΏΡ€ΠΎΠ²ΠΎΠ΄Π½ΠΈΠΊ — ΠΌΠ΅Ρ‚Π°Π» (ΠΏΡ€ΠΎΠ²ΠΎΠ΄, Π½Π°ΠΏΡ€ΠΈΠΌΠ΅Ρ€) — Ρ€Π΅Π°Π»ΡŒΠ½Ρ‹Π΅ носитСли — элСктроны — лСтят Π² ΠΎΠ±Ρ€Π°Ρ‚Π½ΡƒΡŽ сторону — ΠΊ ΠΏΠ»ΡŽΡΡƒ. Если Π½ΠΎΡΠΈΡ‚Π΅Π»ΡŒ ΠΆΠΈΠ΄ΠΊΠΎΡΡ‚ΡŒ с ΠΈΠΎΠ½Π°ΠΌΠΈ ΠΈΠ»ΠΈ ΠΈΠΎΠ½ΠΈΠ·ΠΈΡ€ΠΎΠ²Π°Π½Π½Ρ‹ΠΉ Π³Π°Π· — ΠΈΠΎΠ½Ρ‹ лСтят Π² ΠΎΠ±Π΅ стороны…

Π”Π°Π²Π½Π΅Π½ΡŒΠΊΠΎ принято ΡΡ‡ΠΈΡ‚Π°Ρ‚ΡŒ Π΄Π²ΠΈΠΆΠ΅Π½ΠΈΠ΅ Ρ‚ΠΎΠΊΠ° ΠΎΡ‚ плюса ΠΊ минусу, хотя Ρ€Π΅Π°Π»ΡŒΠ½ΠΎΠ΅ Π΄Π²ΠΈΠΆΠ΅Π½ΠΈΠ΅ носитСлСй заряда Π±Ρ‹Π²Π°Π΅Ρ‚ ΠΎΠ±Ρ€Π°Ρ‚Π½Ρ‹ΠΌ, Π² Π±ΠΎΠ»ΡŒΡˆΠΈΠ½ΡΡ‚Π²Π΅ случаСв.

ΠΎΡ‚ плюса ΠΊ минусу

принято ΠΎΡ‚ + ΠΊ -..Π½ΠΎ элСктрончики Π±Π΅Π³ΡƒΡ‚ Π½Π°ΠΎΠ±ΠΎΡ€ΠΎΡ‚… всС схСмы Ρ‡ΠΈΡ‚Π°ΡŽΡ‚ΡΡ ΠΎΡ‚ + ΠΊ -..

ΠŸΡ€ΠΈΠ½ΡΡ‚ΠΎ ΡΡ‡ΠΈΡ‚Π°Ρ‚ΡŒ, Ρ‡Ρ‚ΠΎ Π²ΠΎ ВНЕШНЕЙ Π¦Π•ΠŸΠ˜ Π½Π°ΠΏΡ€Π°Π²Π»Π΅Π½ΠΈΠ΅ Ρ‚ΠΎΠΊΠ° ΠΎΡ‚ ΠΏΠΎΠ»ΠΎΠΆΠΈΡ‚Π΅Π»Π½ΠΎΠ³ΠΎ полюса ΠΊ ΠΎΡ‚Ρ€ΠΈΡ†Π°Ρ‚Π΅Π»ΡŒΠ½ΠΎΠΌΡƒ. А Π²ΠΎ Π²Π½ΡƒΡ‚Ρ€Π΅Π½Π½Π΅ΠΉ, соотвСтствСнно, Π½Π°ΠΎΠ±ΠΎΡ€ΠΎΡ‚!!!

Π’ Π·Π°ΠΌΠΊΠ½ΡƒΡ‚ΠΎΠΉ элСктричСской Ρ†Π΅ΠΏΠΈ Ρ‚ΠΎΠΊ ΠΈΠ΄Π΅Ρ‚ ΠΎΡ‚ Ρ‚ΠΎΡ‡ΠΊΠΈ с большим ΠΏΠΎΡ‚Π΅Π½Ρ†ΠΈΠ°Π»ΠΎΠΌ Π² Ρ‚ΠΎΡ‡ΠΊΡƒ с мСньшим ΠΏΠΎΡ‚Π΅Π½Ρ†ΠΈΠ°Π»ΠΎΠΌ ΠΈ Π½ΠΈΠΊΠ°ΠΊΠΈΠ΅ + ΠΈΠ»ΠΈ — Ρ‚ΡƒΡ‚ Π½ΠΈ ΠΏΡ€ΠΈ Ρ‡Π΅ΠΌ.

ДвСсти Π»Π΅Ρ‚ Ρ‚ΠΎΠΌΡƒ Π½Π°Π·Π°Π΄ Π€Π°Ρ€Π°Π΄Π΅ΠΉ поставил ΠΎΠΏΡ‹Ρ‚, Π³Π΄Π΅ дСмонстрируСтся ΠΏΠΎΠ»ΡƒΡ‡Π΅Π½ΠΈΠ΅ Ρ‚ΠΎΠΊΠ° Π² Π³Π°Π»ΡŒΠ²Π°Π½ΠΎΠΌΠ΅Ρ‚Ρ€Π΅ ΠΏΡ€ΠΈ Π΄Π²ΠΈΠΆΠ΅Π½ΠΈΠΈ ΠΌΠ°Π³Π½ΠΈΡ‚Π° Π² ΠΊΠ°Ρ‚ΡƒΡˆΠΊΠ΅ индуктивности. БСгодня, осмысляя этот ΠΎΠΏΡ‹Ρ‚, приходится Π΄Π΅Π»Π°Ρ‚ΡŒ Π²Ρ‹Π²ΠΎΠ΄: соврСмСнная тСория Ρ‚ΠΎΠΊΠ° проводимости Π² мСталличСских ΠΏΡ€ΠΎΠ²ΠΎΠ΄Π½ΠΈΠΊΠ°Ρ… ΠΎΡˆΠΈΠ±ΠΎΡ‡Π½Π° ΠΏΠΎΡ‚ΠΎΠΌΡƒ, Ρ‡Ρ‚ΠΎ основой этой Ρ‚Π΅ΠΎΡ€ΠΈΠΈ являСтся Π΄Π²ΠΈΠΆΠ΅Π½ΠΈΠ΅ свободных элСктронов ΠΏΡ€ΠΈ Π½Π΅ΠΏΠΎΠ΄Π²ΠΈΠΆΠ½Ρ‹Ρ… ΠΈΠΎΠ½Π°Ρ…. ΠžΠΏΡ‹Ρ‚ ΠΆΠ΅ ЀарадСя дСмонстрируСт Π΄Π²ΠΈΠΆΠ΅Π½ΠΈΠ΅, ΠΊΠ°ΠΊ ΠΎΡ‚Ρ€ΠΈΡ†Π°Ρ‚Π΅Π»ΡŒΠ½Ρ‹Ρ…, Ρ‚Π°ΠΊ ΠΈ ΠΏΠΎΠ»ΠΎΠΆΠΈΡ‚Π΅Π»ΡŒΠ½Ρ‹Ρ… зарядов. А Ρ‚Π°ΠΊ ΠΊΠ°ΠΊ Π² ΠΏΡ€ΠΎΠ²ΠΎΠ΄Π½ΠΈΠΊΠ΅, ΠΊΡ€ΠΎΠΌΠ΅ ΠΏΠΎΠ΄Π²ΠΈΠΆΠ½Ρ‹Ρ… элСктронов ΠΈ Π½Π΅ΠΏΠΎΠ΄Π²ΠΈΠΆΠ½Ρ‹Ρ… ΠΈΠΎΠ½ΠΎΠ², Π΄Ρ€ΡƒΠ³ΠΈΡ… зарядов Π½Π΅Ρ‚, Ρ‚ΠΎ слСдуСт ΡΠ΄Π΅Π»Π°Ρ‚ΡŒ Π²Ρ‹Π²ΠΎΠ΄: Π€Π°Ρ€Π°Π΄Π΅ΠΉ двСсти Π»Π΅Ρ‚ Ρ‚ΠΎΠΌΡƒ Π½Π°Π·Π°Π΄ ΠΏΠΎΠ»ΡƒΡ‡ΠΈΠ», Π² качСствС Ρ‚ΠΎΠΊΠ° проводимости, элСктронно-ΠΏΠΎΠ·ΠΈΡ‚Ρ€ΠΎΠ½Π½Ρ‹ΠΉ Ρ‚ΠΎΠΊ, Ρ€Π°ΡΠΏΡ€ΠΎΡΡ‚Ρ€Π°Π½ΡΡŽΡ‰ΠΈΠΉΡΡ Π² эфирС Π²ΠΎΠΊΡ€ΡƒΠ³ ΠΏΡ€ΠΎΠ²ΠΎΠ΄Π½ΠΈΠΊΠΎΠ². <img src=»//otvet.imgsmail.ru/download/265070448_436948acc5ab4e150d7ce3c5e7f7cd81_800.gif» data-lsrc=»//otvet.imgsmail.ru/download/265070448_436948acc5ab4e150d7ce3c5e7f7cd81_120x120.gif» data-big=»1″>

ЭлСктричСский Ρ‚ΠΎΠΊ. Π˜ΡΡ‚ΠΎΡ‡Π½ΠΈΠΊΠΈ элСктричСского Ρ‚ΠΎΠΊΠ°

987. Π’ Π³Ρ€ΠΎΠ·Ρƒ ΠΌΠ΅ΠΆΠ΄Ρƒ Ρ‚ΡƒΡ‡Π°ΠΌΠΈ Π²ΠΎΠ·Π½ΠΈΠΊΠ°Π΅Ρ‚ молния. ЯвляСтся Π»ΠΈ ΠΎΠ½Π° элСктричСским Ρ‚ΠΎΠΊΠΎΠΌ? ЯвляСтся Π»ΠΈ элСктричСским Ρ‚ΠΎΠΊΠΎΠΌ молния, возникшая ΠΌΠ΅ΠΆΠ΄Ρƒ ΠΎΠ±Π»Π°ΠΊΠΎΠΌ ΠΈ Π—Π΅ΠΌΠ»Π΅ΠΉ?
Π”Π°, являСтся. Заряди ΠΈΠ· области с большими ΠΏΠΎΡ‚Π΅Π½Ρ†ΠΈΠ°Π»ΠΎΠΌ пСрСходят Π² ΠΎΠ±Π»Π°ΡΡ‚ΡŒ с мСньшим ΠΏΠΎΡ‚Π΅Π½Ρ†ΠΈΠ°Π»ΠΎΠΌ.

988. Π’ мСталличСском ΠΏΡ€ΠΎΠ²ΠΎΠ΄Π½ΠΈΠΊΠ΅, с ΠΏΠΎΠΌΠΎΡ‰ΡŒΡŽ ΠΊΠΎΡ‚ΠΎΡ€ΠΎΠ³ΠΎ разряТаСтся элСктроскоп, Π²ΠΎΠ·Π½ΠΈΠΊΠ°Π΅Ρ‚ элСктричСский Ρ‚ΠΎΠΊ. По ΠΏΡ€ΠΎΠ²ΠΎΠ΄Π½ΠΈΠΊΡƒ, ΡΠΎΠ΅Π΄ΠΈΠ½ΡΡŽΡ‰Π΅ΠΌΡƒ ΠΏΠΎΠ»ΡŽΡΡ‹ Π³Π°Π»ΡŒΠ²Π°Π½ΠΈΡ‡Π΅ΡΠΊΠΎΠ³ΠΎ элСмСнта, Ρ‚ΠΎΠΆΠ΅ ΠΈΠ΄Π΅Ρ‚ элСктричСский Ρ‚ΠΎΠΊ. Π•ΡΡ‚ΡŒ Π»ΠΈ Ρ€Π°Π·Π½ΠΈΡ†Π° ΠΌΠ΅ΠΆΠ΄Ρƒ этими Ρ‚ΠΎΠΊΠ°ΠΌΠΈ? Π’ Ρ‡Π΅ΠΌ состоит это Ρ€Π°Π·Π»ΠΈΡ‡ΠΈΠ΅?
Π Π°Π·Π½ΠΈΡ†Π° Ρ‚ΠΎΠ»ΡŒΠΊΠΎ Π²ΠΎ Π²Ρ€Π΅ΠΌΠ΅Π½ΠΈ протСкания Ρ‚ΠΎΠΊΠ°.

989. Π’ ΠΌΠΎΠΏΠ΅Π΄Π΅ ΠΎΡ‚ Π³Π΅Π½Π΅Ρ€Π°Ρ‚ΠΎΡ€Π° Ρ‚ΠΎΠΊΠ° ΠΊ Ρ„Π°Ρ€Π΅ ΠΏΡ€ΠΎΠ²Π΅Π΄Π΅Π½ Ρ‚ΠΎΠ»ΡŒΠΊΠΎ ΠΎΠ΄ΠΈΠ½ ΠΏΡ€ΠΎΠ²ΠΎΠ΄. ΠŸΠΎΡ‡Π΅ΠΌΡƒ Π½Π΅Ρ‚ Π²Ρ‚ΠΎΡ€ΠΎΠ³ΠΎ ΠΏΡ€ΠΎΠ²ΠΎΠ΄Π°?
Роль Π²Ρ‚ΠΎΡ€ΠΎΠ³ΠΎ ΠΏΡ€ΠΎΠ²ΠΎΠ΄Π° ΠΈΠ³Ρ€Π°Π΅Ρ‚ Ρ€Π°ΠΌΠ° ΠΌΠΎΠΏΠ΅Π΄Π°.

990. На рисункС 92 ΠΈΠ·ΠΎΠ±Ρ€Π°ΠΆΠ΅Π½Π° схСма элСктричСской Ρ†Π΅ΠΏΠΈ. НазовитС элСмСнты, ΠΈΠ· ΠΊΠΎΡ‚ΠΎΡ€Ρ‹Ρ… состоит данная элСктричСская Ρ†Π΅ΠΏΡŒ? Π§Ρ‚ΠΎ Π½ΡƒΠΆΠ½ΠΎ ΡΠ΄Π΅Π»Π°Ρ‚ΡŒ, Ρ‡Ρ‚ΠΎΠ±Ρ‹ Π»Π°ΠΌΠΏΠΎΡ‡ΠΊΠ° Π² Ρ†Π΅ΠΏΠΈ Π·Π°Π³ΠΎΡ€Π΅Π»Π°ΡΡŒ?
ΠšΠ»ΡŽΡ‡, Π»Π°ΠΌΠΏΠΎΡ‡ΠΊΠ°, источник Ρ‚ΠΎΠΊΠ°; Π½ΡƒΠΆΠ½ΠΎ Π·Π°ΠΌΠΊΠ½ΡƒΡ‚ΡŒ ΠΊΠ»ΡŽΡ‡.

991. Из ΠΊΠ°ΠΊΠΈΡ… элСмСнтов состоит Ρ†Π΅ΠΏΡŒ Π½Π° рисункС 93? Π‘ΡƒΠ΄Π΅Ρ‚ Π»ΠΈ ΠΈΠ΄Ρ‚ΠΈ Ρ‚ΠΎΠΊ Ρ‡Π΅Ρ€Π΅Π· сопротивлСниС R, Ссли ΠΊΠ»ΡŽΡ‡ΠΈ 1 ΠΈ 2 Ρ€Π°Π·ΠΎΠΌΠΊΠ½ΡƒΡ‚Ρ‹? Π‘ΡƒΠ΄Π΅Ρ‚ Π»ΠΈ ΠΈΠ΄Ρ‚ΠΈ Ρ‚ΠΎΠΊ ΠΈ Ρ‡Π΅Ρ€Π΅Π· ΠΊΠ°ΠΊΠΈΠ΅ элСмСнты Ρ†Π΅ΠΏΠΈ, Ссли Π·Π°ΠΌΠΊΠ½ΡƒΡ‚ΡŒ:
Π°) Ρ‚ΠΎΠ»ΡŒΠΊΠΎ ΠΊΠ»ΡŽΡ‡ 1; Β  Β  Β  Β  Π±) Ρ‚ΠΎΠ»ΡŒΠΊΠΎ ΠΊΠ»ΡŽΡ‡ 2; Β  Β  Β  Β  Π²) ΠΎΠ±Π° ΠΊΠ»ΡŽΡ‡Π°?

Π”Π²Π΅ Π»Π°ΠΌΠΏΡ‹; ΠΊΠ»ΡŽΡ‡ 1, ΠΊΠ»ΡŽΡ‡ 2; сопротивлСниС, источник Ρ‚ΠΎΠΊΠ°. Если ΠΎΠ±Π° ΠΊΠ»ΡŽΡ‡Π° Ρ€Π°Π·ΠΎΠΌΠΊΠ½ΡƒΡ‚Ρ‹, Ρ‚ΠΎΠΊ ΠΈΠ΄Ρ‚ΠΈ Π½Π΅ Π±ΡƒΠ΄Π΅Ρ‚. А) Π±ΡƒΠ΄Π΅Ρ‚, рСзистор R ΠΈ Π»Π°ΠΌΠΏΡ‹ 1; Π±) Π±ΡƒΠ΄Π΅Ρ‚, рСзистор R Ρ‚ Π»Π°ΠΌΠΏΠ° 2; Π²) Π±ΡƒΠ΄Π΅Ρ‚, Ρ‡Π΅Ρ€Π΅Π· всС элСмСнты.

992. Из ΠΊΠ°ΠΊΠΈΡ… элСмСнтов состоит Ρ†Π΅ΠΏΡŒ Π½Π° рисункС 94? Π‘ΡƒΠ΄Π΅Ρ‚ Π»ΠΈ ΠΈΠ΄Ρ‚ΠΈ Ρ‚ΠΎΠΊ Ρ‡Π΅Ρ€Π΅Π· Π»Π°ΠΌΠΏΠΎΡ‡ΠΊΠΈ, Ссли Π·Π°ΠΌΠΊΠ½ΡƒΡ‚ΡŒ:
Π°) Ρ‚ΠΎΠ»ΡŒΠΊΠΎ ΠΊΠ»ΡŽΡ‡ 1;
Π±) Ρ‚ΠΎΠ»ΡŒΠΊΠΎ ΠΊΠ»ΡŽΡ‡ 2;
Π²) ΠΎΠ±Π° ΠΊΠ»ΡŽΡ‡Π° ΠΎΠ΄Π½ΠΎΠ²Ρ€Π΅ΠΌΠ΅Π½Π½ΠΎ?
Π‘Ρ‚ΠΎΠΈΡ‚ Π»ΠΈ Π² Ρ‚Π°ΠΊΠΎΠΉ Ρ†Π΅ΠΏΠΈ ΠΈΠΌΠ΅Ρ‚ΡŒ Π΄Π²Π° ΠΊΠ»ΡŽΡ‡Π°?

Π”Π²Π΅ Π»Π°ΠΌΠΏΠΎΡ‡ΠΊΠΈ, ΠΊΠ»ΡŽΡ‡ 1, ΠΊΠ»ΡŽΡ‡ 2, источник Ρ‚ΠΎΠΊΠ°.
А) – Π½Π΅Ρ‚; Π±) – Π½Π΅Ρ‚ ; Π²) Π΄Π° , Π±ΡƒΠ΄Π΅Ρ‚. НС стоит Ρ…Π²Π°Ρ‚ΠΈΡ‚ ΠΎΠ΄Π½ΠΎΠ³ΠΎ ΠΊΠ»ΡŽΡ‡Π°.

993. Какова Ρ†Π΅Π½Π° дСлСния ΡˆΠΊΠ°Π»Ρ‹ Π²ΠΎΠ»ΡŒΡ‚ΠΌΠ΅Ρ‚Ρ€Π°, ΠΈΠ·ΠΎΠ±Ρ€Π°ΠΆΠ΅Π½Π½ΠΎΠ³ΠΎ Π½Π° рисункС 95?

1Π’

994. НачСртитС схСму Ρ†Π΅ΠΏΠΈ, содСрТащСй источник Ρ‚ΠΎΠΊΠ° ΠΈ Π΄Π²Π΅ Π»Π°ΠΌΠΏΠΎΡ‡ΠΊΠΈ, ΠΊΠ°ΠΆΠ΄ΡƒΡŽ ΠΈΠ· ΠΊΠΎΡ‚ΠΎΡ€Ρ‹Ρ… ΠΌΠΎΠΆΠ½ΠΎ Π²ΠΊΠ»ΡŽΡ‡Π°Ρ‚ΡŒ ΠΎΡ‚Π΄Π΅Π»ΡŒΠ½ΠΎ.

ЭлСктричСский Ρ‚ΠΎΠΊ. Π˜ΡΡ‚ΠΎΡ‡Π½ΠΈΠΊΠΈ элСктричСского Ρ‚ΠΎΠΊΠ°

Β 

995. Π’ ΡΠ»Π΅ΠΊΡ‚Ρ€ΠΈΡ‡Π΅ΡΠΊΡƒΡŽ Ρ†Π΅ΠΏΡŒ Π²ΠΊΠ»ΡŽΡ‡Π΅Π½ рСостат со ΡΠΊΠΎΠ»ΡŒΠ·ΡΡ‰ΠΈΠΌ ΠΊΠΎΠ½Ρ‚Π°ΠΊΡ‚ΠΎΠΌ (рис. 96). ΠŸΠΎΠΊΠ°ΠΆΠΈΡ‚Π΅ стрСлками, ΠΊΠ°ΠΊ ΠΈΠ΄Π΅Ρ‚ Ρ‚ΠΎΠΊ Π² Ρ†Π΅ΠΏΠΈ ΠΈ Π² рСостатС.

ЭлСктричСский Ρ‚ΠΎΠΊ. Π˜ΡΡ‚ΠΎΡ‡Π½ΠΈΠΊΠΈ элСктричСского Ρ‚ΠΎΠΊΠ°

Β 

996. Π§Π΅Ρ€Π΅Π· Π»Π°ΠΌΠΏΠΎΡ‡ΠΊΡƒ А (рис. 97) ΠΏΡ€ΠΎΡ‚Π΅ΠΊΠ°Π΅Ρ‚ Π² Ρ‚Π΅Ρ‡Π΅Π½ΠΈΠ΅ 5 ΠΌΠΈΠ½ 150 Кл элСктричСства, Π° Ρ‡Π΅Ρ€Π΅Π· Π»Π°ΠΌΠΏΠΎΡ‡ΠΊΡƒ Π’ β€” Π·Π° Ρ‚ΠΎ ΠΆΠ΅ врСмя 60 Кл. ΠžΠΏΡ€Π΅Π΄Π΅Π»ΠΈΡ‚ΡŒ силу Ρ‚ΠΎΠΊΠ° Π² Ρ‚ΠΎΠΉ ΠΈ Π΄Ρ€ΡƒΠ³ΠΎΠΉ Π»Π°ΠΌΠΏΠΎΡ‡ΠΊΠ΅.
Какова Π±ΡƒΠ΄Π΅Ρ‚ сила Ρ‚ΠΎΠΊΠ° Π² ΠΏΡ€ΠΎΠ²ΠΎΠ΄Π°Ρ… D ΠΈ Π‘?

ЭлСктричСский Ρ‚ΠΎΠΊ. Π˜ΡΡ‚ΠΎΡ‡Π½ΠΈΠΊΠΈ элСктричСского Ρ‚ΠΎΠΊΠ°

Β 

Π­Π»Π΅ΠΊΡ‚Ρ€ΠΈΠΊΠΈ! ΠžΠ±ΡŠΡΡΠ½ΠΈΡ‚Π΅ ΠΌΠ½Π΅ нСсколько элСмСнтарных ΠΏΠΎ Ρ„ΠΈΠ·ΠΈΠΊΠ΅ Π²Π΅Ρ‰Π΅ΠΉ.. . Описал Π²Π½ΡƒΡ‚Ρ€ΠΈ…

ЗСмля — это просто Ρ‚ΠΎΡ‡ΠΊΠ°, ΠΎΡ‚Π½ΠΎΡΠΈΡ‚Π΅Π»ΡŒΠ½ΠΎ ΠΊΠΎΡ‚ΠΎΡ€ΠΎΠΉ отсчитываСтся напряТСниС. ΠŸΠΎΡ‚ΠΎΠΌΡƒ Ρ‡Ρ‚ΠΎ напряТСниС ВБЕГДА измСряСтся ΠœΠ•Π–Π”Π£ двумя Ρ‚ΠΎΡ‡ΠΊΠ°ΠΌΠΈ (ΡƒΠ·Π»Π°ΠΌΠΈ) . Π’ΠΎΡ‚ Ρ‚ΠΎΡ‚ ΡƒΠ·Π΅Π», ΠΎΡ‚Π½ΠΎΡΠΈΡ‚Π΅Π»ΡŒΠ½ΠΎ ΠΊΠΎΡ‚ΠΎΡ€ΠΎΠ³ΠΎ ΡƒΠ΄ΠΎΠ±Π½ΠΎ ΠΈΠ·ΠΌΠ΅Ρ€ΡΡ‚ΡŒ напряТСниС для Π’Π‘Π•Π™ схСмы, ΠΈ называСтся «Π·Π΅ΠΌΠ»Ρ». НС зря ΠΆ синоним этого Π΄Π΅Π»Π° — «ΠΎΠ±Ρ‰ΠΈΠΉ». Π€Π°Π·Π° — это Π½Π΅ плюс. Π’ ΠΎΠ±Ρ‹Ρ‡Π½ΠΎΠΉ сСти Π²ΠΎΠΎΠ±Ρ‰Π΅ Π½Π΅Ρ‚ плюса ΠΈ минуса, ΠΏΠΎΡ‚ΠΎΠΌΡƒ Ρ‡Ρ‚ΠΎ Ρ‚Π°ΠΌ ΠΏΠ΅Ρ€Π΅ΠΌΠ΅Π½Π½Ρ‹ΠΉ Ρ‚ΠΎΠΊ. МоТно, с достаточной ΡΡ‚Π΅ΠΏΠ΅Π½ΡŒΡŽ правдоподобия, Π³ΠΎΠ²ΠΎΡ€ΠΈΡ‚ΡŒ, Ρ‡Ρ‚ΠΎ Ρ„Π°Π·Π° — это «Π½Π΅ зСмля». Иногда, ΠΊΠΎΠ³Π΄Π° ΠΈΠ½Π΄ΠΈΠ²ΠΈΠ΄ΡƒΠ°Π»ΡŒΠ½Ρ‹Π΅ «ΠΎΠ΄Π½ΠΎΡ„Π°Π·Π½Ρ‹Π΅» ΠΏΠΎΡ‚Ρ€Π΅Π±ΠΈΡ‚Π΅Π»ΠΈ — Π° это всС ΠΊΠ²Π°Ρ€Ρ‚ΠΈΡ€Π½Ρ‹Π΅ ΠΏΠΎΠ΄ΠΊΠ»ΡŽΡ‡Π΅Π½ΠΈΡ, ΠΊΡ€ΠΎΠΌΠ΅ ΠΌΠΎΡ‰Π½Ρ‹Ρ…, Ρ‚ΠΈΠΏΠ° элСктроплиты, — ΠΏΠΎΠ΄ΠΊΠ»ΡŽΡ‡Π°ΡŽΡ‚ΡΡ ΠΊ Ρ‚Ρ€Ρ‘Ρ…Ρ„Π°Π·Π½ΠΎΠΉ Π²Ρ…ΠΎΠ΄Π½ΠΎΠΉ магистрали ΠΏΠΎ схСмС «Π·Π²Π΅Π·Π΄Π°», Ρ„Π°Π·Π° — это Π΄Π΅ΠΉΡΡ‚Π²ΠΈΡ‚Π΅Π»ΡŒΠ½ΠΎ Ρ„Π°Π·Π° (ΠΎΠ΄Π½Π° ΠΈΠ· Ρ‚Ρ€Ρ‘Ρ…, Π² ΠΊΠ°ΠΆΠ΄ΠΎΠΉ ΠΊΠ²Π°Ρ€Ρ‚ΠΈΡ€Π΅ своя) , Π° «Π½ΠΎΠ»ΡŒ» — это Π½ΡƒΠ»Π΅Π²ΠΎΠΉ ΠΏΡ€ΠΎΠ²ΠΎΠ΄ Ρ‚Ρ€Ρ‘Ρ…Ρ„Π°Π·Π½ΠΎΠΉ сСти. Если ΠΆΠ΅ ΠΎΠ΄Π½ΠΎΡ„Π°Π·Π½Ρ‹Π΅ ΠΏΠΎΡ‚Ρ€Π΅Π±ΠΈΡ‚Π΅Π»ΠΈ ΠΏΠΎΠ΄ΠΊΠ»ΡŽΡ‡Π΅Π½Ρ‹ ΠΊ Ρ‚Ρ€Ρ‘Ρ…Ρ„Π°Π·Π½ΠΎΠΉ сСти ΠΏΠΎ схСмС «Ρ‚Ρ€Π΅ΡƒΠ³ΠΎΠ»ΡŒΠ½ΠΈΠΊ», Ρ‚ΠΎ ΠΎΠ±Π° ΠΏΡ€ΠΎΠ²ΠΎΠ΄Π° Π² Ρ€ΠΎΠ·Π΅Ρ‚ΠΊΠ΅ эквивалСнтны. Они ΠΎΠ±Π° Ρ„Π°Π·Π°. ΠŸΠΎΡΡ‚ΠΎΠΌΡƒ Ρ‘**Ρ‚, Ссли Π·Π° любой Π²Π·ΡΡ‚ΡŒΡΡ. ΠœΠΈΠ½ΡƒΡ — это «ΠΌΠΈΠ½ΡƒΡ» Π² ΠΊΠ°Π²Ρ‹Ρ‡ΠΊΠ°Ρ…. ΠŸΡ€ΠΎΡΡ‚ΠΎ Ρ‚Π΅Ρ€ΠΌΠΈΠ½ ΠΏΠΎΡˆΡ‘Π» с эпохи постоянного Ρ‚ΠΎΠΊΠ°, которая Π±Π»Π°Π³ΠΎΠΏΠΎΠ»ΡƒΡ‡Π½ΠΎ Π·Π°ΠΊΠΎΠ½Ρ‡ΠΈΠ»Π°ΡΡŒ Π΅Ρ‰Ρ‘ Π² 19 Π²Π΅ΠΊΠ΅. Π’Π°ΠΊ Ρ‡Ρ‚ΠΎ это просто Π²ΠΎΠ·Π²Ρ€Π°Ρ‚Π½Ρ‹ΠΉ ΠΏΡ€ΠΎΠ²ΠΎΠ΄ (Π΅Ρ‰Ρ‘ Ρ€Π°Π·: Π² сСтях ΠΏΠ΅Ρ€Π΅ΠΌΠ΅Π½Π½ΠΎΠ³ΠΎ Ρ‚ΠΎΠΊΠ° ΠΎΠ±Π° ΠΏΡ€ΠΎΠ²ΠΎΠ΄Π° эквивалСнтны) . ΠŸΡ€ΠΎΡΡ‚ΠΎ для прохоТдСния Ρ‚ΠΎΠΊΠ° Π½ΡƒΠΆΠ½Π° замкнутая Ρ†Π΅ΠΏΡŒ. Π’ΠΎΡ‚ «ΠΌΠΈΠ½ΡƒΡ» ΠΈ Π·Π°ΠΌΡ‹ΠΊΠ°Π΅Ρ‚ Ρ†Π΅ΠΏΡŒ — ΠΎΡ‚ «ΠΏΠ»ΡŽΡΠ°» Ρ‡Π΅Ρ€Π΅Π· Π½Π°Π³Ρ€ΡƒΠ·ΠΊΡƒ (ΠΈΠ»ΠΈ элСктрика, Сслион ΠΏΠΎΡ‡Π΅ΠΌΡƒ-Ρ‚ΠΎ Ρ€Π΅ΡˆΠΈΠ» Π·Π°ΠΌΠ΅Π½ΠΈΡ‚ΡŒ собой Π½Π°Π³Ρ€ΡƒΠ·ΠΊΡƒ… ) Π² «ΠΌΠΈΠ½ΡƒΡ». Если Π½Π΅Ρ‚ Π²ΠΎΠ·Π²Ρ€Π°Ρ‚Π½ΠΎΠ³ΠΎ ΠΏΡ€ΠΎΠ²ΠΎΠ΄Π°, ΠΊΠ°ΠΊ Π΅Π³ΠΎ Π½ΠΈ ΠΎΠ±Π·ΠΎΠ²ΠΈ, — Ρ‚ΠΎΠΊ Π½Π΅ ΠΏΠΎΠΉΠ΄Ρ‘Ρ‚. Но Ρ‚ΡƒΡ‚ Π½Π°Π΄ΠΎ ΡƒΡ‡ΠΈΡ‚Ρ‹Π²Π°Ρ‚ΡŒ, Ρ‡Ρ‚ΠΎ ΠΏΠ΅Ρ€Π΅ΠΌΠ΅Π½Π½Ρ‹ΠΉ Ρ‚ΠΎΠΊ ΠΌΠΎΠΆΠ΅Ρ‚ ΠΈΠ΄Ρ‚ΠΈ «Ρ‡Π΅Ρ€Π΅Π· Π²ΠΎΠ·Π΄ΡƒΡ…». ΠŸΠΎΡ‚ΠΎΠΌΡƒ Ρ‡Ρ‚ΠΎ элСктрик, стоящий Π½Π° ΠΏΠΎΠ»Ρƒ ΠΈ Π΄Π°ΠΆΠ΅ Π½Π° Ρ€Π΅Π·ΠΈΠ½ΠΎΠ²ΠΎΠΌ ΠΊΠΎΠ²Ρ€ΠΈΠΊΠ΅, — это всё Ρ€Π°Π²Π½ΠΎ Ρ‘ΠΌΠΊΠΎΡΡ‚ΡŒ. Π’ΠΎ Π΅ΡΡ‚ΡŒ Ρ‡Π΅Ρ€Π΅Π· Π½Π΅Π³ΠΎ ΠΌΠΎΠΆΠ΅Ρ‚ ΠΈΠ΄Ρ‚ΠΈ Смкостной Ρ‚ΠΎΠΊ (сущая Π²ΡˆΠΈΠ²ΠΎΡ‚Π°, Π½ΠΎ всё ΠΆΠ΅, всё ΠΆΠ΅…) . ======================== «ΠœΠΈΠ½ΡƒΡ создаётся Π½Π° элСктростанции Π½Π°ΠΊΠ°Ρ‡ΠΈΠ²Π°Π½ΠΈΠ΅ΠΌ ΠΏΡ€ΠΎΠ²ΠΎΠ΄Π° элСктронами ΠΏΠΎΠ΄ Π΄Π°Π²Π»Π΅Π½ΠΈΠ΅ΠΌ» Π£ΠΏΠ°Π» ΠΏΠΎΠ΄ стол….

ЗСмля -это Π·Π°Π·Π΅ΠΌΠ»Π΅Π½ΠΈΠ΅. ΠšΠΎΡ€ΠΏΡƒΡ ΠΏΡ€ΠΈΠ±ΠΎΡ€Π° Π΄ΠΎΠ»ΠΆΠ΅Π½ Π±Ρ‹Ρ‚ΡŒ Π·Π°Π·Π΅ΠΌΠ»Π΅Π½, Ρ‡Ρ‚ΠΎΠ± всС Π½Π°Π²ΠΎΠ΄ΠΊΠΈ ΡƒΡ…ΠΎΠ΄ΠΈΠ»ΠΈ Π² зСмлю.. . Π’ Ρ€ΠΎΠ·Π΅Ρ‚ΠΊΠ΅ ΠΏΠ΅Ρ€Π΅ΠΌΠ΅Π½Π½Ρ‹ΠΉ Ρ‚ΠΎΠΊ. поэтому + ΠΈ — Ρ‚Π°ΠΌ ΠΌΠ΅Π½ΡΡŽΡ‚ΡΡ 50 ΠΈΠ»ΠΈ 60 Ρ€Π°Π· Π² сСкунду (50 -60 Π³Π΅Ρ€Ρ† частота ΠΏΡ€ΠΎΠΌΡ‹ΡˆΠ»Π΅Π½Π½ΠΎΠΉ сСти) ( Ρ‚ΠΎ Π½Π° Ρ„Π°Π·Π΅ + Π½Π° Π½ΡƒΠ»Π΅ — ,Ρ‚ΠΎ Π½Π° ΠΎΠ±ΠΎΡ€ΠΎΡ‚, измСнятся ΠΏΠΎ ΡΠΈΠ½ΡƒΡΠΎΠΈΠ΄Π°Π»ΡŒΠ½ΠΎΠΌΡƒ Π·Π°ΠΊΠΎΠ½Ρƒ).. . НаправлСниС Ρ‚ΠΎΠΊΠ° принято ΡΡ‡ΠΈΡ‚Π°Ρ‚ΡŒ ΠΎΡ‚ + ΠΊ — ,хотя Π½Π° самом Π΄Π΅Π»Π΅ -Ρ‚ΠΎΠΊ Π² ΠΌΠ΅Ρ‚Π°Π»Π»Π΅-это Π΄Π²ΠΈΠΆΠ΅Π½ΠΈΠ΅ элСктронов, Π° ΠΎΠ½ΠΈ двиТутся ΠΎΡ‚ — ΠΊ +.. Если Π΄ΠΎΡ‚Ρ€ΠΎΠ½ΡƒΡ‚ΡŒΡΡ Π΄ΠΎ Ρ„Π°Π·Ρ‹ Ρ‚ΠΎ Ρ‚ΠΎΠΊΠΎΠΌ Π΄ΠΎΠ»Π±Π°Π½Π΅Ρ‚, Ссли Π½ΠΎΠ³ΠΈ соСдинСны с Π·Π΅ΠΌΠ»Π΅ΠΉ. Ρ‚ΠΎ Π΅ΡΡ‚ΡŒ Ссли Ρ…ΠΎΡ€ΠΎΡˆΠ°Ρ изоляция ( толстый Ρ€Π΅Π·ΠΈΠ½ΠΎΠ²Ρ‹ΠΉ ΠΊΠΎΠ²Ρ€ΠΈΠΊ ΠΈ Ρ‚Ρ‹ Π΄ΠΎΡ‚Ρ€ΠΎΠ½Π΅ΡˆΡŒΡΡ Π΄ΠΎ Ρ„Π°Π·Ρ‹ Π½Π΅ сильно Π΄ΠΎΠ»Π±Π°Π½Π΅Ρ‚, Π½ΠΎ Π΄ΠΎΠ»Π±Π°Π½Π΅Ρ‚, Ρ‚Π°ΠΊ ΠΊΠ°ΠΊ образуСтся кондСнсатор ΠΌΠ΅ΠΆΠ΄Ρƒ Ρ‚ΠΎΠ±ΠΎΠΉ ΠΈ Π·Π΅ΠΌΠ»Π΅ΠΉ ΠΈ поэтому Ρ‡ΡƒΡ‚ΡŒ -Ρ‡ΡƒΡ‚ΡŒ ΠΌΠΎΠΆΠ΅Ρ‚ Π΄ΠΎΠ»Π±Π°Π½ΡƒΡ‚ΡŒ.. . А Ссли Ρ…ΠΎΡ€ΠΎΡˆΠΎ Π±ΡƒΠ΄Π΅ΡˆΡŒ соСдинСн с Π·Π΅ΠΌΠ»Π΅ΠΉ ( Π½Π°ΠΏΡ€ΠΈΠΌΠ΅Ρ€, ΠΊΠ°ΠΊ Ρ‚Ρ‹ написал -Π½ΠΎΠ³ΠΈ Π² Π²ΠΎΠ΄Ρƒ) , Ρ‚ΠΎ ΡƒΠ±ΡŒΠ΅Ρ‚ ΠΏΠΎ ΠΏΠΎΠ»Π½ΠΎΠΉ ΠΏΡ€ΠΎΠ³Ρ€Π°ΠΌΠΌΠ΅.. . Для Ρ‡Π΅Π³ΠΎ Π½ΡƒΠΆΠ΅Π½ минус. . Ρ‡Ρ‚ΠΎΠ± Π±Ρ‹Π» Ρ‚ΠΎΠΊ Π½Π΅ΠΎΠ±Ρ…ΠΎΠ΄ΠΈΠΌΠΎ ΡΠΎΠ·Π΄Π°Ρ‚ΡŒ Ρ€Π°Π·Π½ΠΎΡΡ‚ΡŒ ΠΏΠΎΡ‚Π΅Π½Ρ†ΠΈΠ°Π»ΠΎΠ²- Π½Π° ΠΎΠ΄Π½ΠΎΠΌ полюсС ΠΈΠ·Π±Ρ‹Ρ‚ΠΎΠΊ элСктронов -это минус, Π½Π° Π΄Ρ€ΡƒΠ³ΠΎΠΌ нСдостаток -это + .ΠΈ Ссли Π·Π°ΠΌΠΊΠ½ΡƒΡ‚ΡŒ Ρ†Π΅ΠΏΡŒ, Ρ‚ΠΎ элСктроны ΠΏΠΎΠ±Π΅Π³ΡƒΡ‚ ΠΎΡ‚ — ΠΊ +..Π’ΠΎ Π΅ΡΡ‚ΡŒ ΠΏΠΎ ΠΎΠ΄Π½ΠΎΠΌΡƒ ΠΏΡ€ΠΎΠ²ΠΎΠ΄Ρƒ приходят элСктроны ΠΊ ΠΏΡ€ΠΈΠ±ΠΎΡ€Ρƒ, ΠΏΠΎ Π΄Ρ€ΡƒΠ³ΠΎΠΌΡƒ уходят ΠΎΠ±Ρ€Π°Ρ‚Π½ΠΎ Π² источник.

ΠžΡ‡Ρ‡Π΅Π½ΡŒ ΠΌΠ½ΠΎΠ³ΠΎ вопросов! Π€Π°Π·Π°-это Π½Π΅ плюс. Π­Ρ‚ΠΎ Ρ‚ΠΎΡ‡Π½ΠΎ.

Π€Π°Π·Π° Π² элСктротСхникС (Ρ€Π°Π·Π³ΠΎΠ²ΠΎΡ€Π½ΠΎΠ΅) β€” ΠΏΡ€ΠΎΠ²ΠΎΠ΄, нСсущий напряТСниС плюс-ΠΏΠΎΠ»ΠΎΠΆΠΈΡ‚Π΅Π»ΡŒΠ½Ρ‹ΠΉ заряд минус-ΠΎΡ‚Ρ€ΠΈΡ†Π°Ρ‚Π΅Π»ΡŒΠ½Ρ‹ΠΉ заряд ноль-Π½Π΅Ρ‚Ρƒ заряда Π² Ρ€ΠΎΠ·Π΅Ρ‚ΠΊΠ΅ 220 Π½Π° Ρ„Π°Π·Ρƒ подаСтся плюс ΠΈ минус ΠΏΠΎΠΎΡ‡Π΅Ρ€Π΅Π΄Π½ΠΎ с частотой 50 ΠΊΠΎΠ»Π΅Π±Π°Π½ΠΈΠΉ Π² сСкунду. ΠΌΠΎΠΆΠ½ΠΎ ΠΏΡ€Π΅Π΄ΡΡ‚Π°Π²Π»ΡΡ‚ΡŒ Ρ‚ΠΎΠΊ ΠΊΠ°ΠΊ Π²ΠΎΠ΄Ρƒ Ρ‚Π΅ΠΊΡƒΡ‰ΡƒΡŽ ΠΏΠΎ Ρ‚Ρ€ΡƒΠ±Π΅ ΠΏΠΎ Π½Π°Ρ‡Π°Π»Ρƒ ΠΌΠΎΠΆΠ΅Ρ‚ Π±ΡƒΠ΄Π΅Ρ‚ ΠΏΡ€ΠΎΡ‰Π΅. Π½Π°ΠΏΡ€ΠΈΠΌΠ΅Ρ€ Π·Π°ΡΡ‚Ρ€ΡΠ²ΡˆΠΈΠΉ ΠΏΠΎΠΏΠ΅Ρ€Π΅ΠΊ Ρ‚Ρ€ΡƒΠ±Ρ‹ ΠΏΡ€Π΅Π·Π΅Ρ€Π²Π°Ρ‚ΠΈΠ² Π±ΡƒΠ΄Π΅Ρ‚ ΠΊΠ°ΠΊ кондСнсатор ΠΊΠΎΡ‚ΠΎΡ€Ρ‹ΠΉ Π½Π΅ пропускаСт постоянный Ρ‚ΠΎΠΊ (Ρ‚Π΅Ρ‡Π΅Π½ΠΈΠ΅ Π²ΠΎΠ΄Ρ‹ Π² ΠΎΠ΄Π½ΠΎΠΌ Π½Π°ΠΏΡ€Π°Π²Π»Π΅Π½ΠΈΠΈ)

Начнём ΠΏΠΎ порядку. ЗСмля — Π·Π°Ρ‰ΠΈΡ‚Π½Ρ‹ΠΉ Ρ‚Ρ€Π΅Ρ‚ΠΈΠΉ ΠΏΡ€ΠΎΠ²ΠΎΠ΄, соСдинённый с корпусом ΠΏΡ€ΠΈΠ±ΠΎΡ€Π°. ΠŸΡ€ΠΈΡΠΎΠ΅Π΄ΠΈΠ½ΡΠ΅Ρ‚ΡΡ ΠΊ ΠΊΠΎΠ½Ρ‚ΡƒΡ€Ρƒ зазСмлСния. ДСлаСтся для Ρ‚ΠΎΠ³ΠΎ, Ρ‡Ρ‚ΠΎΠ±Ρ‹ ΠΏΡ€ΠΈ ΠΏΠΎΠΏΠ°Π΄Π°Π½ΠΈΠΈ напряТСния Π½Π° корпус ΠΏΡ€ΠΎΠΈΠ·ΠΎΡˆΠ»ΠΎ ΠΊΠΎΡ€ΠΎΡ‚ΠΊΠΎΠ΅ Π·Π°ΠΌΡ‹ΠΊΠ°Π½ΠΈΠ΅ ΠΈ сработала Π·Π°Ρ‰ΠΈΡ‚Π° — ΠΏΡ€Π΅Π΄ΠΎΡ…Ρ€Π°Π½ΠΈΡ‚Π΅Π»ΡŒ, Π°Π²Ρ‚ΠΎΠΌΠ°Ρ‚ ΠΈ Ρ‚. ΠΏ. ΠΈ ΠΈΡΠΊΠ»ΡŽΡ‡Π°Π»Π°ΡΡŒ Π²ΠΎΠ·ΠΌΠΎΠΆΠ½ΠΎΡΡ‚ΡŒ пораТСния элСктричСским Ρ‚ΠΎΠΊΠΎΠΌ. Π€Π°Π·Π° — это Ρ„Π°Π·Π°. «ΠŸΠ»ΡŽΡ» ΠΈ «ΠΌΠΈΠ½ΡƒΡ» Π΅ΡΡ‚ΡŒ Ρƒ постоянного напряТСния, Ρƒ ΠΏΠ΅Ρ€Π΅ΠΌΠ΅Π½Π½ΠΎΠ³ΠΎ — Ρ„Π°Π·Π° ΠΈ ноль. ΠœΠ΅ΠΆΠ΄Ρƒ Ρ„Π°Π·ΠΎΠΉ ΠΈ Π½ΡƒΠ»Ρ‘ΠΌ напряТСниС — Ρ€Π°Π·Π½ΠΈΡ†Π° ΠΏΠΎΡ‚Π΅Π½Ρ†ΠΈΠ°Π»ΠΎΠ² — Ρ‚Π΅ самыС 220 Π²ΠΎΠ»ΡŒΡ‚. БчитаСтся, Ρ‡Ρ‚ΠΎ постоянный Ρ‚ΠΎΠΊ Ρ‚Π΅Ρ‡Ρ‘Ρ‚ ΠΎΡ‚ плюса ΠΊ минусу. ΠŸΡ€ΠΈ касании Ρ„Π°Π·Π½ΠΎΠ³ΠΎ ΠΏΡ€ΠΎΠ²ΠΎΠ΄Π° всё зависит ΠΎΡ‚ сопротивлСния Ρ‚Π΅Π»Π°. Π’ΠΎΠΊ ΠΏΠΎΠΉΠ΄Ρ‘Ρ‚ ΠΏΠΎ ΠΏΡƒΡ‚ΠΈ наимСньшСго сопротивлСния, ΠΎΠ±Ρ‹Ρ‡Π½ΠΎ Ρ‚ΠΎΠΊ ΠΏΡ€ΠΎΡ…ΠΎΠ΄ΠΈΡ‚ Ρ‡Π΅Ρ€Π΅Π· Ρ‚Π΅Π»ΠΎ Π² зСмлю. Если ΡΡ‚ΠΎΡΡ‚ΡŒ Π½Π° диэлСктричСском ΠΊΠΎΠ²Ρ€ΠΈΠΊΠ΅, ΡƒΠ΄Π°Ρ€Π° ΠΌΠΎΠΆΠ½ΠΎ ΠΈΠ·Π±Π΅ΠΆΠ°Ρ‚ΡŒ. Π§Ρ‚ΠΎΠ±Ρ‹ Π·Π°ΡΡ‚Π°Π²ΠΈΡ‚ΡŒ Ρ€Π°Π±ΠΎΡ‚Π°Ρ‚ΡŒ элСктроприбор, Π½Π°Π΄ΠΎ ΡΠΎΠ·Π΄Π°Ρ‚ΡŒ напряТСниС — Ρ€Π°Π·Π½ΠΎΡΡ‚ΡŒ ΠΏΠΎΡ‚Π΅Π½Ρ†ΠΈΠ°Π»ΠΎΠ². Для этого ΡΡƒΡ‰Π΅ΡΡ‚Π²ΡƒΡŽΡ‚ элСктростанции, ΠΏΠΎΠ»ΡƒΡ‡Π΅Π½Π½Ρ‹ΠΉ Ρ‚ΠΎΠΊ ΠΏΠΎ Π΄Π²ΡƒΠΌ ΠΏΡ€ΠΎΠ²ΠΎΠ΄Π°ΠΌ — (Ρ„Π°Π·Π° ΠΈ ноль) — поступаСт Π² Ρ€ΠΎΠ·Π΅Ρ‚ΠΊΡƒ. Если Π½Π΅ Π±ΡƒΠ΄Π΅Ρ‚ Π²Ρ‚ΠΎΡ€ΠΎΠ³ΠΎ ΠΏΡ€ΠΎΠ²ΠΎΠ΄Π°, Ρ‚ΠΎ Π½Π΅ Π±ΡƒΠ΄Π΅Ρ‚ ΠΈ Ρ‚ΠΎΠΉ разности ΠΏΠΎΡ‚Π΅Π½Ρ†ΠΈΠ°Π»ΠΎΠ².. . Как-Ρ‚ΠΎ Ρ‚Π°ΠΊ…

Π€Π°Π·Π° — это ΠΏΡ€ΠΎΠ²ΠΎΠ΄, ΠΏΠΎ ΠΊΠΎΡ‚ΠΎΡ€ΠΎΠΌΡƒ ΠΏΡ€ΠΈΡ…ΠΎΠ΄ΠΈΡ‚ с элСктростанции ΠΏΠ΅Ρ€Π΅ΠΌΠ΅Π½Π½Ρ‹ΠΉ Ρ‚ΠΎΠΊ (мСняСт плюс Π½Π° минус ΠΈ Π½Π°ΠΎΠ±ΠΎΡ€ΠΎΡ‚ 100 Ρ€Π°Π· Π² сСкунду) . Ноль (ΠΈΠ½ΠΎΠ³Π΄Π° говорят «Π·Π΅ΠΌΠ»Ρ») — это ΠΏΡ€ΠΎΠ²ΠΎΠ΄, ΠΏΠΎ ΠΊΠΎΡ‚ΠΎΡ€ΠΎΠΌΡƒ Ρ‚ΠΎΠΊ возвращаСтся Π½Π° ΡΠ»Π΅ΠΊΡ‚Ρ€ΠΎΡΡ‚Π°Π½Ρ†ΠΈΡŽ. Π”Π°, Ρ„Π°Π·Π° Π±ΡŒΡ‘Ρ‚ Ρ‚ΠΎΠΊΠΎΠΌ, Ρ‚ΠΎΠ»ΡŒΠΊΠΎ Ссли другая Ρ‡Π°ΡΡ‚ΡŒ Ρ‚Π΅Π»Π° «Π·Π°Π½ΡƒΠ»Π΅Π½Π°» ΠΈΠ»ΠΈ «Π·Π°Π·Π΅ΠΌΠ»Π΅Π½Π°». ΠœΠΈΠ½ΡƒΡ создаётся Π½Π° элСктростанции Π½Π°ΠΊΠ°Ρ‡ΠΈΠ²Π°Π½ΠΈΠ΅ΠΌ ΠΏΡ€ΠΎΠ²ΠΎΠ΄Π° элСктронами ΠΏΠΎΠ΄ Π΄Π°Π²Π»Π΅Π½ΠΈΠ΅ΠΌ. Π’Π°ΠΌ ΠΆΠ΅ Π½Π° Π΄Ρ€ΡƒΠ³ΠΎΠΌ ΠΏΡ€ΠΎΠ²ΠΎΠ΄Π΅ создаётся «ΠΏΠ»ΡŽΡ», ΠΏΡƒΡ‚Ρ‘ΠΌ отсасывания элСктронов со Π²Ρ‚ΠΎΡ€ΠΎΠ³ΠΎ ΠΏΡ€ΠΎΠ²ΠΎΠ΄Π°. ΠŸΡ€ΠΎΠ²ΠΎΠ΄Π° ΠΊΠ°ΠΊ Π±Ρ‹ ΠΌΠ΅Π½ΡΡŽΡ‚ΡΡ мСстами 100 Ρ€Π°Π· Π² сСкунду πŸ™‚

зСмля слуТит для Π·Π°Ρ‰ΠΈΡ‚Ρ‹ ΠΎΡ‚ пораТСния эл. Ρ‚ΠΎΠΊΠΎΠΌ. Ρ„Π°Π·Π° это Ρ„Π°Π·Π°. ΠΎΠ½Π° Π½Π΅ ΠΌΠΎΠΆΠ΅Ρ‚ ΡΡ‡ΠΈΡ‚Π°Ρ‚ΡŒΡΡ Π½ΠΈ плюсом, Π½ΠΈ минусом. (Ρ‚ΠΎΠΊ ΠΏΠ΅Ρ€Π΅ΠΌΠ΅Π½Π½Ρ‹ΠΉ, мСняСт своС Π½Π°ΠΏΡ€Π°Π²Π»Π΅Π½ΠΈΠ΅ 50 Ρ€Π°Π· Π² сСкунду, Ρ‚. Π΅. частота 50 Π“Π΅Ρ€Ρ†. ) Π΄ΠΎ Ρ„Π°Π·Ρ‹ Π΄ΠΎΡ‚Ρ€Π°Π³ΠΈΠ²Π°Ρ‚ΡŒΡΡ нСльзя, навСрняка Π΄ΠΎΠ»Π±Π°Π½Π΅Ρ‚. ΠΈ Π² Ρ€ΠΎΠ·Π΅Ρ‚ΠΊΠ΅, Ρ‡Ρ‚ΠΎ Ρƒ тСбя Π΄ΠΎΠΌΠ° стоит массы Π½Π΅Ρ‚. Ρ‚Π°ΠΌ Π΅ΡΡ‚ΡŒ Ρ„Π°Π·Π°, ноль ΠΈ зСмля. ΠΎΠ±Ρ‹Ρ‡Π½ΠΎ Ρ„Π°Π·Π° бСлая, ноль Π³ΠΎΠ»ΡƒΠ±ΠΎΠΉ, зСмля полосатая -ТСлтозСлСная. ,

Π€Π°Π·Π° — это Π½Π΅ плюс ΠΈ Π½Π΅ минус, ΠΈΠ±ΠΎ Π² сСти ΠΏΠ΅Ρ€Π΅ΠΌΠ΅Π½Π½Ρ‹ΠΉ Ρ‚ΠΎΠΊ. ΠŸΡ€ΠΎΡΡ‚ΠΎ Ρ„Π°Π·Π° — это Π½Π΅ связанный с Π·Π΅ΠΌΠ»Π΅ΠΉ ΠΏΡ€ΠΎΠ²ΠΎΠ΄Π½ΠΈΠΊ. Ноль (Ρ‚ΠΎ Ρ‡Ρ‚ΠΎ Π²Ρ‹ Π½Π°Π·Π²Π°Π»ΠΈ «ΠΌΠΈΠ½ΡƒΡ») — это ΠΏΡ€ΠΎΠ²ΠΎΠ΄Π½ΠΈΠΊ, Π² ΠΊΠΎΡ‚ΠΎΡ€ΠΎΠΌ напряТСниС ΠžΠ’ΠΠžΠ‘Π˜Π’Π•Π›Π¬ΠΠž Π—Π•ΠœΠ›Π˜ принято ΡΡ‡ΠΈΡ‚Π°Ρ‚ΡŒ Ρ€Π°Π²Π½Ρ‹ΠΌ Π½ΡƒΠ»ΡŽ. «Π—Смля», Ρ‚ΠΎ Π΅ΡΡ‚ΡŒ Π·Π°Π·Π΅ΠΌΠ»Π΅Π½ΠΈΠ΅ — это ΠΏΡ€ΠΎΠ²ΠΎΠ΄, Π½Π°ΠΏΡ€ΡΠΌΡƒΡŽ Π·Π°Π·Π΅ΠΌΠ»Π΅Π½Π½Ρ‹ΠΉ, Ρ‚ΠΎ Π΅ΡΡ‚ΡŒ Π³Ρ€ΡƒΠ±ΠΎ говоря Π²ΠΊΠΎΠΏΠ°Π½Π½Ρ‹ΠΉ Π½Π° Π½Π΅ΠΊΠΎΡ‚ΠΎΡ€ΡƒΡŽ Π³Π»ΡƒΠ±ΠΈΠ½Ρƒ Π² зСмлю. На Π½Π΅ΠΌ ΠΏΠΎΡ‚Π΅Π½Ρ†ΠΈΠ°Π» ΠΎΡ‚Π½ΠΎΡΠΈΡ‚Π΅Π»ΡŒΠ½ΠΎ Π·Π΅ΠΌΠ»ΠΈ ΠžΠ‘Π―Π—ΠΠ Π±Ρ‹Ρ‚ΡŒ Π½ΡƒΠ»Π΅Π²Ρ‹ΠΌ. Π—Π°Ρ‡Π΅ΠΌ Π½ΡƒΠΆΠ΅Π½ ноль ΠΈ зСмля? Ноль — это Ρ‚ΠΎΠΊΠΎΠ²Π΅Π΄ΡƒΡ‰ΠΈΠΉ ΠΏΡ€ΠΎΠ²ΠΎΠ΄, ΠΎΠ½ Π½Π΅ всСгда Π½Π°Π³Π»ΡƒΡ…ΠΎ связан с Π·Π΅ΠΌΠ»Π΅ΠΉ, ΠΈ это Π½ΠΎΡ€ΠΌΠ°Π»ΡŒΠ½ΠΎ. НапримСр, ΠΏΡŒΡΠ½Ρ‹ΠΉ элСктрик ΠΌΠΎΠΆΠ΅Ρ‚ Π² ΠΏΠΎΠ΄Π²Π°Π»Π΅ вашСго Π΄ΠΎΠΌΠ° ΠΏΠ΅Ρ€Π΅ΠΏΡƒΡ‚Π°Ρ‚ΡŒ ΠΏΡ€ΠΎΠ²ΠΎΠ΄Π°, ΠΈ ΠΏΡƒΡΡ‚ΠΈΡ‚ΡŒ ΠΏΠΎ Π½ΡƒΠ»Π΅Π²ΠΎΠΌΡƒ ΠΏΡ€ΠΎΠ²ΠΎΠ΄Ρƒ Ρ„Π°Π·Ρƒ Π° ΠΏΠΎ Ρ„Π°Π·Π½ΠΎΠΌΡƒ ноль. Π’Π°ΡˆΠΈ ΠΏΡ€ΠΈΠ±ΠΎΡ€Ρ‹ этого скорСС всСго Π½Π΅ замСтят. Но располоТСниС нуля ΠΈ Ρ„Π°Π·Ρ‹ Π² Ρ€ΠΎΠ·Π΅Ρ‚ΠΊΠ°Ρ… измСнится. Π—Π°Π·Π΅ΠΌΠ»Π΅Π½ΠΈΠ΅ ΠΆΠ΅ всСгда ΡΡ‚Π°Π±ΠΈΠ»ΡŒΠ½ΠΎ сидит Π½Π° Π·Π΅ΠΌΠ»Π΅. Π­Ρ‚ΠΈΠΌ ΠΎΠ½ΠΎ ΠΈ Ρ†Π΅Π½Π½ΠΎ. Оно страхуСт ΠΏΡ€ΠΈΠ±ΠΎΡ€ ΠΎΡ‚ всяких нСприятностСй.

Π”ΠΎΠ±Π°Π²ΠΈΡ‚ΡŒ ΠΊΠΎΠΌΠΌΠ΅Π½Ρ‚Π°Ρ€ΠΈΠΉ

Π’Π°Ρˆ адрСс email Π½Π΅ Π±ΡƒΠ΄Π΅Ρ‚ ΠΎΠΏΡƒΠ±Π»ΠΈΠΊΠΎΠ²Π°Π½. ΠžΠ±ΡΠ·Π°Ρ‚Π΅Π»ΡŒΠ½Ρ‹Π΅ поля ΠΏΠΎΠΌΠ΅Ρ‡Π΅Π½Ρ‹ *